ML12073A203

From kanterella
Jump to navigation Jump to search
Initial Exam 2011-302 Draft RO Written Exam (Part 3 of 3)
ML12073A203
Person / Time
Site: Watts Bar Tennessee Valley Authority icon.png
Issue date: 04/29/2011
From:
Operator Licensing and Human Performance Branch
To:
Tennessee Valley Authority
References
50-390/11-302
Download: ML12073A203 (257)


Text

3-OT-SYSO57P Rev 12 Page 7 of 106 pages V. TRAINING OBJECTIVES: (continued)

A R S S U 0 R T 0 0 A X X X 16. Correctly locate control room controls and indications associated with the 1 25v DC Vital system, including:

a. Alarms
b. Voltmeters
c. Ammeters X X X X 17. Describe the in-plant location of:
a. 1 25v Vital Batteries
b. 1 25v Vital Battery Boards
c. 1 25v DIG Batteries
d. 250v Batteries
e. 250v Battery Boards
f. 250v Turbine Bldg Distribution Bds
g. 1 25v Vital Battery Chargers
h. 125v Vital Inverters
i. 48v Telephone Battery
j. 48v Plant Battery
k. 24v CAP Battery

WBN 10-2011 NRC RO Exam As Submitted 8/15/2011

51. 064 K3.01 051 Given the following:

- Unit 1 trips due to a safety injection actuation.

- Two minutes after the safety injection, offsite power is lost.

- Diesel Generator lA-A trips twelve seconds after connecting to its shutdown board.

Which ONE of the following identifies how the loads being sequenced onto the shutdown board are affected?

A. Load breakers already connected remain connected; load breakers waiting on the sequencer will not close.

B. Load breakers already connected remain connected; load breakers waiting on the sequencer will close upon completion of the time delay.

C Load breakers already connected will trip open; load breakers waiting on the sequencer will not close.

D. Load breakers already connected will trip open; load breakers waiting on the sequencer will close, then trip open, upon completion of the time delay.

Page 133

WBN 10-2011 NRC RO Exam As Submitted 8115/2011 DISTRA CTOR ANAL YSIS:

A. Incorrect, Plausible because there are breakers (such as feeder breakers to the 480v shutdown boards) that do not trip when a 6.9kv shutdown board loses voltage and one of the functions of the undenioltage relays actuated due to the loss of voltage to the board is to lockout the losing circuit of the breakers feeding from the board to prevent closure of the breaker.

B. Incorrect, Plausible because feeder breakers on the board would not trip open like the motor breakers do and the sequencer timers have been initiated and will time out.

C. Correct, After a loss of voltage to the board following the diesel trip, an undervoltage relay on the board will cause all motor breakers that are closed to be tripped open and the losThg circuit of all motor breakers to be locked out preventing breaker closure.

D. lncorrect Plausible because the loss of voltage to the board following the diesel trip results in an undervoltage relay on the board tripping all closed motor breakers and sending a trip signal all motor breakers on the board, so that if they attempted to close, the breaker would trip free. However the relays also lock out the closing circuit of any breaker feeding from the board to prevent any attempt of the breaker to close.

Page 134

WBN 10-2011 NRC RO Exam As Submitted 811512011 Question Number: 51 Tier: 2 Group 2 KIA: 064 K3.01 Emergency Diesel Generator (ED/G) System Knowledge of the effect that a loss or malfunction of the ED/G system will have on the following:

Systems controlled by automatic loader Importance Rating: 3.8/4.1 10 CFR Part 55: 41.7 / 45.6 IOCFR55.43.b: Not applicable K/A Match: K/A is matched because the questions requires knowledge of how a diesel generator failure will affect load that have been connected during a blackout sequence as well as loads waiting on timers to allow the load to be sequenced on to a shutdown board.

Technical

Reference:

1-45W724-1 R25 1-45W760-74-1 R12 Proposed references None to be provided:

Learning Objective: 3-OT-SYS2O1A

1. Identify the 7 plant 6.9kV Bds & buses and describe each:
e. Protection on the supplies and feeders for each Bd.

Cognitive Level:

Higher X Lower Question Source:

New Modified Bank Bank X Question History: WBN Bank question Comments:

Page 135

3-OT-SYS2O1 A Rev 12 Page 4 of 136 pages I. PROGRAM: WATTS BAR OPERATOR TRAINING II. COURSE:

A. License Training B. NOTP (Including Electrical 2B)

C. LICENSED REQUALIFICATION D. NAUO REQUALIFICATION III. TITLE:

6900V AC BOARDS AND BUSES IV. LENGTH OF LESSON:

A. LICENSE TRAINING 2.0 HOURS B. NOTP 4.0 HOURS Licensed & NAUO Requal Times to be determined when Objectives are identified.

V. TRAINING OBJECTIVES:

Identify the 7 plant 6.9kV Bds & buses and describe each:

a. Location and Number of Bds in the plant.
b. Normal, Alternate, & Control power to each Bd.
c. Purpose/function of each Bd.
d. Feeds from each Bd.
e. Protection on the supplies & feeders for each Bd.
f. Transfer schemes by using a breaker schematic.
g. Indications on each Bd.
2. Describe the 6.9kV Electrical System by drawing a simplified diagram including all 6.9kV Bds and buses and supplies to each.
3. Draw and describe a typical 50, 51, 51 X, 83 relay motor protection scheme on a 6.9kV Unit Bd.
4. Explain the Purpose of the 51 G relay.
5. Explain the electrical print for a 6.9kV RCP Bd. Transfer scheme.

3-OT-SYS2O1 A Rev 12 Page 5 of 136 pages V. TRAINING OBJECTIVES: (continued)

A R S S U 0 R T 0 0 A x x x 6. Explain electrical print for a 6.9kV Common Bd. transfer scheme.

I x 7. Discuss how to de-energize a piece of equipment in the following circumstances:

a. Normal
b. Manually
c. Without the equipments breaker, during an emergency x 8. Explain Purpose for the 27R relay on a 6.9kV Unit Bd.

x 9. Discuss how voltage is maintained on the 6.9kV Shutdown Bds.

x 10. Explain the Design Basis of the 6.9kV Shutdown Power system per FSAR 8.3.1.1.

x 1 1. Explain the electrical print for a 6.9kV SD Bd. transfer scheme.

12. Given condition/status of a 6.9kV Shutdown Bd.

system/component and appropriate Tech Specs, determine if operability requirements are met and what action is required.

13. Identify the 6.9kV Shutdown Bd. parameters governed by T/S.
14. Explain the Purpose of the 161 kV Contingency scheme.

(Objective is applicable after Unit 2 operation is established.)

15. List the 3 positions of the 161 kV Contingency switch and explain the function of each. (Objective is applicable after Unit 2 operation is established.)

x x 16. Identify the Arming signals for Load shedding.

(Objective is applicable after Unit 2 operation is established.)

3-OT-SYS2O1 A Rev 12 Page 6 of 136 pages V. TRAINING OBJECTIVES: (continued)

AR S S U OR T 0 0 A X X 17. Evaluate the effects of load shedding on the plant after trip of both units and an undervoltage on either 6.9kV Start Bd.

(Objective is applicable after Unit 2 operation is established.)

X X X 18. Explain the 6.9 kV Equipment Operating Guidelines concerning Motor Operating and Starting Limits.

X X X 19. Describe the specific starting duty limits on the RCPs.

X X X 20. Identify onsite power system Voltage Limits per SOI-21 1.01-04.

X X 21. Identify consequences of abnormal conditions in or improper work on electrical distribution systems, and include what those conditions are. SOER 90-001, Rec. 4 X X 22. Correctly locate control room controls and indications associated with the 6.9kV systems, including:

a. CSSTs A, B, C, D
b. 6.9 kV Start Bus Bds A & B
c. RCP Start Buses A & B
d. RCPBds
e. 6.9 kV Unit Bds
f. 6.9 kV Common Bds
g. 6.9 kV Shutdown Bds X X X 23. As a minimum participant should use prints 15E500-1 & 2 to locate and identify the following:
a. CSSTs A, B, C and D
b. 6.9 kV Start Bus Bds A & B
c. RCP Start Buses A & B
d. RCP Bds
e. 6.9 kV Unit Bds
f. 6.9 kV Common Bds
g. 6.9 kV Shutdown Bds

3-OT-SYS2O1 A Rev 12 Page 7 of 136 pages V. TRAINING OBJECTIVES: (continued)

24. Picking one of the 6.9 kV Bds (Unit, Common, or Shutdown), participant should locate and identify the following:
a. Normal Supply Breaker
b. Alternate Supply Breaker
c. Voltmeters
d. Ammeters
e. Watt-Hr Meter
f. Protective Relays:

(1) 87 Differential (2) 51 N Neutral Overcurrent (3) 51 Overcurrent (4) 27 Undervoltage VI. TRAINING AIDS A. Marker Board and Markers B. Computer and Multimedia Projector VII. TRAINING MATERIALS:

A. WBN Prints: (Prints will vary with objectives covered)

1. 15E500-1,-2
2. 1-45N713
3. 45W714
4. 1-45W715
5. 1-45W720
6. 1-45W721-1,-2
7. 1 -45W724-1
8. 1 -45W760-200-1, -2, -3, -4, -5
9. 1-45W760-201-1, -2, -3
10. 1-45W760-211-1, -2, -3, -4, -6, -17, -18
11. 1 -45W760-68-1, -2, -5, -6
12. 75W502&503 B. Student Training Guide 3-OT-STG-0201A, 6.9kV System

A rr in r -

27S1A (OA & OC) Load shed MA I NTE I FROM 6 sensors set at 70% (-3 second delay)

BD lB PERMIT AUTO

  • 27D1A (0A & OC) Bus voltage less CLOSING OF BKR 1912 than 70% DG start sensor (-.5 second delay)

BETWEEN 3 TO 5 SEC. TRIP BKR 1716,

  • 27DAT, 27DBT, 27DCT 96% degraded 17181932 ALL MOTOR BKRS, ALL P 1932 ELECTRICALLY OPERATED BKRS ON 1932, 51 i 48OV SHDN BDS 1A1A & 1A2A EXCEPT voltage sensor. 2 out of 3 picks up 8,1716. CNTMT AIR RETURN FAN. LOWER COUPT

-ix BETWEEN .5 TO 1 .5 SEC. COOLING FAN AND CRD COOLING START DIESEL GENERATORS FAN: INITIATE BLACKOUT DS-1 & DS-2 timers 1AA,1BB,2AA,2BB SIGNAL AND INTERLOCK ALL SEQUENTIAL TIMERS.

rI j UV ANN. & SIC COMPUTER ANN. ,_..

1912

  • DS1, DS2 Degraded voltage 10 second timers.
  • DS Degraded voltage trip (1 of 2 DS1 or DS2). Trip feeders and initiate load shed.

27LVA, LVB, LVC 87% bus low voltage sensor

  • LV1, LV2 .75 second low voltage timers (picked up by 2 of 3 27LV)

LV Trip Normal, Alternate and Maintenance Bkrs (1 of 2, Lvi or FDR CKT LV2)

Shutdovn Board Key Diagram 45W760-211-1

F rA IT - I fl f

  • AX932 I CX932 Alternate feeder voltage available set at -95% (close interlock for Alternate BRK)
  • AX912 I CX912 DG voltage available BETWEEN 3 TO 5 SEC.

2 1718,1932 ALL MOTOR I

/ ELECTRICALLY OPERATEI set at 95% (DG breaker auto close 480V S[1DN RDS 1A1A 1,,, CNTMT AIR RETURN FAN BETWEEN .5 TO 1.5 SEC. COOLING FAN AND CR0 permissive)

START DIESEL GENERATORS FAN: INITIATE BLACKO 1AA,1B8,2AA,2BB SIGNAL AND INTERLOCK UV ANN. & SIC COMPUTER SEQUENTIAL TIMERS.

ANN.

  • 59 Overvoltage annunciator input set at 105% (8 second time delay)
  • R716, R718, R932 Auxiliary relays on Maint., Norm, & Alternate BKR trip circuits. Energize when breaker handswitch placed to CLOSE.

Energized to remove the trips from the DV, LV, and load shed relays to allow operations to energize a dead SD Bd from offsite power source.

Shutdovn Board Key Diagram 45W760-211-1

WBN 10-2011 NRC RO Exam As Submitted 811512011

52. 073 K1.01 052 Given the following:

- Annunciator 186A, MCR INTAKE O-RM-90-1 25/1 26 RAD HI, alarms.

- Both O-RM-90-125, MCR INTAKE and O-RM-90-126, MCR INTAKE have the RED light LIT on O-M-12.

Which ONE of the following describes the Main Control Room ventilation alignment?

The MCR is maintained at a A. positive pressure by the Main Control Room Air Handling Units.

B. negative pressure by the Main Control Room Air Handling Units.

C. negative pressure by the Control Building Emergency Cleanup Fans.

D positive pressure by the Control Building Emergency Air Pressurization Fans.

Page 136

WBN 10-2011 NRC RO Exam As Submitted 8115/2011 DISTRA CTOR ANAL YSIS:

A. Incorrect, RM-90-125 and 126 initiate a CR!. A CR! isolates the ductwork from the Control Building Air Pressurization Fans and starts the Control Building Emergency Air Pressurization Fans. A positive pressure is maintained to minimize outside contaminants from enterhg the MCR. Plausible since this is the normal ventilation fiowpath/pressure condition in MCR.

B. Incorrect, RM-90-125 and 126 initiate a CR!. A CR! isolates the ductwork from the Control Building Air Pressurization Fans and starts the Control Building Emergency Air Pressurization Fans. A positive pressure is maintained to minimize outside contaminants from entering the MCR. Plausible since other areas (e.g. Aux Bldg) are maintained at a slight negative pressure during normal and abnormal conditions.

C. Incorrect, RM-90-125 and 126 initiate a CR!. A CR! isolates the ductwork from the Control Building Air Pressurization Fans and starts the Control Building Emergency Air Pressurization Fans. A positive pressure is maintained to minimize outside contaminants from entering the MCR. Plausible since other areas (e.g. Aux Bldg) are maintained at a slight negative pressure during normal and abnormal conditions.

D. Correct, RM-9O-125 and 126 initiate a CR!. A CR! isolates the ductwork from the Control Building Air Pressurization Fans and starts the Control Building Emergency Air Pressurization Fans. A positive pressure is maintained to minImize outside contaminants from entering the MCR.

Question Number: 52 Tier: 2 Group 1 K!A: 073 K1.01 Process Radiation Monitoring (PRM) System Knowledge of the physical connections and/or cause-effect relationships between the PRM system and the following systems:

Those systems served by PRMs Importance Rating: 3.6 / 3.9 10 CFR Part 55: 41.2 to 41.9 / 45.7 to 45.8 10CFR55.43b: Not applicable K/A Match: This question matches the K/A by having the candidate determine the cause-effect of Rad Monitors 0-RM-90-125 & 126 and the Control Room Ventilation system.

Page 137

WBN 10-2011 NRC RO Exam As Submitted 8/1512011 Technical

Reference:

1-47W61 1-31-1 R23 SOl-31.01, Control Building HVAC System, Rev. 0051 N3-300B-4002, Control Building Heating, Ventilation, Air Conditioning, and Air Cleanup System, Revision 16 SO1-30.05, Auxiliary Bldg HVAC Systems, Revision 0051 Proposed references None to be provided:

Learning Objective: 3-OT-SYSO3IA

3. Describe the ventilation flow path provided by the control building ventilation system during normal and emergency operation.

Cognitive Level:

Higher Lower X Question Source:

New Modified Bank Bank X Question History: SQN bank question CB Vent-B.4.G #1 used on the SQN SEPT 2010 NRC exam. Made wording changes in the stem, relocated the correct answer and changed one distractor. Modification is not significant.

Comments:

Page 138

TVA

Title:

CONTROL BUILDING HEATING, VENTILATING, AIR N3-3OCB-4002 CONDITIONING, AND AIR CLEANUP SYSTEM 1.0

SUMMARY

1.1 General Description The Control Building Heating, Ventilating, Air-Conditioning, and Air Cleanup (HVACAC) system consists of the following subsystems:

a. Main Control Room (MCR) air-conditioning (including the Technical Support Center)
b. Deleted
c. Control Building emergency pressurization
d. Control Building emergency air cleanup
e. Electrical Board Rooms (EBR) Air Conditioning
f. Spreading room ventilation
g. Battery rooms exhaust
h. Toilet and locker rooms exhaust
i. Deleted
j. Humidification
k. Computer room supplemental air conditioning
1. Communications room supplemental Air Conditioning Refer to Section 3.0, System Description, and Section 8.0 Figures for description of each subsystem.

The Control Building HVACAC system is designed to maintain the temperature and humidity in the building for personnel comfort, protection and operation of plant controls, and to provide safe, uninterrupted occupancy of the NCR habitability zone (MCRHZ) during normal, accident, and postaccident recovery conditions. The MCR equipment is capable of operating within the temperature range listed in WB-DC-40-42 (Ref. 7.2.6) . The MCRHZ, with a total volume of approximately 260,000 ft . includes all rooms on El 755.0 of CB.

3 During normal plant operation, outside air is supplied to the control building to replace exhausted air and to makeup for outleakage from the building. To

,m minimize air inleakage, the Control Building Habitability Zone is maintained

\ at a positive pressure with respect to adj acent spaces and the outdoors. Upon

.j ,,\ receipt of a Safety Injection Signal (SIS) or upon indication of high radiation or smoke concentrations in the outside air supply to the building, the MCRHZ is automatically isolated. The system maintains the MCRHZ at the positive pressure relative to the pressures of the outdoors, surroundings and the adjacent shutdown board room by recirculating the air through high-efficiency particulate air (HEPA) filters and charcoal adsorbers, and supplying a small amount of outside air for pressurization.

The MCRHZ may be manually isolated at any time by the control room operator.

1

TVA

Title:

CONTROL BUILDING HEATING, VENTILATING, AIR N3-3OCB-4002 CONDITIONING, AND AIR CLEANUP SYSTEM Relay Room, DPSO Engineers Shop, Operations Office, Assistant Shift engineers office, technical support center, NRC office, conference rooms, kitchen, toilet, locker rooms, and the mechanical equipment room at El 755.0. Air is taken from the MCRHZ, filtered and cooled by the operating AHU, then returned to the MCRHZ. Each AHU is belt-driven, and is supplied with isolation dampers which close when the AHU is not operating. Makeup air from outside is drawn by AHU through the return ductwork for pressurization of MCRHZ during normal operation, or by the emergency pressurization subsystem during isolation conditions.

The MCR A/C subsystem operates during normal and isolation modes. It provides a controlled environment for the operators and for the equipment in the MCRHZ spaces. This subsystem is safety-related.

Thermostats in each train of this subsystem are designed to maintain the MCRHZ between 60°F and 104°F during all modes. One train operates while the other is on standby. The standby train will start if sensors on the operating AHU detect low airflow, high AHU inlet air temperature, or low differential pressure across the operating chilled water pump. Failure of the operating train for any of these cases is alarmed in the MCR.

Humidistats in main supply ducts are designed to maintain the MCRHZ relative humidity at approximately 50% during normal operation.

During isolation conditions, CR1 dampers close to isolate the MCRHZ from the outside environment. These dampers, which are tight sealing with resilient seals along blade edges are located in various Control Building HVACAC subsystems, and are listed in Section 9.0, Table 9.1. The dampers automatically close, within 14 seconds (Ref. 7.4.19) for the following conditions: high radiation in the outside air intake, smoke concentrations in the intake air supply, or receipt of a Safety Injection Signal (SIS) . Isolation valves FCV-03l-3 & -4 should not close sooner than 11 seconds (Ref. 7.4.19). However, valves 0-FCV-3l-3 & -4 may close sooner than 11 seconds if a positive MCRHZ pressure of 0.125 W.G. minimum is maintained during the transition from the normal to CR1 mode. See Section 6.0 for details. In addition, the CR1 signal automatically de-energizes both trains of Shutdown Board Rooms Pressurizing Supply Air Fans B-A, B-B, A-A and C-B in the Auxiliary Building El 757.0. The MCRHZ can also be manually isolated by the operator. Upon CR1 reset, the outside air supply and exhaust isolation dampers open. This signal can be reset only if the initiating signal no longer exists.

3.1.2 Deleted.

3.1.3 Control Building Emergency Pressurization Subsystem (Ref. 7.4.14)

The control building emergency pressurization subsystem consists of two 100% trains, designed to supply a reduced stream of outside air to the MCR A/C system to keep the MCRHZ area at 0.125 WG minimum positive pressure with respect to outdoors and all surrounding areas (for all modes of operation except during a tornado warning) . During CR1 operation mode, this positive static pressure will prevent the infiltration of radioactive gases from the areas surrounding the MCRHZ. These areas are the Turbine Building (atmospheric pressure), El 757.0 of the Auxiliary Building (atmospheric pressure), and the spreading room. Each train of the emergency pressurization subsystem consists of a direct-driven vane-axial fan and associated ductwork, dampers, instrumentation, and controls.

9

TVA

Title:

CONTROL BUILDING HEATING, VENTILATING, AIR N3-3OCB-4002 CONDITIONING, AND AIR CLEANUP SYSTEM Differential pressure indication between the MCRHZ and the Cable Spreading Room, MCRHZ and outside atmosphere, MCRHZ and Stairwell Cl, and MCRHZ and Auxiliary Building 6.9 KV Shutdown Board Rooms El. 757.0 are provided to insure that the required positive pressure is maintained in the MCRHZ by MCR A/C subsystem during normal plant operation (except during a tornado warning) . The time delay relay in the differential pressure indicating circuit for MCRHZ pressurization is provided to eliminate nuisance alarm in the MCR. Inadequate differential pressure in MCR will alarm.

A flow switch on the air discharge of each AND starts the standby train and shuts down the energized train on low flow. Failure of the operating train is alarmed in the MCR.

3.3.1.2 Deleted 3.3.1.3 Control Building Emergency Pressurization Subsystem The status of each emergency pressurization fan and damper is indicated by lights in the MCR.

During the CR1 the positive pressure is maintained in the MCRHZ by Control Building Emergency pressurization subsystem. If any one of four pressure differential transmitters sensing the MCR pressure relative to the outdoor and surrounding areas senses below 0.125 WG, its pressure differential switch sends a signal to automatically start the standby emergency pressurization fan to correct the MCRHZ pressure back to above 0.12511 WG

/7 ()1 positive pressure.

3.3.1.4 Control Building Emergency Air Cleanup Subsystem Status of each air cleanup fan and its associated dampers is indicated by lights in the MCR.

During the CR1 the positive pressure is maintained in the MCRHZ by Control Building Emergency pressurization subsystem (except during a tornado warning) . If any one of four pressure differential transmitters sensing the MCR pressure relative to the outdoor and surrounding areas senses below the 0.125 WG, its pressure differential switch sends a signal to automatically start standby emergency pressurization fan to correct the MCRHZ pressure back to above 0.125 WG positive pressure.

A temperature indicator is located on the outlet of each emergency air cleanup filter assembly charcoal adsorber bed.

Pressure differential indicators across the HEPA filter and the charcoal adsorber allow periodic surveillance of dust loadings and pressure drops on individual components in filter trains.

21

WBN Control Building HVAC System SOI-31.01 Unit I Rev. 0051 Page 37 of 107 6.0 NORMAL OPERATION A. Fresh air is continuously supplied to the Control Bldg to replace exhausted air and to provide makeup for outleakage. The Control Bldg is maintained at

+1/8 in. H 0 relative to atmosphere to minimize air inleakage. The Control Bldg 2

W t.V air-conditioned spaces are maintained 80°F and 5O% relative humidity for protection of instruments and for the comfort and safety of the operators.

B. Spreading Room is maintained at a negative press relative to Control Bldg.

C. Control Bldg Pressurization Fans have fan blades positioned in parallel with air flow. Control Dampers 0-FCO-31-01, -02, -01A, -02A are secured full open.

Balancing Damper and ductwork are installed from Pressurization Fans discharge duct to Air Handling Unit inlet duct.

D. Computer Room Supplemental Cooling is provided from either of two AHU5 located on a platform in EL 708 corridor outside computer room. Each AHU can receive safety-related (trained) chilled water from the EBR chiller piping systems. Isolation valves shall be aligned such that only one supplemental chilled water loop shall be in-service at any time. The chilled water loop which is NOT in-service shall have its isolation valves closed to preclude cross-connecting the EBR Train A and Train B chilled water headers. The temperature controller should be set for 68 degrees.

E. Applicable Attachments will be performed at discretion of Operations Superintendent or designee. Attachments will normally be performed for System Alignment Verification in Mode 5 or whenever alignment verification is needed.

F. The MCR steam generator (humidifier) shall be in service any time the relative humidity in the MCR is <50% as indicated at the MCR AHUs. Humidity levels of less than 20% may affect function of touch screens and will affect operability of Unit 2 Common Q PAMS.

  • WBN Auxiliary Bldg HVAC Systems SOl-30.05 Unit I Rev. 0051 Page 8 of 111 30 PRECAUTIONS AND LIMITATIONS A. During winter, Unit Operator should be notified before starting an Aux Bldg Gen Supply Fan due to larger heat load demand.

B. Do NOT operate AB Gen Supply Fans when outside temp is 40°F or less, UNLESS the Aux Steam system is in operation to supply building heat to fan inlet coils to prevent coil freeze-up.

C. Aux Bldg (AB) Gen Exh Fan controllers 1-FCO-30-164 (Fan IA) or -168 (Fan I B), and 2-FCO-30-273 (Fan 2A) or -277 (Fan 2B), should automatically maintain -0.25 in. HO in Aux Bldg relative to atmosphere.

2 D. Fuel Handling Exhaust Fan controllers 0-FCO-30-169 (Fan A) and/or 0-FCO-30-1 70 (Fan B), should automatically maintain -0.25 in. H0 in 2

Aux Bldg relative to atmosphere.

E. Starting/stopping supply and/or exhaust fans (Aux Bldg or Fuel Handling) could result in Aux Bldg pressure NOT being maintained slightly negative, which could lead to unmonitored release paths. Negative pressure should be maintained in the Aux Bldg at all times (ABGTS may be used if required)[c2]

F. Starting/stopping of AB Gen Supply, Exhaust, and Fuel Handling Exhaust Fans causes large Aux Bldg press transients, which can cause personnel injury when entering and exiting Aux Bldg doors.

G. Changes in Aux Bldg HVAC may change radiological conditions in specific areas. Before changing HVAC line-ups or starting/stopping fans, notify Radiological Protection.

H. Prolonged operation of A and B fans on a single unit of AB Gen Supply, Exhaust, and Fuel Handling Exhaust Fans can cause duct work and damper damage due to excessive pressure.

I. Work in a Radiological Control Area (RCA) requires the use of existing RWPs, and may require additional ALARA Preplans. Failure to follow posted Rad control requirements can cause unnecessary radiation exposure. Radiation Protection should be notified of work having the potential to change radiological conditions.

J. Notify Instrument Maintenance to ensure required instruments are placed in service as necessary to support system operation.

073 K1.01 052 3Q ((-i7X jc7i-c-Given the following plant alarms:

- O-RA-90-125A, MAIN CNTRL RM INTAKE MON HIGH RAD

- O-RA-90-126A, MAIN CNTRL RM INTAKE MON HIGH RAD Which ONE of the following describes the Main Control Room ventilation alignment?

The MCR is maintained at a A. positive pressure by the Main Control Room Air Handling Units.

B. negative pressure by the Main Control Room Air Handling Units.

C a positive pressure by the Control Building Emergency Air Pressurization Fans.

D. a negative pressure by the Control Building Emergency Air Pressurization Fans.

3-OT-SYSO3 1 A Revision 7 Page 5of38 PROGRAM Watts Bar Operator Training II. COURSES A. License Training B. NOTP C. License Operator Requalification D. NAUO Requalification III. TITLE Control Building Heating and Ventilation IV. LENGTH OF LESSON A. Licensed Training 2 hours2.314815e-5 days <br />5.555556e-4 hours <br />3.306878e-6 weeks <br />7.61e-7 months <br /> B. NOTP 3 hours3.472222e-5 days <br />8.333333e-4 hours <br />4.960317e-6 weeks <br />1.1415e-6 months <br /> V. TRAINING OBJECTIVES AR S S U OR T 0 0 A X X X X 1. State the purpose of the control building ventilation system.

X X X X 2. State the purpose of the control building emergency ventilation system.

X X X X 3. Describe the ventilation flow path provided by the control building ventilation system during normal and emergency operation.

X X X X 4. List the signals which initiate a main control room isolation.

X X X X 5. List events that occur during a main control room isolation.

X X X X 6. Identify the location of the equipment required for a lineup of the control building ventilation system.

X X X X 7. Describe the steps required to lineup and start a main control room air handling unit.

3-OT-SYSO3 1 A Revision 7 Page 6of38 A R S S U 0 R T 0 0 A X X X X 8. Describe the common factor in the four instances of safety system inoperability described in SER 11 -92.

VI. TRAINING AIDS A. Whiteboard and markers.

B. Multimedia projector.

C. Overhead projector VII. MATERIALS One copy of each of the following for each participant as required:

Appendices None B. Attachments None VIII. REFERENCES N3-3OCB-4002 Control Building Heating, Ventilating, Air Conditioning and 15 Air Cleanup System 6.4 Habitability Systems 7 9.4.1 Control Room Area Ventilation System 7

WBN 10-2011 NRC RO Exam As Submitted 8/15/2011

53. 076 A4.01 053 Given the following conditions:

- Unit I is operating at 100% power.

- ERCW pump handswitch status:

- A-AisinP-T-L

- B-A is in A-AUTO and pump stopped.

- C-A is in A-AUTO and pump running.

- D-A is in A-AUTO and pump stopped.

- Emergency Power Selector Switches are positioned to B-A and C-A, respectively.

- A loss of offsite power occurs.

Which ONE of the following identifies...

(1) if a Tech Spec LCO 3.7.8 Essential Raw Cooling Water (ERCW) System, entry was required prior to the loss of offsite power and (2) how the indication on on 0-Pl-67-18A, A ERCW SUP HDR PRESS, will respond after the blackout sequence relays complete restoration of loads on the Shutdown Boards?

LCO entry required 0-Pl-67-18A A. YES Same as before loss of offsite power.

B. YES Higher than before loss of offsite power.

C. NO Same as before loss of offsite power.

D NO Higher than before loss of offsite power.

Page 139

WBN 10-2011 NRC RO Exam As Submitted 811512011 DISTRACTOR ANAL YSIS:

A. lncorrect, Plausible because the A-A pump is inoperable as aligned in the stem (but only one of the paired pumps is required) and the demand on the Train A system will be the same after the blackout sequence but an additional pump will be started to raise the pressure in the system.

B. Incorrect, Plausible because the A-A pump is inoperable as aligned in the stem (but only one of the paired pumps is required) and since an additional pump is started due to the blackout sequence, without an increase in system demand, the system pressure will be higher.

C. Incorrect, Plausible because no LCO entry being required prior to the blackout signal is correct and the demand on the Train A system will be the same after the blackout sequence. But an additional pump will be started to raise the pressure in the system.

D. Correct, the alignment met the LCO prior to the loss of offsite power, so no entry was required. The requirement is for that either the A-A pump or the B-A pump to be operable. The B-A pump, while not in service, will start on a blackout or a Safety injection signal. With the current alignment, after the blackout sequence was completed there would be 2 pumps running on Train A as opposed to only one pump prior to the blackout. Thus the pressure would be and indicate higher after the blackout sequence.

Question Number: 53 Tier: 2 Group 1 KIA: 076 A4.01 Service Water System (SWS)

Ability to manually operate and/or monitor in the control room:

SWS pumps Importance Rating: 2.9 / 2.9 10 CFR Part 55: 41.7 / 45.5 to 45.8 IOCFR55.43.b: Not applicable KIA Match: K/A is matched because the question requires the ability to identify how the ERCW pump parameters would be expected to change following restart during a blackout condition (monitoring) and the affect the event would have on the ERCW pressure (monitoring) after the board was restored.

Technical

Reference:

Tech Spec 3.7.8, Essential Raw Cooling Water Page 140

WBN 10=2011 NRC RO Exam As Submitted 8/15/2011 Technical

Reference:

Tech Spec 3.7.8, Essential Raw Cooling Water (ERCW), Amendment 69 and the associated Bases 1-47W611-67-1 Rh Proposed references None to be provided:

Learning Objective: 3OT-SYSO67A

13. Given a loss of power, determine the correct response of the ERCW System including:
b. ERCW Pumps.
24. Regarding Technical Specifications and Technical Requirements for this system:
c. Given a status set of plant conditions, apply the appropriate Technical Specifications and Technical Requirements.

Cognitive Level:

Higher X Lower Question Source:

New X Modified Bank Bank Question History: New question for the WBN 10/2011 NRC exam Comments:

Page 141

ERCW 3.7.8 3.7 PLANT SYSTEMS 3.7.8 Essential Raw Cooling Water (ERCW) System LCO 3.7.8 Two ERCW trains shall be OPERABLE.

APPLICABILITY: MODES 1,2, 3, and 4.

ACTIONS CONDITION REQUIRED ACTION COMPLETION TIME A. One ERCW train A.1 NOTES inoperable, other than for 1. Enter applicable Condition C. Conditions and Required Actions of LCO 3.8.1, AC Sources- Operating, for emergency diesel generator made inoperable by ERCW.

2. Enter applicable Conditions and Required Actions of LCO 3.4.6, RCS Loops-MODE 4, for residual heat removal loops made inoperable by ERCW.

Restore ERCW train to 72 hours8.333333e-4 days <br />0.02 hours <br />1.190476e-4 weeks <br />2.7396e-5 months <br /> OPERABLE status.

(continued)

Watts Bar-Unit 1 3.7-19 Amendment 69

ERCW 3.7.8 ACTIONS (continued)

CONDITION REQUIRED ACTION COMPLETION TIME B. Required Action and B.1 Be in MODE 3. 6 hours6.944444e-5 days <br />0.00167 hours <br />9.920635e-6 weeks <br />2.283e-6 months <br /> associated Completion Time of Condition A not AND met.

B.2 Be in MODE 5. 36 hours4.166667e-4 days <br />0.01 hours <br />5.952381e-5 weeks <br />1.3698e-5 months <br /> C. Two Train A ERCW pumps C.1 Align the operable pumps (C-A 72 hours8.333333e-4 days <br />0.02 hours <br />1.190476e-4 weeks <br />2.7396e-5 months <br /> (A-A and B-A) inoperable and D-A) to concurrently and two Train A ERCW autostart from the 2A-A 6.9 KV pumps operable (C-A and Shutdown Board.

D-A).

  • AND C.2 Restore at least one of the 10 days pumps (A-Aor B-A) to OPERABLE status.

SURVEILLANCE REQUIREMENTS SURVEILLANCE FREQUENCY SR 3.7.8.1 NOTE Isolation of ERCW flow to individual components does not render the ERCW inoperable.

Verify each ERCW manual, power operated, and 31 days automatic valve in the flow path servicing safety related equipment, that is not locked, sealed, or otherwise secured in position, is in the correct position.

(continued)

This CONDITION will apply until the A-A or B-A pump is repaired and declared operable or until July 31, 2008, whichever occurs first.

Watts Bar-Unit 1 3.7-20 Amendment 69

ERCW 3.7.8 SURVEILLANCE REQUIREMENTS (continued>

. SURVEILLANCE FREQUENCY SR 3.7.8.2 Verify each ERCW automatic valve in the flow path 18 months that is not locked, sealed, or otherwise secured in position, actuates to the correct position on an actual or simulated actuation signal.

SR 3.7.8.3 Verify each ERCW pump starts automatically on an 18 months actual or simulated actuation signal.

Watts Bar-Unit 1 3.7-20a

ERCW B 31.8 B 3.7 PLANT SYSTEMS B 3.7.8 Essential Raw Cooling Water (ERCW) System BASES BACKGROUND The ERCW provides a heat sink for the removal of process and operating heat from safety related components during a Design Basis Accident (DBA) or transient. During normal operation, and a normal shutdown, the ERCW System also provides this function for various safety related and nonsafety related components. The safety related function is covered by this LCO.

The shared ERCW system consists of eight 50% ERCW pumps, four traveling water screens, four screen wash pumps, four strainers, associated piping, valves, and instrumentation.

Water for the ERCW system enters two separate sump areas of the pumping station through four traveling water screens, two for each sump. Four ERCW pumping units, all on the same plant train, take suction from one of the sumps, and four more on the opposite plant train take suction from the other sump. One set of pumps and associated equipment is designated Train A, and the other Train B. These trains are redundant and are normally maintained separate and independent of each other. Each set of four pumps discharges into a common manifold, from which two separate headers (1A and 2A for Train A, and 1 B and 2B for Train B) each with its own automatic backwashing strainer, supply water to the various system users. Two pumps per train are adequate to supply worst case conditions. Two pumps per train are aligned to receive power from different diesel generators. Operator designated pumps and valves are remote and manually aligned, except in the unlikely event of a loss-of-coolant accident (LOCA). The pumps are automatically started upon receipt of a safety injection (SI) signal, and some essential valves are aligned to their post-accident positions. Some manual realignments of motor-operated valves (MOVs) are necessary. The ERCW System also provides emergency makeup to the Component Cooling System (CCS) and is the backup water supply to the Auxiliary Feedwater System.

Additional information about the design and operation of the ERCW, along with a list of the components served, is presented in the FSAR, Section 9.2.1 (continued)

Watts Bar-Unit 1 B 3.7-43

ERCW B 3.7.8 BASES BACKGROUND (Ref. 1). The principal safety related function of the ERCW System is the (continued) removal of decay heat from the reactor via the CCS.

APPLICABLE The design basis of the ERCW System is for one ERCW train, in conjunction SAFETY ANALYSES with the CCS and a 100% capacity Containment Spray System and Residual Heat Removal (RHR), to remove core decay heat following a design basis LOCA as discussed in the FSAR, Section 9.2.1 (Ref. 1). This prevents the containment sump fluid from increasing in temperature during the recirculation phase following a LOCA and provides for a gradual reduction in the temperature of this fluid as it is supplied to the Reactor Coolant System by the ECCS pumps. The ERCW System is designed to perform its function with a single failure of any active component, assuming the loss of offsite power.

The ERCW System, in conjunction with the CCS, also cools the unit from RHR, as discussed in the FSAR, Section 5.5.7, (Ref. 2) entry conditions to MODE 5 during normal and post accident operations. The time required for this evolution is a function of the number of CCS and RHR System trains that are operating.

One ERCW train is sufficient to remove decay heat during subsequent operations in MODES 5 and 6. This assumes a maximum ERCW temperature of 85°F occurring simultaneously with maximum heat loads on the system.

The ERCW System satisfies Criterion 3 of the NRC Policy Statement.

LCO Two ERCW trains are required to be OPERABLE to provide the required redundancy to ensure that the system functions to remove post accident heat loads, assuming that the worst case single active failure occurs coincident with the loss of offsite power.

An ERCW train is considered OPERABLE during MODES 1, 2, 3, and 4 when:

(continued)

Watts Bar-Unit 1 B 3.7-44

ERCW B 3.7.8 BASES LCO a. Two pumps, aligned to separate shutdown boards, are OPERABLE; and (continued)

b. The associated piping, valves, heat exchanger, and instrumentation and controls required to perform the safety related function are OPERABLE.

APPLICABILITY In MODES 1, 2, 3, and 4, the ERCW System is a normally operating system that is required to support the OPERABILITY of the equipment serviced by the ERCW System and required to be OPERABLE in these MODES.

In MODES 5 and 6, the OPERABILITY requirements of the ERCW System are determined by the systems it supports.

ACTIONS A.1 If one ERCW train is inoperable, action must be taken to restore OPERABLE status within 72 hours8.333333e-4 days <br />0.02 hours <br />1.190476e-4 weeks <br />2.7396e-5 months <br />. In this Condition, the remaining OPERABLE ERCW train is adequate to perform the heat removal function. However, the overall reliability is reduced because a single failure in the OPERABLE ERCW train could result in loss of ERCW System function. Required Action A.1 is modified by two Notes.

The first Note indicates that the applicable Conditions and Required Actions of LCO 3.8.1, AC Sources Operating, should be entered if an inoperable ERCW train results in an inoperable emergency diesel generator. The second Note indicates that the applicable Conditions and Required Actions of LCO 3.4.6, RCS Loops MODE 4, should be entered if an inoperable ERCW train results in an inoperable decay heat removal train. This is an exception to LCO 3.0.6 and ensures the proper actions are taken for these components. The 72 hour8.333333e-4 days <br />0.02 hours <br />1.190476e-4 weeks <br />2.7396e-5 months <br /> Completion Time is based on the redundant capabilities afforded by the OPERABLE train, and the low probability of a DBA occurring during this time period.

B.1 and B.2 If the ERCW train cannot be restored to OPERABLE status within the associated Completion Time, the plant must be placed in a MODE in which the LCO does not apply. To achieve this status, the plant must be placed in at least (continued)

Watts Bar-Unit 1 B 3.7-45

ERCW B 3.7.8 BASES ACTIONS B.1 and B.2 (continued)

MODE 3 within 6 hours6.944444e-5 days <br />0.00167 hours <br />9.920635e-6 weeks <br />2.283e-6 months <br /> and in MODE 5 within 36 hours4.166667e-4 days <br />0.01 hours <br />5.952381e-5 weeks <br />1.3698e-5 months <br />. The allowed Completion Times are reasonable, based on operating experience, to reach the required plant conditions from full power conditions in an orderly manner and without challenging plant systems.

SURVEILLANCE SR 3.7.8.1 REQUIREMENTS This SR is modified by a Note indicating that the isolation of the ERCW System components or systems may render those components inoperable, but does not affect the OPERABILITY of the ERCW System.

Verifying the correct alignment for manual, power operated, and automatic valves in the ERCW System flow path provides assurance that the proper flow paths exist for ERCW System operation. This SR does not apply to valves that are locked, sealed, or otherwise secured in position, since they are verified to be in the correct position prior to being locked, sealed, or secured. This SR does not require any testing or valve manipulation; rather, it involves verification that those valves capable of being mispositioned are in the correct position. This SR does not apply to valves that cannot be inadvertently misaligned, such as check valves.

The 31 day Frequency is based on engineering judgment, is consistent with the procedural controls governing valve operation, and ensures correct valve positions.

SR 3.7.8.2 This SR verifies proper automatic operation of the ERCW System valves on an actual or simulated actuation signal. The ERCW System is a normally operating system that cannot be fully actuated as part of normal testing. This Surveillance is not required for valves that are locked, sealed, or otherwise secured in the required position under administrative control. The 18 month Frequency is based on the need to perform this Surveillance under the conditions that apply (continued)

Watts Bar-Unit 1 B 3.7-46

ERCW B 3.7.8 BASES SURVEILLANCE SR 3.7.8.2 (continued)

REQUIREMENTS during a unit outage and the potential for an unplanned transient if the Surveillance were performed with the reactor at power. Operating experience has shown that these components usually pass the Surveillance when performed at the 18 month Frequency. Therefore, the Frequency is acceptable from a reliability standpoint.

SR 3.7.8.3 This SR verifies proper automatic operation of the ERCW pumps on an actual or simulated actuation signal. The ERCW System is a normally operating system that cannot be fully actuated as part of normal testing during normal operation.

The 18 month Frequency is based on the need to perform this Surveillance under the conditions that apply during a unit outage and the potential for an unplanned transient if the Surveillance were performed with the reactor at power.

Operating experience has shown that these components usually pass the Surveillance when performed at the 18 month Frequency. Therefore, the Frequency is acceptable from a reliability standpoint.

REFERENCES 1. Watts Bar FSAR, Section 9.2.1, Essential Raw Cooling Water.

2. Watts Bar FSAR, Section 5.5.7, Residual Heat Removal System.

Watts Bar-Unit 1 B 3.7-47

3-OT-SYSO67A Revision 12 Page 5 of 41 I. PROGRAM Waifs Bar Operator Training II. COURSES A. License Training B. Non-License Training III. TITLE Essential Raw Cooling Water System IV. LENGTH OF LESSON A. Licensed Training 2 hour2.314815e-5 days <br />5.555556e-4 hours <br />3.306878e-6 weeks <br />7.61e-7 months <br /> B. Non-Licensed Training 3.5 hours5.787037e-5 days <br />0.00139 hours <br />8.267196e-6 weeks <br />1.9025e-6 months <br /> V. TRAINING OBJECTIVES 0

D 1-Cl)

X X X 1. State the function of the Essential Raw Cooling Water System in accordance with the system description.

X X X 2. State the design basis of the Essential Raw Cooling Water System in accordance with FSAR Section 9.2.1.

X X X X 3. Describe the ERCW System flow path from the river to the cooling tower basin and discharge holding pond including:

a. Interfaces
b. Major components
c. Paths to and from the Auxiliary Building X X X X 4. Deleted.

X X X X 5. Deleted.

X X 6. Explain the purpose of Biocide injection.

X X 7. Discuss the precaution associated with contact of Bromide with the j body X 8. State the ERCW System normal discharge path and given a failure X

J X of the path, discuss the alternate discharge paths.

Describe the purpose of the hydraulic gradient.

- A 9.

X X X X 10. Describe the purpose of the discharge overflow structure.

X X IX X 11. Deleted.

3-OT-SYSO67A Revision 12 Page 6 of 41 0 0 D 0 a:

a: C,) Cl)

X X X X 12. Deleted.

X X X 13. Given a loss of power, determine the correct response of the ERCW System including:

a. C CCS Heat Exchanger outlet valves.
b. ERCW Pumps.

X X X X 14. Explain how and when the ERCW System would be used to feed the Steam Generators.

X X X X 15. List the places where the ERCW System can be interconnected to another system with a spool piece.

X X X X 16. Given a failure in the ERCW System, determine how radiation would be detected.

X X X 17. Given a loss of instrument air/control power, determine the effect on temperature control valves.

X X X X 18. [Identify 2 indications of biofouling in heat exchangers as stated in SOER 84-1. (REC. #4)]

X X X X 19. Describe the ERCW pumps including capacity, logic, power supplies, type, lubrication, heat removal means and alarm setpoints.

X X X X 20. Describe the normal and backwash flow paths of the ERCW strainers.

X X X X 21. Describe the purpose of the ERCW prelube, what components it supplies and from where it is supplied.

X X X X 22. Describe the operation of the traveling screen system, normal and backwash, and how P is determined.

X X X X 23. Briefly describe how to place the ERCW System in service per SQl-67.01.

X X X 24. Regarding Technical Specifications and Technical Requirements for this system:

a. Identify the conditions and required actions with completion time of one hour or less.
b. Explain the Limiting Conditions for Operation, Applicability, and Bases.
c. Given a status/set of plant conditions, apply the appropriate Technical Specifications and Technical Requirements.

X X X X 25. List the ERCW parameters governed by Tech Specs.

3-OT-SYSO67A Revision 12 Page 7 of 41 0 0<

DO a:

< a: Cl) Cl)

X X X X 26. Correctly locate control room controls and indications associated with the Essential Raw Cooling Water System, including:

a. Pump and Selector Switch Controls
b. Header Isolation Valves and Flow Indications
c. Supply and Discharge Headers
d. CCS Heat Exchanger Alignments X X X X 27. [Identify the action(s) to be taken by the operator if significant heat exchanger degradation due to fouling is detected.

(SOER 84-1, Rec. 4)]

3-OT-SYSO67A Revision 12 Page 22 of 41

4. Discuss the switchgear mounted controls associated with ERCW pumps.

LU I Three position START/STOP/A AUTO control

-J switch.

U)

  • START With switchgear selector switch in Auxiliary and no emergency signals, pump will start.
  • STOP With switchgear selector switch in Auxiliary pump will stop.
  • A-AUTO With the switchgear selector switch in Auxiliary, pump selected, and the following, the pump will auto start:
  • Blackout with voltage available after a 20 second delay.
5. Discuss the function and operation of the ERCW EROW Pump Selector Switch Pump Diesel Generator Power Selector Switch.

Lii

  • There are eight ERCW pumps and four 6.9 KV shutdown boards.

M-27A

  • Two pumps receive power from each shutdown board.
  • To ensure diesel generator loading is not exceeded, only one pump will start on each diesel generator.
  • Selector switch for each pump pair selects the pump that will receive an auto-start signal during an emergency.

WBN 10-2011 NRC RO Exam As Submitted 811512011

54. 078 K2.02 054 Given the following:

- Unit I is in Mode 3 following a reactor trip when a loss of off-site power occurs.

- All Diesel Generators start and restore the shutdown boards.

Which ONE of the following identifies...

(1) how the Control and Station Air Compressor A will respond as the control air pressure is dropping and (2) the 480V Shutdown Board that supplies power supply to the compressor?

A. (1) Compressor will start and load as needed automatically; (2) 480V SD Bd 1A2-A B. (1) Compressor will start and load as needed automatically; (2) 480V SD Bd 2A2-A Cv (1) Compressor must be manually started but will load as needed automatically; (2) 480V SD Bd 1A2-A D. (1) Compressor must be manually started but will load as needed automatically; (2) 480V SD Bd 2A2-A Page 142

WBN 10-2011 NRC RO Exam As Submitted 8/15/2011 DISTRACTOR ANAL YSIS:

A. Incorrect, Plausible because the compressor will load/unload as needed automatically after it is manually started and the power supply being from a Unit I 480V board is correct.

B. Incorrect, Plausible because the compressor will load/unload as needed automatically after it is manually started and the Auxiliary Air Compressor A-A is supplied from a Unit 2 480V board.

C. Correct, The Control and Service Air Compressor A is one of the two compressors that can be restarted following a Blackout but it must be started manually and the compressor is supplied from the 480V Shutdown Board IA-2A.

D. Incorrect, Plausible because the compressor start being manual is correct and the Auxiliary Air Compressor A-A is supplied from a Unit 2 480V board.

Question Number: 54 Tier: 2 Group 1 K/A: 078 K2.02 Instrument Air System Knowledge of bus power supplies to the following:

Emergency air compressor Importance Rating: 33* / 3*5*

10 CFR Part 55: 41.7 IOCFR55.43.b: Not applicable KIA Match: K/A is matched because the question requires the applicant to have knowledge of the bus power supplies to the auxiliary(emergency) air compressors.

Technical

Reference:

SOI-32.01, Control Air System, Revision 0051 AOl-i 0, Loss of Control Air, Revision 0040 AC 1-35, Loss of Offsite Power, Revision 0038 Proposed references None to be provided:

Learning Objective: 3-OT-SYSO32A

9. Explain how the sequencing device operates to control the control air compressors in Hand and Page 143

WBN 10-2011 NRC RO Exam As Submitted 811512011 Learning Objective: 3-OT-SYSO32A

9. Explain how the sequencing device operates to control the control air compressors in Hand and Auto.

3-OT-AO 13500

2. Given a list of symptoms, Identify Event, determine Automatic Actions and Operator Actions required per AOl-35.

Cognitive Level:

Higher X Lower Question Source:

New Modified Bank Bank X Question History: WBN bank question 078 K2.02 054 Comments:

Page 144

WBN Loss of Offsite Power AOI-35 Unit I Rev. 0038 Step Action/Expected Response Response Not Obtained 3.3 Loss of BOTH 161kV Power Supplies (continued)

15. CHECK cooling water available to PERFORM controlled shutdown USING Secondary Systems (RCW pumps GO-4 Normal Power Operation, running). GO-5, Unit Shutdown From 30%

Reactor Power To Hot Standby, and GO-6, Unit Shutdown From Hot Standby To Cold Shutdown, as applicable.

16. DISPATCH personnel to the following locations to inspect for equipment abnormalities:
  • Relay room.
  • 6.9kV CSST switchgear housing.
  • Start bus switchgear housing.
17. CHECK control air NORMAL: REFER TO AOl-i 0, Loss of Control Air.
  • Required compressors running

[T4M/708].

  • Pressure between 90 and 105 psig [T4M/708].
18. CHECK aux air header pressure REFER TO AOl-b, Loss of Control Air.

greater than 75 psig [1-M-15].

19. DISPATCH AUO to DG bldg to monitor DG operating parameters USING SOI-82 Series.

NOTE Attachment 3 provides reference to applicable Tech Specs as well as recovery actions for loss of both 161 kV lines.

20. REFER TO Attachment 3 for consideration for recovery from loss of both 161kV lines.

Page 10 of 42

WBN Loss of Control Air AOl-b Unit I Rev. 0040 Attachment I (Page 1 of 5)

Local Restart of C&SS Air Compressors Step Action/Expected Response Response Not Obtained NOTE I Continuous contact between UO and NAUO should be maintained to aid quick recovery of air compressors.

NOTE 2 Attachment 1 is posted on panel O-L-240 near the station air compressors. The posting must be revised if this Attachment is revised. Changes in revision level or page number will NOT require re-posting, as long as the content does NOT change CHECK C&SS AIR COMPR SELECT ALT control power using SEQUENCER UNDERVOLTAGE 125V DC Manual Transfer Switch

[42-El, DARK. O-XS-32-5049 [O-L-240].

2. Locally CHECK O-PCV-33-4, CLOSE O-PCV-33-4 using O-HS-33-4.

SERVICE AIR SUPPLY ISOLATION, CLOSED [T7M/708].

3. CHECK local station alarm DARK IF local station alarm LIT,

[O-L-240, T7M1708]. THEN PERFORM the following for each compressor in alarm:

a. RESET high air temp at each compressor (O-TS-32-41, -36, -31).
b. RESET high oil temp at each air compressor (O-TS-32-40, -35, -30).
c. RESET common alarm using 0-HS-32-25B, COMPRESSOR A, B, and C RESET [0-L-240, yellow PB].
d. CHECK common and all local alarms, DARK.

Page 38 of 44

WBN Loss of Control Air AOl-b

  • Unit I Rev. 0040 Attachment I (Page 2 of 5)

Local Restart of C&SS Air Compressors Step Action/Expected Response Response Not Obtained

4. PLACE the following C&SS Compressors to HAND [O-L-2401:
  • A, O-HS-32-25D.
  • B, O-HS-32-26A.
5. PLACE O-HS-32-25A, STATION AIR COMPRESSOR SEQUENCE CONTROL, to Posilion 1.
6. START Compressor A by pushing O-HS-32-25E.
7. CHECK Compressor A loads IF Compressor A does NOT Auto load, automatically. THEN PLACE O-HS-32-43A and -43B to ON (Local O-JB-291-226).

IF Compressor A does NOT load from local panel, THEN:

a. CLOSE O-ISV-32-578, STATION AIR COMPR A UNLOADING HDR ISOL.
b. VENT O-TV-32-579, STATION AIR COMPR A UNLOADING HDR TEST.
8. START Compressor B by pushing, O-HS-32-26B.

Page 39 of 44

WBN Loss of Control Air AOl-b Unit I Rev. 0040 Attachment I (Page 3of5)

Local Restart of C&SS Air Compressors Step Action/Expected Response Response Not Obtained

9. CHECK Compressor B loads IF Compressor B does NOT Auto load, automatically. THEN PLACE 0-HS-32-38A and -38B to ON (Local 0-JB-291-226).

IF Compressor B does NOT load from local panel, THEN:

a. CLOSE 0-ISV-32-581, STATION AIR COMPR B UNLOADING HDR ISOL.
b. VENT 0-TV-32-582, STATION AIR COMPR B UNLOADING HDR TEST.
10. MONITOR Compressor operation:
  • Oil press 25-30 psig on A, B, and C.
  • Cooling water flow.
  • Compressors auto-loading.
  • LOAD and UNLOAD compressors manually to maintain greater than 83 psig using RNO for Steps 7 & 9.

Page 40 of 44

WBN Loss of Control Air AOl-I 0 Unit I Rev. 0040 Attachment I (Page 4 of 5)

Local Restart of C&SS Air Compressors Step Action/Expected Response Response Not Obtained NOTE If 480V Aux Bldg Common Bd is energized, Compressors C should be running and loaded by the load sequencer relay. If the compressors have to be restarted, the SAFESTOP/RESET button for each compressor may have to be placed in SAFESTOP, then back to RESET to allow restart.

11. CHECK 480V Aux Bldg Common Bd **

GO TO Step 14.

ENERGIZED.

12. CHECK local control station alarm IF any local alarm LIT, DARK [0-L-240, T7M/708]. THEN PERFORM the following for each compressor in alarm:
a. RESET high air temp at each compressor (0-TS-32-41, -36, -31).
b. RESET high oil temp at each compressor (0-TS-32-40, -35, -30).
c. RESET common alarm using 0-HS-32-25B, COMPRESSOR A, B, and C RESET [yellow PB].
d. CHECK common and all local alarms, DARK.
13. PLACE C C&SS Compressor to AUTO USING 0-HS-32-27A.
14. CHECK air press returning to between 95 and 100 psig.

Page 41 of44

  • WBN Loss of Control Air AOl-10
  • Unit I Rev. 0040 Attachment I (Page 5of5)

Local Restart of C&SS Air Compressors Step Action/Expected Response [sponse Not Obtained

15. IF compressor A or B being operated in manual mode, THEN MONITOR compressor operation and system press until compressors can be returned to auto mode.
16. RETURN TO Instruction in effect.

Page 42 of 44

WBN Control Air System SOI-32.01 Unit I Rev. 0051 Page 23 of 63 6.0 NORMAL OPERATION 6.1 Normal Operating Parameters and General Information The Control Air System during normal plant operation, supplies compressed air for all Control and Service Air System requirements. It also supplies air to the Auxiliary Air System while the AAS compressors are in standby. Aux Air Compressors auto start when Control Air System header press falls to 83 psig lowering. Control Air System auto isolates to Service Air at 80 psig lowering. Aux air isolates from Control Air System at 79.5 psig lowering. Compressor D will be the normal lead compressor. Compressors A, B, and C will normally be placed in Standby/AUTO and operates according to sequencer setting.

Compressor D has several protective trip(s) as follows:

D Compressor Trips:

1. LowOilTemp8O°F
2. High Oil Temp 135°F
3. High Air Temp Stage 1, 125°F
4. DischAirTemp 125°F
5. High Pinion Shaft Vibration 1st Stage, 1.0 mil
6. High Pinion Shaft Vibration 2nd Stage, .96 mil
7. Low Water Flow
8. Low Seal Air Press 6 psig (also an interlock)
9. Low Oil Press 16 psig A Low seal air interlock of 6 psig for Air Compressor D prevents prelube pump from operating and thus prevents Compressor start. Compressor Ds prelube pump runs when local panel is energized. After Compressor D starts, prelube pump will auto stop and a time delay gives the shaft driven oil pump time to provide a normal oil press of 26 psig. The prelube pump auto starts on Compressor D shutdown and will continue to run until panel deenergized. Upon Compressor D shutdown allow prelube pump to run a minimum of 30 minutes to allow compressor bearings to cooldown. Oil press on running Compressors A, B, and C, should be approx 30 psig, and Oil level at the normal mark on the indicator.

WBN Control Air System SOI-32.01 Unit I Rev. 0051 Page 24 of 63 6.1 Normal Operating Parameters and General Information (continued)

Cooling water discharge temp from running Compressor A, B, and C water jacket should be less than 135°F (Nominal 120°F) and intercooler discharge temp should be less than 110°F (Nominal 90°F). TCV bypass may be OPENED up to 1/4 turn to maintain temp. If more water flow is needed, a WO should be issued for TCV.

Control Air Receiver normal press is approx 95-1 00 psig. Press entering Air Dryers should be 90 psig[c.i]. A? across each Air Dryer Station prefilter should be 5 psid[c.11, if NOT, filter should be cleaned[c.1]. Air flow on Air Dryer Station rotameter during purge normally indicates 55%. One Air Dryer Station is sufficient for one unit operation, although 3 dryers in-service is normal.

Applicable Checklists will be performed at discretion of Operations Superintendent or designee. Checklists will normally be performed for System Alignment Verification in Mode 5 or whenever alignment Verification is needed.

SEE tables below for Compressors A, B, and C loading and unloading sequence.

RISING PRESSURE COMPRESSOR FULL LOAD HALF LOAD UNLOADS LEAD 91 psig 96 psig 99 psig SECOND 85 psig 90 psig 93 psig THIRD 79 psig 84 psig 87 psig DROPPING PRESSURE COMPRESSOR HALF LOADED FULL LOADED LEAD 94 psig 91 psig SECOND 88 psig 85 psig THIRD 82 psig 79 psig

3-OT-SYSO32A Revision 9 Page 4of45 I. PROGRAM Watts Bar Operator Training II. COURSES License Training NOTP License Requalification NAUO Requalification III. TITLE Control Air System IV. LENGTH OF LESSON A. License Training 3 Hours B. NOTP 3 Hours License Requalification and NAUO Requalification times will be determined after objectives are identified.

V. TRAINING OBJECTIVES A S U R 0 0 X X 1. State the function of the Control Air system in accordance with the system description.

X 2. State the design basis of the Control Air system in accordance with FSAR section 9.3.

X X 3. Sketch a simple diagram of the control air system from the compressors through the dryers.

X X 4. Describe the station air compressors; include cooling methods, lubrication, capacities, power supplies, stages, and operation.

X X 5. Discuss the purpose of the intercooler and list the cooling medium and the pressure at which the intercooler safety valve is set to relieve.

X X 6. Given a high intercooler pressure condition on Air Compressor A, B or C, explain how to troubleshoot the control air compressor.

3-OT-SYSO32A Revision 9 Pagesof45 A RSS U ORT 0 CA X X X X 7. Discuss the purpose of the aftercooler and list the cooling medium.

X X X X 8. Describe how the moisture traps work, and why is it important to check and drain the intercooler moisture trap during each shift.

X X X X 9. Explain how the sequencing device operates to control the control air compressors in Hand and Auto.

X X X X 10. Explain how the control air compressors are affected by a loss of power to the sequencer.

X X X 1 1. Explain how the control air compressors may be loaded manually.

X X X X 12. List all trips associated with the control air compressors.

X X X X 13. List the steps necessary to restore a compressor to operation after a trip.

X X X X 14. State the purpose of the control air receivers and pulsation dampener.

X X X X 15. Describe the control air dryers, include modes, drying medium, alarms, indications and flow paths.

X X X X 16. List the events and their corresponding set points that take place on decreasing control air pressure.

X X X 17. Given a loss of non-essential air, explain why a reactor trip will occur.

X X X X 18. Describe the most common problem in instrument air systems. SOER 88-1 Rec. 3 (COMMITMENT)

X X X 19. Describe the major problems facing operators in a loss of instrument air events. SOER 88-1 Rec. 3 (COMMITMENT)

X X X 20. Correctly locate control room common alarms associated with the Control Air system, including:

a. Low Lube Oil Pressure
b. High Discharge Air Temp
c. High Oil Temp

3-OT-SYSO32A Revision 9 Page 6 of 45 A RSS U ORT C CA X X X X 21. Describe the operation of the Centrifugal Air Compressor.

X X X X 22. Describe the two methods used by the Centrifugal Air Compressor to control system pressure.

3-OT-AO13500 Rev 9 I. PROGRAM Page 4 of 63 WATTS BAR OPERATOR TRAINING II. COURSE A. NRC LICENSE PREP B. CERTIFICATION C. ONSITE D. NOTP E. LICENSED REQUAL F. AUO REQUAL III. TITLE AC 1-35, Loss of Offsite Power IV. LENGTH OF LESSON

-1 .5 hours5.787037e-5 days <br />0.00139 hours <br />8.267196e-6 weeks <br />1.9025e-6 months <br /> V. TRAINING OBJECTIVES AR S S U OR T 0 0 A X X 1 Describe the Purpose of AOI-35 X X 2. Given a list of symptoms, Identify Event, determine Automatic Actions and Operator Actions required per AOI-35.

X X X 3. Given one 161 kVWBN Hydro line has deenergized, determine the Automatic Actions that occur.

4. Deleted X X X X 5. Given conditions, determine if Technical Specifications LCO entry is required, what Actions (s) must be taken, and the Bases for the action(s).
6. Deleted 7 Deleted X X X X 8 Given a set of plant conditions, use AOI-35 to correctly diagnose and implement Action Steps, RNOs, Notes and Cautions X X X X 9 Describe the Blackstart procedure and explain when it is used.

WBN 10-2011 NRC RO Exam As Submitted 8/15/2011

55. io3i.oo Which ONE of the following identifies ventilation systems that exhaust out a Shield Building Exhaust Vent?

A. EGTS Cleanup Fans and Annulus Vacuum Control Fans B EGTS Cleanup Fans and Containment Purge Exhaust Fans C. Fuel Handling Exhaust Fans and Annulus Vacuum Control Fans D. Fuel Handling Exhaust Fans and Containment Purge Exhaust Fans DIS TRACTOR ANAL YSIS:

A. Incorrect, EGTS Fans can exhaust out the Shield Building Exhaust Vents but Annulus Vacuum Control Fans exhaust out the Auxiliary Building Vent. Plausible because the EGTS Fans exhausting out the Shield Building Exhaust Vents is correct and the annulus vacuum fans perform the same function of maintaining differential pressure between the containment and the annulus, only they run during normal operation and are isolated when the EGTS cleanup fans start.

B. Correct, the EGTS Fans and Containment Purge Exhaust Fans can exhaust out the Shield Building Exhaust Vents. The Containment Purge Exhaust Fans only exhaust through the Shield Building Exhaust Vent for the applicable unit. The EGTS Fans discharge to a Shield Building Exhaust Vent or recirculation back to the containment annulus depending on differential pressure.

C. lncorrect, Both the Fuel Handling Exhaust Fans and Annulus Vacuum Control Fans exhaust out the Auxiliary Building Vent. Plausible because the Fuel Handling Exhaust Fans are moving air that has the potential to have radioactivity and the annulus vacuum fans perform the same function of maintaining differential pressure between the containment and the annulus, only they run during normal operation and are isolated when the EGTS cleanup fans start.

D. Incorrect, Fuel Handling Exhaust Fans exhaust out the Auxiliary Building Vent (not the shield building vent) but Containment Purge Exhaust Fans do exhaust out the Shield Building Exhaust Vents. Plausible because the Fuel Handling Exhaust Fans are moving air that has the potential to have radioactivity and the Containment Purge Exhaust Fans exhausting out the Shield Building Exhaust Vents is correct.

Question Number: 55 Tier: 2 Group 1 KIA: 103 K1.03 Containment System Page 145

WBN 10-2011 NRC RO Exam As Submitted 811512011 KIA: 103 K1.03 Containment System Knowledge of the physical connections and/or cause effect relationships between the containment system and the following systems:

Shield building vent system Importance Rating: 3.1* / 35*

10 CFR Part 55: 41.2 to 41.9 / 45.7 to 45.8 IOCFR55.43.b: Not applicable KIA Match: K/A is matched because the question requires knowledge of the which stacks provide the flow release path for various ventilation systems including the identification of containment ventilation systems that exhaust out the shield building vent Technical

Reference:

1-47W866-1 R59 1-47W866-10 R33 1 -47W866-1 1 R27 Proposed references None to be provided:

Learning Objective: 3-OT-SYSO65A

4. Describe the Annulus Vacuum System Flowpath per 1 -47W866-1.
5. Describe how the EGTS and Annulus Vacuum Systems maintain annulus pressure.

Cognitive Level:

Higher Lower X Question Source:

New Modified Bank Bank X Question History: SQN bank question 103 K1.03 055 used on the SQN 1/2009 RETAKE exam.

Comments: SQN question written new for the retake exam Page 146

3-OT-SYSO65A Revision 12 Page 5of30 I. PROGRAM Watts Bar Operator Training II. COURSES A. License Training B. Non-License Training III. TITLE Emergency Gas Treatment System (EGTS)

IV. LENGTH OF LESSON A. Licensed Training 2 hours2.314815e-5 days <br />5.555556e-4 hours <br />3.306878e-6 weeks <br />7.61e-7 months <br /> B. Non-Licensed Training 3 hours3.472222e-5 days <br />8.333333e-4 hours <br />4.960317e-6 weeks <br />1.1415e-6 months <br /> V. TRAINING OBJECTIVES 0 0 D 0 (I)

X X X 1. State the Design Basis of the EGTS per FSAR 6.2 and 6.5.

X X X X 2. State the Function of the EGTS subsystems per the system description.

X X X X 3. List the AP maintained by Annulus Vacuum Fans; EGTS Fans.

X X X X 4. Describe the Annulus Vacuum System Flowpath per I -47W866-1.

X X 5 Describe how the EGTS and Annulus Vacuum Systems maintain annulus pressure X X X 6 Given a loss of instrument air/control power, determine the effect on the EGTS subsystems.

XXX7. Correctly locate EGTS subsystems MCR controls & indications including:

a. Annulus Vacuum Fans and Dampers.
b. EGTS Fans and Dampers.
c. CTMT Annulus AP indicators.

x Describe the EGTS filter bank, include purpose of Moisture Separator (Demister), Humidity Heater, Prefilter, HEPA filter, and Charcoal absorbers.

Describe the EGTS fans power supplies.

3-OT-SYSO65A Revision 12 Page6of3o 0 0<

D0QF-

<QØLl)

X X X 10. Describe the Auto starts for the EGTS Cleanup Fans:

a. In A-Auto
b. In P-Auto X X X X 11. Explain how decay heat is removed from a used, out-of-service EGTS filter bank X X X X 12. Describe why an in-service EGTS filter bank would heat up.

X X X X 13. Explain how the Air Cleanup subsystem maintains Annulus pressure.

X X X 14. State how Radiation is monitored on the EGTS.

X X X X 15. Discuss Precautions for the EGTS Annulus Vacuum Subsystem listed in S0l-65.01.

X X X 16. Discuss an auto start failure of the EGTS Air Cleanup System using SOl-65.02.

X X X X 17. Discuss Precautions for the EGTS Air Cleanup System listed in SQl-65.02.

X X X 18. Regarding Technical Specifications and Technical Requirements for this system:

a. Explain the Limiting Conditions for Operation, Applicability, and Bases.
b. Given a status/set of plant conditions, apply the appropriate Technical Specifications.

X X X X 19. Describe the in-plant location of the following:

a. Annulus Vacuum Fans.
b. Annulus Vacuum Fans Damper Reset Switches.
c. EGTS Air Cleanup Fans.
d. EGTS Air Cleanup System AP Control Logic Reset Switches.
e. MCR Indicators and Switches for EGTS.

3-OT-SYSO65A Revision 12 Page 13of30 2 DETAILED DESCRIPTION Annulus Vacuum Subsystem B- This subsystem has two independently uontroliod B-branches.

The two branches draw air frow their assigned annulus

1. Describe the general arrangement of the Annulus and release it to the Auxiliary Building Fuel Needling Area exhaust duct system.

Vacuum Subsystem.

B- Note that the Unit 2 AVCS is isolated by blank-off plates located in coremon portions of the ductwork a This subsystem has two independently controlled LU branches.

a The two branches draw air from their assigned

-J U) annulus and release it to the Auxiliary Building Fuel Handling Area exhaust duct system.

a Note that the Unit 2 AVCS is isolated by blank-off plates located in common portions of the ductwork.

2. Provide an overview of the Annulus Vacuum Subsystem major components.

a isolation Dampers.

In a Modulating Dampers.

a Vacuum Fans.

Ui Each of the major components will be discussed in

-J detail on subsequent slides.

Cl)

3. Describe the function and operation of the Annulus The air inlet for each I respective secondary containment volume above the Vacuum Subsystem.

steel uontainnneet down.

The Unit 2 annulus vacuum control subsystem is a The air inlet for each branch is centrally located in its isolated by means of blank-off plaIns located in tfre common portions of dootwork. respective secondary containment volume above the steel containment dome.

a The Unit 2 annulus vacuum control subsystem is isolated by means of blank-off plates located in the common portions of ductwork.

Ui

-J Cl)

3-OT-SYSO65A Revision 12 Page 26 of 30 m The Shield Building Stack discharge dampers are 5 The Shield Building Stack discharge dumpers are mechanically interlocked with the annulas discharge mechanically interlocked with the annulus discharge dampers. dampers.

Pthth

  • As one damper opens the other closes.

I For additional information refer to:

UI 47W866-1

-J 47W61 1-65-3 Cl) 47W61 0-65-1 I Also refer to 3-OT-STG-065A MFlowControl Modules

  • As AP decreases (annulus pressure rises), the 5- As tP decreases (annulus pressure rises), the echaust damper modulates opec and the anaulus modulates exhaust damper modulates open and the annulus 5.

closed.

This prooides the capability to adjust the amount of air modulates closed.

returned to tIre annulus. with the remainder echaustieg to the Shield Bldg. Stack. I This provides the capability to adjust the amount of air 1() returned to the annulus, with the remainder exhausting UI to the Shield Bldg. Stack.

-J C) 0mm 1VA Flow Control Modules 3. Discuss the MCR controls and indications associated i c5ce5.c,rr MCR Controls with the Flow Control Modules 5-

  • Three position (CLOSE IA AUTO /OPEN) control switch.

.. I CLOSE Manually closes damper.

ID

  • A AUTO:
  • Dampers 1-PCO-65-80 & 1-PCO-65-88 or 1-5-

0 5-PCO-65-82 & 1-PCO-65-89 will modulate to maintain the annulus pressure relative to the outside environment.

  • OPEN Manually opens dampers 1-PCV U,

U, 83/87 (1 -PCV-65-8 1/86).

UI

-J C,)

WBN 10-2011 NRC RO Exam As Submitted 8115/2011

56. 001 1(2.01 056 Given the following plant conditions:

- 480v Emergency Common Transformer is out of service.

Which ONE of the following describes the effect on the Control Rod Drive MG Sets if 6900v Unit Board 1 C is de-energized by relay operation?

A. Both MG sets lose power to the motor.

B Neither MG set loses power to the motor.

C. Only 11 A MG set loses power to the motor.

D. Only B MG set loses power to the motor.

DISTRACTOR ANAL YSIS:

A. Incorrect, Plausible if the applicant does not recall how the loss of the Unit Board power supplies affects the operation of the MG sets or misapplies the interaction between MG set motor breaker tripping due to board undervoltage resulting in the MG set generator breaker tripping and concludes the both M-G sets would be tripped.

B. Correct, Plausible because while the 480v Unit Boards that supply the MG sets are powered from the Unit 1 6.9 kV Unit Boards, neither is supplied from the 6.9kV Boards IC. The 480v Unit Boards supplying the M-G sets 1A and lB are supplied from the 6.9kv Unit Boards IA and ID, respectively.

C. Incorrect, Plausible because M-G Set IA would lose power if the 6.9kV Board IA were to trip.

D. Incorrect, Plausible because M-G Set lB would lose power if the 6.9kV Board ID were to trip.

Page 147

WBN 10-2011 NRC RO Exam As Submitted 811512011 Question Number: 56 Tier: 2 Group: 2 K/A: 001 K2.01 Control Rod Drive System Knowledge of bus power supplies to the following:

One line diagram of power supply to the M/G sets.

Importance Rating: 3.5 I 3.6 10 CFR Part 55: 41.7 IOCFR55.43.b: Not applicable KIA Match: KJA is matched because the question requires knowledge of the one line diagram of the power supply to the M/G Sets from the 6.9kv Unit Board to the 480v MG set motors.

Technical

Reference:

1-1 5E500-1 ,R35 SOl-85.01 ATT 1 P, Control Rod Drive and Indication System Power Checklist 85.01 P, Revision 0040 Proposed references None to be provided:

Learning Objective: 3-OT-SYSO85A

2. Identify the power supply to the MG Sets.

Cognitive Level:

Higher Lower X Question Source:

New Modified Bank X Bank Question History: SQN bank question used on the SQN 1/2008 Audit exam with stem changed to make a different answer correct Comments:

Page 148

WBN Control Rod Drive And Indication SOI-85.01 ATT IP Unit I System Revision 0040 Effective Date: 06-14-2011 Power Checklist 85.01-IP Page 1 of I Performed On PERF VERIFIER NOMENCLATURE LOCATION POSITION UNID INITIALS INITIALS 480V UNIT BD IA CRD MG SET 1A CI4D CONNECTED 1-BKR-85-A (1-.GEN-85-A) CV 480V UNIT BD lB CRD MG SET lB CI3B CONNECTED 1-BKR-85-B (l-GEN-85-B) CV 120V INST PWR RACK A (1-M-7)

UNIT CNTL BD 1-M-7 BKR BKR 7 ON l-BKR-85-R42A 7ATO 1-PNL-085-R42 CV 120V INST PWR RACK B (l-M-7)

UNIT CNTL BD l-M-7 BKR BKR 7 ON l-BKR-85-R42B 7B TO l-PNL-85-R42 CV 120V INST PWR DIST PNL A INST PWR DIST PNL A Vital Batt ON l-BKR BKR7TO 1-PNL-085-L117 BoardRm I, LI17A Bkr7 CV

I.OOS3S i I liii I I III III I

111111 3

I I

I III I I

I 0

III I ii 0

II I I 7 I I 8 I 11111111 10 11 12 TO 9*115 909 V70ROELOLC0N)C TO NOTES OUR AT0RUELELCTRIC TO 540KV 0027407090. BOY

  • 101KV SNI0CIAY*R*, OUT 13 1*36472) 10)40 SNITCAYUOO. 0*7 * )NTNSO2) TO 30040 SN)TCOT060. 0*7 0 117 *1500) IlI NI50 00001*706 1 T

01.2220*6 02I 3-0.500-2.01 I d

{E:] o- 0274040 STU 009* 127127

  • A SIN B

SN _] A N

S*4.4.4.L.44 UNIT STO 50EV S060 UNITCT*SERV UNIT STU 50EV

  • 0 22.5N.R.0.NKV, 50
  • 7 50/00 i50*0*U o:

2.5LS oo !0N n \.X 744 7*0

.093 277

%2000* 20000 TON

.0*0 20T 21.4 0/!

- 0 04440*40* 4C* CW 5.5. 1*40

,_ *0*4* RON BUS A 409 0400 10000 [_21400 UNlESS 00)4 5 UNIT NC? OUS 1*

B UNIT 90? 9*5 00 1 20004 002T S.S. NUS IA 8 I * / 3000*

12*0*

I200* 20000 1200*

2004 frI

.f__

2004 0044 I l000U 252N 10 20220 2)22 0 2B20I2 21221 I90202< 25220 N4I 009) 23 202-9 NC NC 100-202-U 00 501) -05010OI 2000*

I 0 2000* 2000* 15

  • 12000 I

1200*.) 2000* I2000 I I 12000.)

2000*

00004 0440* 2000* 00004 1204 1222 - 154 0

1530 D 0 I 2*00* 420000 I 1214 $1012 1552 01210$ 1020 1520 0 Al2S 1004 0)30_ 1124 llO2440 I5200IN22 O 1110 0 111210 1302 0 C I_OV - I)

  • U IN

)  ;* 0I 0-2*7.N 1451715) 54030107 0-44-300..o 60 *

)so)

To 10 jI) 100001B I) I..N5074iI oc) oc)

I*O20I*

T.55R7O)-I 0

so)

-c 1*12)01*)

]) I2000 sc)]))

I -

I20O0J 04 N - -

  • c )oc)*)

2*00*.. 20000-.

1722 ITI I 2000*-) 20000.)

I 044205A TO 0.900 SO TO 0.9KV 00 NVU NO 10-0 1c T 11 E:: d:-

625* .0* 2

)II54500.BI 574 91St RISE .00 2 Lobe 0 L- L -0 t2esA\4i, L-ss N%)

UTILITY 040 I

7 TE0 oo) 2r;;Al) 0N4V UNIT NO 20 00*72210 000221I I4000VU 1

_oos_B 44 Iy_ I oo)

IBO2050 SE-TO °T)/.5200A 0U0A_.\

rC, e U 3 0 V 1 0 NOV N

!4oV 002V

-3200*

8100 *020 00 1

CT.32T00 oc)

I i°o I0*0011 52O0OT TO I

ITS

) .*ov/2o*9 I 0-0*7-I

>oos

? ?

z 0-044-I 9

A0 z

4-007-I 4 9

1,oá)o,5 o 2

)j NEC (TIllS 000)

T

)TVI0 0901 I)

It 0001 Lss oç )So&

30

..L4 L

...:I..00fl 0000 T 1j I 52T)I

-E r) t 1

00402300002 0B*aaN2 000230I 0-0022*I 350 lIVE 2 071 OLOC EXT PEE 55EV SLOG LXX 44 2 11-444757)

OFF NUOC ETC *0 1145*140)

SITU SETS VENT 00 1 1115fl48I 2I0C2221 SON? ZOO ETC 44 I 1145*137) oc) I UNO 0100*10204 IN o) 150)10) oo) 0 L1 I oo)

IN Lir II C 501* *100 flINT 30 2 11450473. 4)0.4*152222

//

TAt02090 440 NT 5* I 1004 90 TA o-oor-s,-so, oc) NC) 9 URONE0CVtT0T?A0 fl F //I/ 50 4 F NE

/ 00440250 s* 0_O44_226_NI+/-

2 z

44 J.,o-o 00*7221U 0

[.0752 [.0152 0-044-221-US 44*6 4 44*1 N 000526I

  • T 45 NOV loTs 0*1 2:00)350 T ° ] 0 1*51704I) 400V 00* 076 0NTI*T 00 N )I.*SNTS7II 0507 50KV BLOC AOT 5007 40

.44-225-I I )1-55l7l3-I),-40, I 4)00 1000* .) I 0-40-001-1 ooo 1.

0-00221IS 0C) OOU 2

S

,!, IIOTOS 9) 500 *11*27 51* C 0 / I544T13l. 2) NC I. OLU *SO0-450V TRA05F010BNS 060 0011* 7014461 *00 500441*01. 50, C l)fl_0 -

uNLESS 0*0040150 *09040, 124511705002 0*0404000 AULUOS 064 C

I 0. 4 I101CATES 001IPONT 9170 CLASS 0 REOGIREUSOT L0UITEO 10 I 3010010 OU*LIFICOTOON.

I 3. 5. 0 INIOCUIOS 0600546 AOS UUI010IIE 1*03*700 50110100. 54109 TO NC 4) 0PPL0400I.E SOOULO 1130 2100600 700 040040002 000UILS.

0-007-200-N I

.1.,

0-00-2001 I 110 ,J.4-UI*7-200-* 0-044205-0 3) )T45N752I. 2 5)

A. ALL *0700 C*1NTROU 000TER OU550S NAVE U 000U EOTIT00UUUS 0*694*1 60710. B0UOPT0S OCT00.

BLOC 0*1* I 000U 010 NOR TN*T ICC 0. *00*0030 1512. 510. 05)2 2514 *60 L0C0TE0 ON START 5*060 0 07*21 N 0747

  • 4406 0 *704! 2 0-00-205-I 1000/1150 1000/1150 40* 500 I)v022o0o%2 40*40* 1014. 2012 0 2030 AlE L000TOO ON 751 01*10251I TART100RSBI*NO2O0SN).
    • C 44444

}T00OA43 ?oI *0) ,__,00) 0. ONLY OIl! 07 lAO 104100100 TOO 8000UIENS *0*1 NE CEOSEO 01 057 01000 TIES TO 4*010 EXCESSIVE 10*5)50 011 TOE 07*07.

1)

Bus B 0*50 12 004 *05 N IN00A 1200*1 00! 520 UT 200200-A COuNT. 4*

)1) ITUEN ONE OF TUOS0 OBEUIIERS, 5

iSIiIC C*ER*TI0NS OUST ASSURE TOOT 00 CONTROL NOBEl 05 UU*0L0010 70! TOO NREAKEN fiND 000TN0L RELAYS.

1. TTE 011490040 SUPPLY 10 6.SRU C0200 00000 0 04*1 NE 20004900240.

H C 0*7 j 10O0S *010 r )

lj_.

0 l

2 N)C-00 146

5. 00 lONE THAN N NIP. ORE ULLOWEO 10 BC P004900 79002 A 516010 ESSO.

0 P90*001 THIS ALIOTNIONI. 1050 S000 CIRCUIT I 0 2 N*NSSN)T070S )I0S200I 000 20520021 lOST 00 PIStOl 05 TAO 00110* POSITION 00100 10 9*470100 CSST

  • U! 9 7601 SERVICE. 1007 000 l4507N02002)

U2 907040/ *0)0 BOARD.

TISIITNI 0

  • 1U!

4065 25 *01 OA*7L5*L0, H

N NO) so)

I .___L )

00R. 0ON. 10-004-207-4 C

  • 1J 4025090 DENOTES UNIT IESNIT 2 051090400.

IIR4 so) 00)

TIC)

I J

TIllS C0NTICI*RAT1OII 00021001 OIAWITIC 0UPERSEOES UNIT I 69)

- 0-lO0-200-Cçl-4507*0-3, 01

-T 0-000-207-N V-NSR7AO-21 AS4050IRUCTEO 0601100 130000I OEVI5IO* 0.

I ESJ ROE -ia.

1 001)

NEVISED PER OCO 34*44232.

I I NOV SCALE,0T5 C00050 REF PREPARER CHECKER j UPPRCVEO COCEPT US NOTED DATE 7T0*OCT FOCILITY NA) GENERAL 0-1010-200-i 11009750-I UNITS 1 & 2 Si.  ?

5 U7t T

1 l4TN10 VO 00 N 01010-217-0 1453740) 0000240I 400U 72525 0*244046 TASTE 4709 515 00 KEY DIAGRAM NOSN 0*0*

I 00A I I STATION AUX POWER SYSTEM 1? 1) 11 1I ( Li WATTS BAR NUCLEAR PLANT I I I COOFASION IR*I101CS II5E500I. 2 1 TENNESSEE VALLEY AUTHORITY Q ii NUCLEAR SAFETY RELATED 000ICN INITIAl. ISSUE I ENGINEENIND K

0044220-I 00*70202 200 NyU 0001220N 30001*0 0007225A 200 000 0007257-I ISO 40*

00072202 50 0*0 00071273 ISO OVA 0044227N ISO 100 ORUFTEN 098)059 0100000 4011/OSJ 00 ISSUE PER I 11 APPROVAL

  • 1010! P004000 2 WSEP5.174*RUUS S.*. 40010402 1200A f2000 ]2O0O ]20V0 FSAR FIC. 8.1.2 1.1. ITONION 9.1. FONESTEN Ii 11.0. 1700 FOB *012 NO. I 00. 2 *0. S NO. * °° OUX 0100 ETC 005 11451725) 1*00 0100 C 005 11450734) 0-14-RI 851 0 115E5001 930 I I I I I I I I I I I I I I I I I I I I I I I II I I 1 r 1 1111F1 I I I I I I I Iiir [1 I I 11IIIIrrII1 1 I I I I I I I I I I I I I I I I I 5*10T54409.

1 2 3 4 I 5 I 6 I 7 8 9_

CAD MAINTAINED DRAWING IIC0NFIGURAT1ON CONTROL DRAWING)

5QN #1C

1. Given the following:

- UnitatlOO%RTP

- 480v Emergency Common Transformer Out of Service Which ONE of the describes the effect on the Control Rod Drive MG Sets if 6900v Unit Board I D is de-energized by relay operation?

A. Both MG sets lose power to the motor.

B. Neither MG set loses power to the motor.

C. Only A MG set loses power to the motor.

D Only B MG set loses power to the motor.

-cIvIsu[I t Page 5of58 I. PROGRAM Watts Bar Operator Training II. COURSES A. License Training B. Non-License Training III. TITLE Rod Control and Motor Generator Sets IV. LENGTH OF LESSON A. Licensed Training 6 hour6.944444e-5 days <br />0.00167 hours <br />9.920635e-6 weeks <br />2.283e-6 months <br /> B. Non-Licensed Training 6 hours6.944444e-5 days <br />0.00167 hours <br />9.920635e-6 weeks <br />2.283e-6 months <br /> V. TRAINING OBJECTIVES NOTE Octives will be identified in the text as Objective RC1 etc.

A R S S U 0 R T 0 0 A X X X X 1. State the design basis (purpose) of the control rod drive system.

X X X X 2. State the number of RCCAs and their compositions.

X X X X 3. Identify the number of banks, groups per bank, and rods per group for the shutdown control rods.

X X X X 4. Describe the sequence of shutdown bank withdrawal or insertion including mode of control and speed.

X X X X 5. Identify the number of banks, groups per bank, and rods per group for the control banks.

X X X X 6. Describe how the rod drive mechanism moves rods on withdrawal, rest, or insertion.

X X X X 7. Describe the effects of normal control rod motion on RCS Tavg.

X X X X 8. Describe the controls for the control rods, including mode selector switch, speeds, and bank overlap.

X X X 9. Sketch the control rod drive control logic from the input signals to the cyclers.

X X X X 10. Identify and explain the input channels to the automatic rod control system.

X X X 11. Explain how the rod control inputs serve to position the control rods on a given change in any one.

X X X X 12. Describe the operation of the rate comparator circuit.

X X X X 13. Discuss the purpose of the non-linear gain circuit.

X X X X 14. Discuss the purpose of the variable gain circuit.

r.eviiuii Page6of58 ARSS iL _ _ ._

Li Li 1 I 0 CA X X X X 15. Draw and explain the gull wing program.

X X X X 16. Briefly describe the purpose of each type of control rod system cabinet.

X X X X 17. Briefly explain how to start up the motor generator sets.

X X X X 18. Explain the purpose of the maintenance hold system for the control rod system.

X X X X 19. Describe the power supplies for the control rod drive system.

X X X 20. Differentiate between the Rod Urgent Failure and Non-Urgent Failure alarms. Explain the cause and effect of the alarms and how resetting of alarms is accomplished.

X X X X 21. List each of the rod control stops/interlocks and give its purpose.

X X X 22. For the rod position indicators, state the sources of signals, type of indication, and all alarms generated by each circuit.

X X X 23. Given a failure of the controlling input instrumentation for rod control and no operator action, describe the effects of rod motion on the plant, if any.

X X X X 24. Explain how a normal reactor trip occurs and how to perform an emergency reactor trip from outside the main control room.

X X X 25. Explain the bases, input, alarms, and operator actions relative to the rod insertion limits.

X X X 26. Discuss applicable Technical Specifications, Technical Requirements, and Bases. V

reviiuii t, Page 7of58 TRAINING OBJECTIVES MG SETS NOTE: Objectives wiI[ be identified in the text as Obfective MGI etc.

A R S S U 0 R T 0 0 A X X X X 1. Describe the power supply to the Control Rod Drive Mechanisms.

X X X X 2. Identify the power supply to the MG Sets.

X X X X 3. Explain what the bypass breakers are used for.

X X X X 4. Explain the function of the protective relaying equipment provided to each MG Set.

X X X X 5. Describe the position indication or annunciations the Operator has in the Main Control Room for the reactor trip and bypass breakers.

X X X X 6. Describe the daily routine checks an AUO makes on the MG Sets and CRD Equipment Room as specified in Electronic Logs.

X X X X 7. Explain how to place a MG Set in service.

X X X X 8. Explain how to take a MG Set out of service normally.

X X X X 9. Explain how to locally trip the reactor in the event of an ATWS.

WBN 10-2011 NRC RO Exam As Submitted 8/1512011

57. 015 A1.08 057 Given the following conditions:

- Unit 1 is operating at 80% steady state power with all controls in automatic.

Which ONE of the choices below completes the following sentence?

Assuming no operator action, RCS temperature will INCREASE if the (1) fails (2)

A. N43 Power Range lower detector HIGH B. N43 Power Range uer detector LOW C. Rod control auctioneered high nuclear power signal HIGH D Rod control auctioneered high nuclear power signal LOW DIS TRACTOR ANAL YSIS:

A. Incorrect, N4 I PR lower detector falling high causes sum of N4 I upper and lower detectors to fall high. This is the input into the rod control power mismatch circuit and since the auctioneered high signal (N4 1) is greater than turbine power, control rods move in, causing RCS temperature to go down. Plausible because power range detectors are input to the control program and failures can cause RCS temperature changes.

B. Incorrect, N4 I PR upper detector failing low causes sum of N4 I upper and lower detectors to decrease. One of the remaining PR detector outputs N42-N44 is already or becomes the auctioneered high signat, O.e. there is no change to the auctioneered high signal) and control rods do not move, so no change to RCS temperature. Plausible because power range detectors are input to the control program and failures can cause RCS temperature changes.

C. Incorrect, This would cause the RCS temperature to decrease because rods would insert as a result of the failure. Plausible because a failure of the auctioneered signal will cause RCS temperature to increase but it would be a failure in the opposite direction.

D. Correct, The auctioneered high nuclear power signal failing low causes control rods to move out and RCS temperature to go up. This is because the Tavg would appear to be lower than Tref.

Question Number: 57 Page 149

WBN 10-2011 NRC RO Exam As Submitted 8/15/2011 Tier: 2 Group 2 K/A: 015A1.08 Nuclear Instrumentation Ability to predict and/or monitor changes in parameters to prevent exceeding design limits) associated with operating the NIS controls including:

Changes in RCS temperature.

Importance Rating: 3.3 I 3.4 IOCFRPart55: 41.5/45.5 IOCFR55.43.b: Not applicable KIA Match: Question requires applicant to predict the effect of a failure of NIS or associated components on RCS temperature.

Technical

Reference:

WBN-SDD-N3-85-4003, Control Rod Drive System, Revision 0013 Proposed references None to be provided:

Learning Objective: 3-OT-SYSO85A

7. Describe the effects of normal control rod motion on RCS Tavg.
10. Identify and explain the input channels to the automatic rod control system.
11. Explain how the rod control inputs serve to position the control rods on a given change in any one.

Cognitive Level:

Higher Lower X Question Source:

New Modified Bank Bank X Question History: WBN bank question 015 A1.08 058 used on 2006 exam with minor formatting changes.

Comments:

Page 150

WBN System CONTROL ROD DRIVE SYSTEM WBN-SDD-N3-85-4003 Description Rev. 0013 Document Paae 59 of 81 8.3 Figure 8.3 Reactor Control System Functional Block Diagram Figure 8.3 Reactor Contr1 System Functional Block Diagram

WBN System CONTROL ROD DRIVE SYSTEM WBN-SDD-N3-85-4003 Description Rev. 0013 Document Page 61 of 81 8.5 Figure 8.5 Power Mismatch Channel, Nonlinear Gain FIGURE 8.5 Power Mismatch Channel, Nonlinear Gain 1.5 1.0 L5°F/%Aq

-3 1 2 3

-0.5

-1.0

-1.5

% aq (% Fuji Load Turbine Impulse Pressure %

Full Load Auctioeered Nuclear Flux)

WBN System CONTROL ROD DRIVE SYSTEM WBN-SDD-N3-85-4003 Description Rev. 0013 Document Paqe 62 of 81 8.6 Figure 8.6 Turbine Impulse Chamber Variable Gain FIGURE 8.6 Turbine Impulse Chambr Variable Gain 2.0 Inversely Proportional To Turbine Power

  • 1.0 0 50% 100%

Turbine Impulse Chamber (Percent Power)

Variable Gain

WBN System CONTROL ROD DRIVE SYSTEM WBN-SDD-N3-85-4003 Description Rev. 0013 Document Page 63 of 81 8.7 Figure 8.7 Rod Speed Program FIGURE 8. 7 Rod Speed Progrem

Revision 10 Page 4 of 57 I. PROGRAM Watts Bar Operator Training II. COURSES A. License Training B. Non-License Training III. TITLE Rod Control and Motor Generator Sets IV. LENGTH OF LESSON A. Licensed Training 6 hour6.944444e-5 days <br />0.00167 hours <br />9.920635e-6 weeks <br />2.283e-6 months <br /> B. Non-Licensed Training 6 hours6.944444e-5 days <br />0.00167 hours <br />9.920635e-6 weeks <br />2.283e-6 months <br /> V. TRAINING OBJECTIVES NOTE: OjcUves will be identified in the text as Objective RC1 etc.

A R S S U 0 R T 0 0 A X X X X 1. State the design basis (purpose) of the control rod drive system.

X X X X 2. State the number of RCCAs and their compositions.

X X X X 3. Identify the number of banks, groups per bank, and rods per group for the shutdown control rods.

X X X X 4. Describe the sequence of shutdown bank withdrawal or insertion including mode of control and speed.

X X X X 5. Identify the number of banks, groups per bank, and rods per group for the control banks.

X X X X 6. Describe how the rod drive mechanism moves rods on withdrawal, rest, or insertion.

X X X X 7. Describe the effects of normal control rod motion on RCS Tavg.

X X X X 8. Describe the controls for the control rods, including mode selector switch, speeds, and bank overlap.

X X X 9. Sketch the control rod drive control logic from the input signals to the cyclers.

X X X X 10. Identify and explain the input channels to the automatic rod control system.

X X X 1 1. Explain how the rod control inputs serve to position the control rods on a given change in any one.

X X X X 12. Describe the operation of the rate comparator circuit.

X X X X 13. Discuss the purpose of the non-linear gain circuit.

X X X X 14. Discuss the purpose of the variable gain circuit.

Revision 10 Page 5 of 57 ARSS U ri I 0 CA X X X X 15. Draw and explain the gull wing program.

X X X X 16. Briefly describe the purpose of each type of control rod system cabinet.

X X X X 17. Briefly explain how to start up the motor generator sets.

X X X X 18. Explain the purpose of the maintenance hold system for the control rod system.

X X X X 19. Describe the power supplies for the control rod drive system.

X X X 20. Differentiate between the Rod Urgent Failure and Non-Urgent Failure alarms. Explain the cause and effect of the alarms and how resetting of alarms is accomplished.

X X X X 21. List each of the rod control stops/interlocks and give its purpose.

X X X 22. For the rod position indicators, state the sources of signals, type of indication, and all alarms generated by each circuit.

X X X 23. Given a failure of the controlling input instrumentation for rod control and no operator action, describe the effects of rod motion on the plant, if any.

X X X X 24. Explain how a normal reactor trip occurs and how to perform an emergency reactor trip from outside the main control room.

X X X 25. Explain the bases, input, alarms, and operator actions relative to the rod insertion limits.

X X X 26. Discuss applicable Technical Specifications, Technical Requirements, and Bases.

Revision 10 Page 6 of 57 TRAINING OBJECTIVES MG SETS NOTE: Objectives will be identified in the text as Objective MGi etc.

A R S S U 0 R T 0 0 A X X X X 1. Describe the power supply to the Control Rod Drive Mechanisms.

X X X X 2. Identify the power supply to the MG Sets.

X X X X 3. Explain what the bypass breakers are used for.

X X X X 4. Explain the function of the protective relaying equipment provided to each MG Set.

X X X X 5. Describe the position indication or annunciations the Operator has in the Main Control Room for the reactor trip and bypass breakers.

X X X X 6. Describe the daily routine checks an AUO makes on the MG Sets and CRD Equipment Room as specified in Electronic Logs.

X X X X 7. Explain how to place a MG Set in service.

X X X X 8. Explain how to take a MG Set out of service normally.

X X X X 9. Explain how to locally trip the reactor in the event of an ATWS.

WBN 10-2011 NRC RO Exam As Submitted 811512011

58. 016 A3.02 058 Given the following:

- Unit I is operating at 100% power with all controls in automatic.

- 1-XS-68-339E, PZR LEVEL CONTROL CHANNEL SELECT, is selected to the Ll-68-339/B320 position.

- A small leak develops on the upper tap fitting for Pressurizer level transmitter I -LT-68-339.

Which ONE of the following identifies...

(1) the affect the leak will have on I -Ll-68-339 and (2) how the actual pressurizer level will be affected?

1 -Ll-68-339 Indication Actual Pressurizer Level A. Will start rising Will remain constant B Will start rising Will be dropping C. Will start dropping Will remain constant D. Will start dropping Will be dropping Page 151

WBN 10-2011 NRC RO Exam As Submitted 811512011 DISTRACTOR ANAL YSIS:

A. Incorrect, Plausible because the leak causing 1-LI-68-339 to start rising is correct and if 1-XS-68-339E, PZR LEVEL CONTROL CHANNEL SELECT been in the LI-68-335/B320 position the pressurizer level would have remained constant. Also the indicated level would be dropping if the leak had been on the opposite sensing tap.

B. Correct, The leak will cause 1-LI-68-339 to start rising and since 1-Ll-68-339 is selected for contro4 the pressurizer level control system will throttle the level control valve in the closed direction causing the actual pressurizer level to start dropping.

C. Incorrect, Plausible because the leak causes the ziP to start dropping but a dropping DP causes indicated level to start rising (this concept could be reversed, in error, by the applicant). If 1-XS-68-339E, PZR LEVEL CONTROL CHANNEL SELECT been in the LI-68-335/B320 position the pressurizer level would have remained constant. Also, the indicated level would be dropping if the leak had been on the opposite sensing tap.

D. lncorrect Plausible because the leak causes the zIP to start dropping but a dropping ziP causes indicated level to start rising (this concept could be reversed by the applicant) and with 1-XS-68-339E, PZR LEVEL CONTROL CHANNEL SELECT, in the LI-68-339/B320 position, 1-LI-68-339 is selected for control. The pressurizer level control system will be throttling the level control valve in the closed direction causing the actual pressurizer level to start dropping.

Page 152

WBN 10-2011 NRC RO Exam As Submitted 8/15/2011 Question Number: 58 Tier: 2 Group 2 K/A: 016 A3.02 Non-Nuclear Instrumentation System (NNIS)

Ability to monitor automatic operation of the NNIS, including:

Relationship between meter readings and actual parameter value Importance Rating: 2.9*12.9*

IOCFRPart55: 41.7/45.5 IOCFR55.43.b: Not applicable K/A Match: K/A is matched because the question requires the ability to determine the relationship between actual pressurizer level and an indicated level when a malfunction occurred on the system.

Technical

Reference:

AOl-20, Malfunction of Pressurizer Level Control System, Revision 0032 Proposed references None to be provided:

Learning Objective: 3-OT-SYSO68C

12. Identify the program setpoints, and describe any automatic actions relative to the pressurizer level program.

Cognitive Level:

Higher X Lower Question Source:

New Modified Bank Bank X Question History: WBN bank question SYSO68C.08 014 with editorial changes in the stem and changes to parts of 2 d istractors.

Comments:

Page 153

ir r 1CONTROLXFER PZR L C-IPNNEL SELECT PT-68-MU & 334 IJ68--339 & 335 a 1 PT-8-334 L/B23 U-68-339-\

8320 \A

Page 1 of 1 I

PZR LEVEL 80

- 94 4O I

p file://C :\Documents and Settings\NRC4Test\Desktop\MCR pictures\WBN Sim Photos\1 M... 4/17/2011

WBN MALFUNCTION OF PRESSURIZER AOl-20 Unit I LEVEL CONTROL SYSTEM Rev. 0032 4.0 DISCUSSION 4.1 General This procedure covers failures within the pzr level control system which could result in abnormal pzr level. Since this is a system AOl, failures considered include sensing instrument failures, circuitry failures, controller failures and equipment failures such as failed valves.

4.2 Pressurizer Level Control Channels Three hot calibrated pzr level channels (1-LT-68-339, -335, & -320) are used for indication, control, and RPS input. Each channel indicates on Panel 1-M-4 and inputs to the RPS. Any one channel can be selected by 1-XS-68-339B [1-M-5] to record on 1-LR-68-339.

Selector switch 1-XS-68-339E is a 3 position switch which is used to select two of the three pzr level instruments for control. 1-LT-68-335 & -339 are NORMALLY selected for control.

1-LT-68-339 NORMALLY controls level via 1-FCV-62-93. It closes 1-FCV-62-70,

-72, -73 and -74 to isolate letdown and turns off the pzr heaters on low level (17%). It provides input to the following annunciators:

  • 92-A, PZR LEVEL HI/LO
  • 92-B, PZR LEVEL HIDEVN
  • 92-C, PZR LEVEL LO-HTRS OFF & LTDN CLOSED 1-LT-68-335 NORMALLY closes 1-FCV-62-69, -72, -73 and -74 to isolate letdown and turns off pzr heaters on low level (17%). It provides input to annunciators 92-A (HI only) and 92-C.

1-LT-68-320 NORMALLY has no control functions; however, it can be selected, via XS-68-339E, to replace 1-LT-68-339 or -335.

Pzr level program is derived from Auctioneered Tavg compared to no load Tavg.

This program level signal is displayed on 1-LR-68-339. This signal is compared to I -LT-68-339 (or -320) to develop an error signal used to control I -FCV-62-93.

Page 10 of 16

WBN MALFUNCTION OF PRESSURIZER AOI-20 Unit I LEVEL CONTROL SYSTEM Rev. 0032 4.3 Instrument or Control Circuitry Failure Instrument failures can be quickly diagnosed by a channel check. Pzr program signal failure can be quickly diagnosed by checking recorder 1-LR-68-339. Failure low of a pzr level instrument channel will result in letdown isolation resulting in actual level rising. This level rise will be greater if Channel 339 fails since it also would result in a large error signal to raise charging. In this case (low failure), an operable channel can be selected and letdown quickly restored. Failure high of a pzr level instrument will result in a reduction of charging to the minimum 55 gpm Instrument and program level signal failure can be compensated for by taking manual control of charging. Signal failures up to the individual controllers for the charging valve can be compensated for by manual control of the individual controller. If necessary, local manual control of the charging valve can be taken.

4.4 Loss of Charging Loss of charging can occur due to loss of Charging Pump or closing of a valve in the charging line. Since all air operated valves in the charging line fail open, the most likely cause of a loss of charging would be due to an instrument failure causing letdown isolation or trip of the Charging Pump. Instrument failures have already been addressed. If a Charging Pump were to trip, letdown should be manually isolated to conserve RCS inventory and to prevent damage to the regenerative heat exchanger. Another Charging Pump can be started and letdown reestablished.

It should be noted that Operator response to a tripped Charging Pump would be to immediately isolate letdown. Entry into this procedure from this point would result in basically verification of actions already taken with additional actions to restore charging and letdown.

4.5 Loss of Letdown Loss of letdown can occur due to failure low of a pzr level instrument, actual pzr level of 17%, valve mispositioning or due to valve failure. Since instrument failures have been discussed and an actual low pzr level a normal occurrence for loss of letdown, valve mispositioning or failures are the remaining reason for loss of letdown. In the case of valve failure, normal letdown may not be restored and excess letdown will have to be used.

Page 11 of 16

1 v fgN I1,iL 7 i oJ SYSO68C.08 014 Given the following plant conditions:

  • 1-XS-68-339E PRZ LVL CTRL CHAN SELECT is in the 339 /335 position.
  • With actual pressurizer level at 50%, 1-LT-68-339 Pressurizer Level Transmitter develops a slow leak in the reference leg.

What is the effect on Pressurizer level and charging flow?

a. Actual level in the Pressurizer will be increasing, causing charging flow to lower.
b. Actual level in the Pressurizer will be decreasing, causing charging flow to rise.
c. Indicated level on 1-LT-68-339 will be increasing, causing charging flow to lower.
d. Indicated level on 1-LT-68-339 will be decreasing, causing charging flow to rise.

3-OT-SYSD68C Revision 13 P 5 of 45 I. PROGRAM Watts Bar Operator Training II. COURSES A. License Training B.. Non-License Training III. TITLE PZR, PZR Pressure Control System! PZR Level Control System, and PRT IV. LENGTH OF LESSON A. License Training 4 Hours B. Non-License 6 Hours V. TRAINING OBJECTIVES 0 0 D 0 (I) (I)

X X X X 1. Identify the three (3) main purposes of the Pressurizer.

X X X X 2. Describe the major components of the Pressurizer.

X X X X 3. Describe the purposes of the Manual Bypass Pressurizer Spray Throttle Valves.

X X X X 4. Identify the normal setpoint required to auto open the PZR Relief

. Valves (PORVs).

X X X X 5. Identify each setpoint and resulting automatic action for the Pressurizer Pressure Program.

X X X 6. State the basis for the low pressure reactor trip, as stated in Tech Specs Section 2.1.1.

X X X 7. State the basis for the high pressure reactor trip, as stated in Tech Specs Section 2.1.1.

X X X 8. Describe the operation of the master pressure controller.

X X X 9. Describe what control room indication would alert the operator that the pressurizer spray valves were open.

X X X 10. Describe the method of control for the power operated relief valves.

X X X 1 1. Describe the indication an operator has that a PORV is open or leaking through.

X X X X 12. Identify the program setpoints, and describe any automatic actions relative to the pressurizer level program.

3-OT-SYSO68C Revision 13 Pages or45 0 0 D 0 C) C,)

X X X X 13. Describe the basis for the program setpoints of the pressurizer level program circuit.

X X X X 14. Explain the basis for programming the level vs. maintaining the level constant in the pressurizer.

X X X X 15. Describe the response to a deviation from pressurizer level program.

X X X X 16. Explain the purpose of the PRT.

X X X X 17. Identify the components which drain into the Pressurizer Relief Tank.

X X 18. Deleted.

X X 19. Deleted X X X X 20. Describe the in-plant location of major system components, instrumentation, controls, and piping/header arrangements.

X X X X 21. Describe the flow path of sources of supply, discharges, vents, drains, leakoff, and connections/penetrations that intertie this system to other systems.

X X X X 22. Explain the operation of major system components.

X X X X 23. Deleted X X X 24. Deleted

3-OT-SYS068C Revision 13 Page 16of45 Wp sue,rV 7. Two additional indicating channels, PT-68-337C and -

Two additionol ivdicnting channels. PT-68-337C cod -

PT-68-342C, are located in the Aux. Control Room.

PT-68-342C. are located in the Ann. Control Roon,.

LC)

C4 Li.I

-J Co I7!l 8. Discuss the M5 temperature indications available for Teniperatore Indications

  • Vavcand Liquid spnve - 0 - -

the pressurizer.

p

  • . PZRTEMP
  • 1-Tl-68-319 (5,4
  • 1-Tl-68-324 LU  ; I1H*

-j Ci) 0

9. Discuss the M,level indications available for the pressurizer.
  • 1 -LI-68-339A F
  • 1-LI-68-335A LU
  • 1-LI-68-320

-J U)

  • 1-LI-68-321 (Cold Calibrated)

NOTE: A 1% change in Hot calibrated Pressurizer level requires approximately 62 gallons of charging makeup.

10. Discuss how same four transmitters provide signals to Level Indications Three transmitters provide signals to pressurizer level pressurizer level control and reactor protection.

control and reactor protection.

n- HI Prnn,urlunr LceI Rncctnr Top 192%l

  • 2 of 3 hot calibrated channels> 92% = High Level Reactor Trip.

C LU

  • Cold Calibrated channel (321) does not input to level control or reactor protection.

-j U)

WBN 10-2011 NRC RO Exam As Submitted 811512011

59. 028 K5.04 059 Given the following:

- Unit 1 was operating at 100% power when a small break LOCA resulted in a reactor trip and safety injection.

- The crew is now performing E-1, Reactor or Secondary Leak, and is determining if the Hydrogen Recombiners are to be placed in service.

- Containment Hydrogen analyzers 1-H21-43-200 and 1-H21-43-210 indicate 0.5%.

Assuming conditions do not change, which ONE of the following identifies the hydrogen mitigation systems that will be in service when E-1 is completed?

A ONLY the hydrogen igniters B. ONLY the hydrogen recombiners C. BOTH the hydrogen igniters and the hydrogen recombiners D. NEITHER the hydrogen igniters nor the hydrogen recombiners DISTRA CTOR ANAL YSIS:

A. Correct, The hydrogen igniters would have been placed in service during the performance of E-0 Appendix A and the recombiners would not be placed in service because the hydrogen concentration is below the minimum value of 0.6%

required for placing them in service.

B. Incorrect, Plausible because the hydrogen recombiners are placed in service during performance of E-1, but only if the hydrogen concentration greater than 0.6% and lower than 5.0%. The current value of 0.5% could be misapplied to the operation of the recombiners and igniters.

C. Incorrect, Plausible because both the recombiners and igniters would be in service if the hydrogen concentration at been 0.6% when checked during performance of E-1.

D. Incorrect, Plausible because the concentration at 0.5% is below the concentration of hydrogen to be flammable and there is a concentration range where the recombiners are placed in service and if outside the range on either side they are not placed in service.

Question Number: 59 Page 154

WBN 10-2011 NRC RO Exam As Submitted 811512011 Tier: 2 Group 2 K/A: 028 K5.04 Hydrogen Recombiner and Purge Control System (HRPS)

Knowledge of the operational implications of the following concepts as they apply to the HRPS:

The selective removal of hydrogen Importance Rating: 2.6? I 3.2?

IOCFRPart55: 41.5/45.7 IOCFR55.43.b: Not applicable KIA Match: K/A is matched because the question requires applying supplied data to the procedural requirements to determine the impact on the operation of the hydrogen removal systems.

Technical

Reference:

E-0, Reactor Trip or Safety Injection, Revision 0030 E-i, Loss of Reactor or Secondary Coolant, Revision 0016 Proposed references None to be provided:

Learning Objective: 3-OT-EOP0001

5. For a given H 2 concentration in containment determine if the H2 igniter should be energized and explain why or why not.
8. Given a set of plant conditions, use E-1, ES-I .1, ES-i .2, ES-i .3, and ES-i .4 to correctly diagnose and implement: Action Steps, RNOs, Foldout Pages, Notes, and Cautions.

Cognitive Level:

Higher X Lower Question Source:

New X Modified Bank Bank Question History: New question for the WBN 10/2011 NRC exam Comments:

Page 155

WBN 10-2011 NRC RO Exam As Submitted 8/1512011 Page 156

WBN Reactor Trip or Safety Injection E-O

  • Unit I Rev. 0030 Step Action/Expected Response Response Not Obtained
5. PERFORM Appendixes A and B, E-0, pages 16-30
6. ANNOUNCE reactor trip and safety injection over PA system.
7. ENSURE secondary heat GO TO FR-H.1, Loss of Secondary sink available with either: Heat Sink.
  • Total AFW flow greater than 410 gpm, OR
  • At least one S/G NR level greater than 29% [39% ADVJ.

Page6of3l

WBN ReactGr Trip o-r Safety Injection E-Q Unit I Rev. 0030 Appendix A (Page 7 of 9)

Equipment Verification Step Action/Expected Response Response Not Obtained

18. WHEN Attachment 1 is complete (Ice Condenser AHU Breakers OPEN),

THEN ENERGIZE hydrogen igniters

[1-M-1 0]:

  • 1-HS-268-73 ON.
  • 1-HS-268-74 ON.

NOTE The following equipment is located on 1-M-9.

19. CHECK CNTMT PURGE fans STOP fans AND STOPPED.

PLACE handswitch in PULL-TO-LOCK.

20. CHECK FUEL HANDLING EXH fans STOP fans AND STOPPED, Fuel and Cask loading dampers CLOSED: PLACE handswitch in PULL-TO-LOCK, THEN Manually CLOSE dampers.
21. ENSURE AB GEN SUPPLY and STOP fans AND EXH fans STOPPED.

PLACE handswitch in PULL-TO-LOCK.

NOTE Dampers 1-HS-30-158 and 2-HS-30-270 remain open during ABI.

22. ENSUREABGEN SUP& EXH Manually CLOSE dampers.

dampers CLOSED.

Page22of3l

WBN - Loss of Reactor or Secondary Coolant .1 Unit I Rev. 0016 Step _Action/Expected Response Response Not Obtained

29. DETERMINE if hydrogen recombiners should be placed in service:
a. CHECK hydrogen a. NOTIFY Chemistry to evaluate analyzers in service. sampling cntmt concentration.
b. CHECK cntmt hydrogen b. CONSULTTSCto less than 5% [M-10]. determine further action.

GO TO Step 30.

c. CHECK cntmt hydrogen c. PLACE hydrogen recombiners less than 0.6% [M-10]. in service. USING SO1-83.01, Containment Hydrogen Recombiners.

Page 18of24

3-OT-EOPO1 00 Revl4 Page 4 of 138 pages I. PROGRAM:

Watts Bar Operator Training II. COURSE:

A. License Training B. License Operator Requal III. TITLE:

E-1, Loss of Reactor or Secondary Coolant IV. LENGTH OF LESSON:

A. License training 3 Hours B. License Operator Requal License operator REQUAL time will be determined after objectives are identified.

V. TRAINING OBJECTIVES:

O 0<

D L1 O F f) U)

X X X 1. Describe the purpose of procedure E-1 as listed in Section

. 1.0 of the procedure.

X X X 2. Explain the basis for tripping the RCPs in an accident situation given the following conditions:

a. RCS press less than1500 psig
b. Phase B isolation signal initiated.

X X X 3. List the condition that must be checked and satisfied before removing a RCP from service in accident conditions due to low RCS pressure (< 1500 psig).

X X X 4. Explain the basis for the continuous action step to monitor containment pressure and stop the CS pumps when containment pressure is verified less than 2.0 psig.

X X X 5. For a given H2 concentration in containment determine if the H2 igniter should be energized and explain why or why not.

3-OT-EOPO1 00 Rev 14 Page 5 of 138 pages V. TRAINING OBJECTIVES: (continued) 0 0 0 D Cl)

Cl)

.X X X 6. Explain the basis for isolating the CLAs when RCS press decreases to less than 250 psig.

X X X 7. Explain the reason for transfer to Hot Leg recirc following a LOCA including the location of the worst case break for this concern.

X X X 8. Given a set of plant conditions, use E-1, ES-i .1, ES-i .2, ES-i .3, and ES-i .4 to correctly diagnose and implement: Action Steps, RNOs, Foldout Pages, Notes, and Cautions.

X X X 9. List the four parameters (not setpoints) that must be verified prior to SI termination.

X X X 10. Determine the correct procedure transition if during the SI termination steps of ES-i .i it is determined that PZR level cannot be maintained using the normal charging flowpath.

X X X 1 1. Explain the basis for waiting for a faulted S/G to complete depressurization before checking RCS press stable or increasing following the establishment of normal charging and prior to stopping any running SI pumps.

X X X 12. Discuss the purpose of ES-i .2 Post LOCA Cooldown and Depressurization.

X X X 13. Justify the procedure step to shutdown the RHR pumps if RCS pressure is greater than 150 psig.

X X X 14. Identify the procedural transition required if any S!G level continues to increase with feedflow isolated.

3-OT-EOPO1 00 Revl4 Page 6 of 138 pages V. TRAINING OBJECTIVES: (continued) 0 0 0 D F-U) U)

X X X 15. Explain the basis for limiting the RCS cooldown rate to 100°F in one hour.

X X X 16. Discuss the requirement to check RCS subcooling greater than 65°F prior to RCS depressurization.

X X X 17. Describe how depressurization of the RCS might result in the capability to maintain PZR level when PZR level could not be maintained prior to depressurization.

X X X 18. Analyze and explain the process that leads to a new RCS equilibrium pressure following the shutdown of an ECCS pump during the ES-i .2 reduction sequence.

X X X 19. Explain why subcooling is minimized following the alignment of normal charging in procedure ES-i .2.

X X X 20. Discuss and justify the priority of usage given to procedure ES-i .3, Transfer to RHR Containment Sump.

X X X 21. Justify the ES-i .3 procedural requirement to shutdown the SI pumps if RCS press increase to greater than 1350 psig while aligned for sump recirc.

X X X 22. Identify and explain the basis of the interlock on the RHR pump discharge to the SI and CCP suction (FCV-63-8 and 1 1).

X X X 23. State from memory the action required if offsite power is lost following transfer to RHR containment sump cold leg recirc. Explain the basis for the required action.

WBN 10-2011 NRC RO Exam As Submitted 8!151201 I

60. 034 K6.02 060 Given the following:

- Unit 1 at 100% RTP.

- Fuel Assembly shuffles are being made in the Spent Fuel Pit.

- 0-RM-90-102, Spent Fuel Pit Radiation Monitor, has been declared INOPERABLE and removed from service due to an instrument malfunction.

Which ONE of the following identifies...

(1) whether Technical Specifications would allow continued movement of fuel assemblies in the Spent Fuel Pit and (2) the Auxiliary Building isolation that would automatically initiate if 0-RM-90-103, Spent Fuel Pit Radiation Monitor, subsequently detected Hi Radiation?

A. (1) Fuel assembly shuffles must stop; (2) Both trains of isolation would be automatically initiated.

B. (1) Fuel assembly shuffles must stop; (2) Only one train of isolation would be automatically initiated.

C. (1) Fuel assembly shuffles can continue; (2) Both trains of isolation would be automatically initiated.

D (1) Fuel assembly shuffles can continue; (2) Only one train of isolation would be automatically initiated.

Page 157

WBN 10-2011 NRC RO Exam As Submitted 811512011 DISTRA CTOR ANAL YSIS:

A. Incorrect, Fuel shuffles would not have to be stopped, the movement could continue because with only one Rad Mon operable in the Spent fuel Pit and both trains of isolation of the Aux Building would not be isolated if radiation was detected by the other SFP Radiation Monitor. The monitors are trained in respect to isolating the Auxiliary Building with 102 being Train A and 103 being Train B. Plausible because other conditions would cause fuel movement to be stopped.

B. Incorrect, Fuel shuffles would not have to be stopped the movement could continue with only one Rad Mon operable in the Spent fuel Pit. Plausible because other conditions would cause fuel movement to be stopped and only one train of the isolation in the Aux Building is correct.

C. lncorrec1, Tech Specs allows fuel movement in the Spent Fuel Pit with only one SFP Radiation monitor and one ABGTS system operable. Both trains would not be isolated if radiation was detected by the other SFP Radiation Monitor. The monitors are trained in respect to isolating the Auxiliary Building with 102 being Train A and 103 being Train B. Plausible because fuel shuffle continuing is correct.

D. Correct, Tech Specs allows fuel movement in the Spent Fuel Pit with only one SFP Radiation monitor and one ABGTS system operable. The SFP Radiation Monitors are trained in respect to isolating the Auxiliary Building with 102 being Train A and 103 being Train B.

Question Number: 60 Tier: 2 Group 2 KIA: 034 K6.02 Fuel Handling Equipment System (FHES)

Knowledge of the effect of a loss or malfunction on the following will have on the Fuel Handling System Radiation monitoring systems Importance Rating: 2.6 I 3.3 IOCFRPart55: 41.7/45.7 IOCFR55.43.b: Not applicable KIA Match: K/A is matched because the question requires the knowledge of the effect of a loss of spent fuel pool area radiation monitors and the effect of that loss on Fuel Handling Systems, in this case the effect Page 158

WBN 10-2011 NRC RO Exam As Submitted 811512011 KIA Match: K/A is matched because the question requires the knowledge of the effect of a loss of spent fuel pool area radiation monitors and the effect of that loss on Fuel Handling Systems, in this case the effect on whether or not fuel movement can continue and the effect on the actuation of isolations due to a potential fuel handling accident.

Technical

Reference:

1-47W611-30-5 R7 Technical Specifications 3.3.8, Auxiliary Building Gas Treatment System (ABGTS)

Actuation Instrumentation.

Proposed references None to be provided:

Learning Objective: 3-OT-SYSO3OB

13. Given a set of plant conditions, determine the correct response of the ABGT system.

3-OT-T/S0303

3. Given plant parameters/conditions, correctly determine applicable Limiting Conditions for Operation or Technical Requirement limits for the various instrumentation systems covered by T/S or T/R.

Cognitive Level:

Higher X Lower Question Source:

New Modified Bank Bank X Question History: SQN question 034 K6.02 061 used on the SQN 1/2009 RETAKE exam with very minor formatting changes.

Comments:

Page 159

ABGTS Actuation Instrumentation

- 3_.3.8

3. 3 INSTRUMENTATION 3.3.8 Auxiliary Building Gas Treatment System (ABGTS) Actuation Instrumentation LCO 3.3.8 The ABGTS actuation instrumentation for each Function in Table 3.3.8-1 shall be OPERABLE.

APPLICABILITY: According to Table 3.3.8-1 ACTIONS NOTE Separate Condition entry is allowed for each Function.

CONDITION REQUIRED ACTION COMPLETION TIME A. One or more Functions A.l Place one ABGTS train in 7 days with one channel or operation.

train inoperable.

B. One or more Functions B.1.1 Place one ABGTS train Immediately with two channels or in operation.

two trains inoperable.

AND B.1.2 Enter applicable Immediately Conditions and Required Actions of LCO 3.7.12, Auxiliary Building Gas Treatment System (ABGTS), for one train made inoperable by inoperable actuation instrumentation.

OR (continued)

Watts Bar-Unit 1 3.3-61

ABGTS Actuation Instrumentation

3. 3 ;E ACTIONS CONDITION REQUIRED ACTION COMPLETION TIME B. (continued) B.2 Place both trains in Immediately emergency radiation protection mode.

C. Required Action and C.l Suspend movement of Immediately associated Completion irradiated fuel Time for Condition A assemblies in the or B not met during fuel, handling area.

movement of irradiated fuel assemblies in the fuel handling area.

D. Required Action and D.l Be in MODE 3. 6 hours6.944444e-5 days <br />0.00167 hours <br />9.920635e-6 weeks <br />2.283e-6 months <br /> associated Completion Time for Condition A AND or B not met in MODE 1, 2, 3, or 4. D.2 Be in MODE 5. 36 hours4.166667e-4 days <br />0.01 hours <br />5.952381e-5 weeks <br />1.3698e-5 months <br /> SURVEILLANCE REQUIREMENTS NOTE Refer to Table 3.3.8-1 to determine which SR5 apply for each ABGTS Actuation Function.

SURVE I LLANCE FREQUENCY SR 3.3.8.1 Perform CHANNEL CHECK. 12 hours1.388889e-4 days <br />0.00333 hours <br />1.984127e-5 weeks <br />4.566e-6 months <br /> SR 3.3.8.2 Perform COT. 92 days (continued)

Watts Bar-Unit 1 3.362

ABGTS Actuation Instrumentation

- 3.3.8 SURVEILLANCE_REQUIREMENTS__(continued)

SURVEILLANCE FREQUENCY SR 3.3.8.3 NOTE Verification of setpoint is not required.

Perform TADOT. 18 months SR 3.3.8.4 Perform CHANNEL CALIBRATION. 18 months Watts BarUnit 1 3.363

ABGTS Actuation Instrumentation 3.3.8 Table 3.3.81 (page 1 of 1)

ABGTS Actuation Instrumentation APPLICABLE MODES OR OTHER SPECIFIED REQUIRED SURVEILLANCE ALLOWABLE FUNCTION CONDITIONS CHANNELS REQUIREMENTS VALUE I. Manual Initiation 1,2,3,4 2 SR 3.3.8.3 NA (a) 2 SR 3.3.8.3 NA

2. Fuel Pool Area Radiation Monitors (a) 2 SR 3.3.8.1 1161 SR 3.3.8.2 mR/hr SR 3.3.8.4
3. Containment Isolation Refer to LCO 3.3.2, Function 3.a., for all Phase A initiating functions and requirements.

(a) During movement of irradiated fuel assemblies in the fuel handling area.

Watts Bar-Unit 1 3.3-64

1. Given the following: C

- Both Units at 100% RTR

- Fuel Assembly shuffles are being made in the Spent Fuel Pit.

- 0-RM-90-102, Spent Fuel Pit Radiation Monitor, has been declared INOPERABLE and removed from service due to an instrument malfunction.

Which ONE of the following identifies...

(I) whether Technical Specifications would allow continued movement of fuel assemblies in the Spent Fuel Pit, and (2) the Auxiliary Building isolation that would automatically initiate if O-RM-90-1 03, Spent Fuel Pit Radiation Monitor, subsequently detected Hi Radiation?

A. (1) Fuel assembly shuffles must stop; (2) Both trains of isolation would be automatically initiated.

B. (1) Fuel assembly shuffles must stop; (2) Only one train of isolation would be automatically initiated.

C. (1) Fuel assembly shuffles can continue; (2) Both trains of isolation would be automatically initiated.

D (1) Fuel assembly shuffles can continue; (2) Only one train of isolation would be automatically initiated.

3-OT-SYSO3OB ReVision 9 Page 4 of 27 I. PROGRAM Watts Bar Operator Training II. COURSES A. License Training B. NOTP B. License Req ualification D. NAUO Requalification III. TITLE AUXILIARY BUILDING GAS TREATMENT SYSTEM (ABGTS) & AUXILIARY BUILDING ISOLATION SYSTEM IV. LENGTH OF LESSON A. Licensed Training 2 hour2.314815e-5 days <br />5.555556e-4 hours <br />3.306878e-6 weeks <br />7.61e-7 months <br /> B. NOTP 3 hours3.472222e-5 days <br />8.333333e-4 hours <br />4.960317e-6 weeks <br />1.1415e-6 months <br /> V. TRAINING OBJECTIVES A R SS U 0 R.T 0 0 A X X X X 1. State the design basis of the ABGTS system in accordance with

. FSAR section 6.2.3.

X X *X X 2. State the function of the ABGT system in accordance with the system description.

X: X X X. 3. State the function of the ABI system in accordance with the system description.

X X X X 4. Sketch the Auxiliary Building Gas Treatment System from the suction to the discharge.

X X X X 5. List the general areas from which the ABGTS takes sution and to which it discharges.

X X X X 6. List the components that make up the ABGTS filter housing and

  • the purpose of each.

3-OT-SYSO3OB ReviiQn9 Page 5 of 27 A R S S U R T 0 0 A X X X X 7. Given a loss of instrument air/control power, determine the

. effect on the ABGTS dampers.

X X X X 8. Describe how pressure is maintained in the Auxiliary Building Secondary Containment Enclosure (ABSCE) by the ABGTS and state the pressure.

9. Objective deleted.

X X X X 10. Describe the interlocks for the ABGTS humidity control heater operation.

X* X X X 11. State the power supplies to the ABGTS.

X X X X 12. Describe the automatic start signals for the ABGTS.

X X X X 13. Given a set of plant conditions, determine the correct response of the ABGT system.

X X X X 14. Explain the conditions and precautions associated with the operation of the ABGTS as listed in SQl 30.6.

X X X X 15. Identify the ABGTS parameters that are governed by TS.

X X X 16. Regarding Technical Specifications and Technical Requirements for this system:

a. Identify the conditions and required actions with completion time of one hour or less.
b. Explain the Limiting Conditions for Operation, Applicability,

. and Bases.

c. Given a status/set of plant conditions, apply the appropriate Technical Specifications and Technical Requirements.

X X X X 17. Correctly locate control room controls and indications for the ABGT system, including:

a.Fans

. b. Dampers X X X X 18. Describe the Auxiliary Building Secondary Containment

. Enclosure (ABSCE) as to structure and penetrations.

X X X X - 19. Describe the ways that Auxiliary Building Isolation (ABI) can be initiated.

X X X 20. Correctly locate the control room ABI hand switches.

3-OT-SYSO3OB Revision 9 Page 6 of 27 A R S S U 0 R T 0 0 A X X X X 21. Describe the steps involved to reset and recover from an Auxiliary Building Isolation.

3-OT-T/S0303 Revisioa4 Page 3 of 3 I. PROGRAM WATTS BAR OPERATOR TRAINING II. COURSE A. LICENSE TRAINING B. LICENSED REQUAL III. TITLE T/S AND T/R 3.3, INSTRUMENTATION AND BASES IV. LENGTH OF LESSON A. LICENSE TRAINING 1 Hrs LICENSED OPERATOR REQUAL TIME WILL BE DETERMINED AFTER OBJECTIVES ARE IDENTIFIED.

V. TRAINING OBJECTIVES ARSS UORT 0 OA X X X 1. Demonstrate the ability to extract specific information from the Technical Specification, and Technical Requirements, as they pertain to Instrumentation Systems.

X X X 2. Determine the bases for OPERABILITY of the Reactor Trip System or other instrumentation system.

X X X 3 Given plant parameters/conditions, correctly determine applicable Limiting Conditions for Operation or Technical Requirement limits for the various instrumentation systems covered by T/S or T/R.

X X X 4. Given plant parameters/conditions, correctly determine the OPERABILITY of the various instrumentation systems covered by T/S or T/R.

X X X 5. Given plant parameters/conditions, correctly determine applicable Action Conditions, Required Actions, and Completion Times for the various instrumentation systems covered by T/S or T/R.

WBN 10-2011 NRC RO Exam As Submitted 811512011

61. 041 A2.02 061 Given the following:

- Unit 1 was operating at 100% power when a reactor trip occurred.

- A steam dump Tavg Hi bistable malfunction resulted in 6 Steam Dump valves remaining full open as Tavg dropped following the reactor trip.

Which ONE of the following identifies...

(1) the effect Tavg dropping to 550°F will have on the steam dump valves and (2) when the steam dump valves would have closed if the operators had placed the handswitches listed below to OFF prior to Tavg reaching 550°F?

Note:

1-HS-1-103A, STEAM DUMP FSV A 1-HS-1-103B, STEAM DUMP FSV B A (1) All 6 of the steam dump valves will close.

(2) After 1-HS-1-103A was placed to OFF.

B. (1) All 6 of the steam dump valves will close.

(2) ONLY after 1-HS-1-103A and 1-HS-1-103B were placed to OFF.

C. (1) ONLY 3 of the valves will close but the 3 cooldown valves will remain open.

(2) After I -HS-1 -1 03A was placed to OFF.

D. (1) ONLY 3 of the valves will close but the 3 cooldown valves will remain open.

(2) ONLY after 1-HS-1-103A and 1-HS-1-103B were placed to OFF.

Page 160

WBN 10-2011 NRC RO Exam As Submitted 8/15/2011 DISTRA CTOR ANAL YSIS:

A. Correct, When Tavg drops to 550°F, the Lo-Lo Tavg interlock causes all 6 of the steam dumps valves to close. Placing either of the handswitches to OFF will result in removing the air to the steam dump valves thus causing the valves to close; so placing 1-HS-1-103A to OFF will close the valves.

B. Incorrect, Plausible because all 6 steam dump valves closing is correct and placing both handswitches to OFF is a action directed in some procedures when the steam dump valves are to be closed (e.g., AOl-39).

C. Incorrect, Plausible because there is a relationship between the cooldown valves and 550°F. The cooldown valves are the only steam dump valves that can be reopened with Tavg less than 550°F, but require operator action to bypass the interlock. Also, because placing both handswitches to OFF will result in removing the air to the steam dump valves thus causing the valves to close; so placing 1-HS-1-103A to OFF will close the valves.

D. Incorrect, Plausible because there is a relationship between the cooldown valves and 550°F. The cooldown valves are the only steam dump valves that can be reopened with Tavg less than 550°F, but require operator action to bypass the interlock. Also, because placing both handswitches to OFF is a action directed in some procedures when the steam dump valves are to be closed (e.g., AOl-39).

Question Number: 61 Tier: 2 Group 2 K/A: 041 A2.02 Steam Dump System (SDS)ITurbine Bypass Control Ability to (a) predict the impacts of the following malfunctions or operations on the SDS; and (b) based on those predictions or mitigate the consequences of those malfunctions or operations:

Steam valve stuck open Importance Rating: 3.6 I 3.9 10 CFR Part 55: 41.5 / 43.5/45.3/45.13 IOCFR55.43.b: Not applicable K/A Match: K/A is matched because the question requires the ability to predict how Tavg dropping due to steam dump valves being stuck open will affect the dump valves and the identification of the action required by the operating crew to mitigate the affect prior to reaching a Lo-Lo setpoint.

Page 161

WBN 10-2011 NRC RO Exam As Submitted 8/15/2011 Technical

Reference:

1-47W611-1-2 R12 SQl-i .02, Steam Dump System, Revision 0013 Proposed references None to be provided:

Learning Objective: 3-OT-SYSOOI B

04. Identify the purpose and function of the steam dump valves designated as cooldown valves.
18. Explain when the bistables open the steam dump valves and how many valves will be tripped open at one time by the Hi or Hi-Hi bistables.

Cognitive Level:

Higher X Lower Question Source:

New Modified Bank X Bank Question History: WBN bank question SYSOO1 B.25 007 modified.

Comments:

Page 162

WBN Steam Dump System 501-1.02 Unit I Rev. 0013 Pagel2ofl5 Date________ INITIALS 7.0 SHUTDOWN 7.1 Steam Dump Operation During Unit Shutdown NOTE The Steam Dump System should be left in service as long as condenser vacuum and feedwater flow to the S/Gs are maintained or until RCS Tavg is 200°F.

[1] ENSURE Steam Dump System in Steam Pressure Mode per SubSection 6.2 of this Instruction.

[2] IF Cooldown will NOT continue below 550°F, THEN NIA remainder of Section 7.1.

[3] IF Cooldown is to continue below 550°F, THEN COMPLETE remainder of Section 7.1.

CAUTION If temperature rises above 550°F, Bypass Interlock will reset.

NOTE Tavg less than or equal to 550°F is a closing interlock for steam dump cooldown valves. The following step will enable the three cooldown valves to operate.

(Gr

[4] WHEN Tavg less than or equal to 550°F, and Window 68A C

[XA-55-4A], P-12, LO-LO TAVG STM DUMP BLOCK, annunciator is LIT, THEN PLACE and HOLD 1-HS-1-103A, STEAM DUMP FSV A, and 1-HS-1-103B, STEAM DUMP FSV B, in BYPASS INTLK.

[5] WHEN annunciator windows 67E, STM DMP TR-A COOLDOWN INTLK BYPASSED [XA-55-4A], and 68E, STM DMP TR-B COOLDOWN INTLK BYPASSED

[XA-55-4A], are LIT, THEN RELEASE 1-HS-1-103A and 1-HS-1-103B.

QUESTIONS REPORT for ILT EXAM BANK MARCH 2007

1. SYSOO1B.25007 The following conditions exist on Unit 1:

- Reactor power 50%

- Auct High Tavg = Tref

- Steam Dumps in Tavg Mode An instantaneous 20% load rejection AND simultaneous failure of Loop #3 Cold Leg RTD HIGH have occurred. Assuming NO operator action, Which of the following describes the response of the steam dump system?

a All twelve of the steam dump valves would actuate and OPEN until Tavg drops to 550°F, which would block the steam dumps and stop the cooldown.

b. ONLY the first TWO groups of steam dump valves (6 valves) would actuate and OPEN until Tavg drops to 550°F, which would block the steam dumps and stop the cooldown.
c. ONLY the first group of steam dump valves (3 valves) would actuate and OPEN until Tavg drops to 550°F, which would block the steam dumps and stop the cooldown.
d. All twelve of the steam dump valves would actuate and OPEN and would continue to blowdown past indicated Tavg of 550° F, RCS cooldown would not be stopped.

The correct answer is A Monday, August 08, 2011 8:44:35 AM

OPERATIONS 3-OT-SYS0OI B STEAM DUMP CONTROL SYSTEM REVISION 9 Page 5 of 38 INSTRUCTOR GUIDE PROGRAM Watts Bar Operator Training IL COURSES License Training NOTP License Requalification NAUO Requalification III. TITLE Steam Dump Control System IV. LENGTH OF LESSON A. License Training 2 Hours B. NOTP 3 Hours License Requalification and NAUO Requalification times will be determined after objectives are identified.

V. TRAINING OBJECTIVES AR S S U OR T 0 0 A X X X X 01. State the function/purpose of the steam dump system in accordance with the system description.

X X X 02. State the design bases of the steam dump system in accordance with .FSAR section 10.4.4.

X X X X 03. Describe the dump valves as to, number, flow capacity, and failure position.

X X X 04. Identify the purpose and function of the steam dump valves designated as cooldown valves.

X X X X 05. Given a loss of instrument air/control power, determine the effect on the STEAM DUMP valves.

I OPERATIONS 3-OT-SYSOOI B REVlS[O STEAM DUMP CONTROL SYSTEM Page6of38 INSTRUCTOR GUIDE AR S S U OR T 0 0 A X X X X 06. Identify the control board indications and the arming signals of the A and B solenoids of the Steam Dump System.

X X X X 07. Identify the MODES of operation of the Steam Dump Control System and when each mode would be used.

X X X 08. Identify which mode of steam dump control will be used to maintain the unit in HOT STANDBY, prior to increasing unit load.

X X X X 09. Identify which steam dump controller is in service when Tavg mode is selected with the unit at power.

X X X X 10. Identify which steam dump controller is in service upon a reactor trip from full power operation.

X X X X 1 1. Describe the reset function of the Steam Dump Control System Mode Switch.

X X X X 12. Describe the condition for using the Bypass Interlock handswitch position in Steam Dump Control, what function the switch performs and how many valves will operate.

X X X X 13. Explain how the main steam header pressure is controlled while the Steam Dump System is in Steam Pressure Mode.

X X X X 14. List the Steam Dump interlocks, including indications, and explain the purpose of each.

X X X 15. Describe the two signals fed to the Steam Dump Load Rejection Controller and the two signals fed to the Steam Dump Reactor Trip Controller.

X X X X 16. Draw the load rejection controller, explaining the curve, when it is in service and its inputs.

X X X X 17. Draw the reactor trip controller, explaining the curve, when it is in service and its inputs.

X X X X 18. Explain when the bistables open the steam dump valves and how many valves will be tripped open at one time by the Hi or Hi-Hi bistables.

19. OBJECTIVE DELETED.

OPE RATIONS 3-OT-SYSOO1 B STEAM DUMP CONTROL SYSTEM REVISION 9 Page 7 of 38 INSTRUCTOR GUIDE AR S S U 0 R T.

0 0 A X X X X 20. Explain how the steam dump bistables and steam dump controllers differ in how they operate the steam dump valves.

X X X X 21. Describe the functions of the flow controllers located on panel 1 -L 65, El 708, T6H of Turbine Building.

X X X X 22. List the sequence that the steam dump valves open and close.

X X X X 23. Describe the position in which the Bailey Positioner Supply and Bypass Valve handles must be in for automatic operation of the Steam Dump Valve; for handwheel operation of the Steam Dump Valve.

X X X X 24. Explain which direction the steam dump valve handwheel must be turned to manually open a valve.

X X X 25. Given a steam dump instrument and failure mode, identify how the instrument will respond and what interlock(s) or control function(s) will be affected, including effects on system/component operation.

X X X 26. Given a set of plant conditions, determine the correct response of the steam dump system.

X X X 27. Evaluate the result of a steam dump system malfunction on the Reactor Coolant System and on the reactor power level.

OPE RATIONS 3-OT-SYSOO1 B RVIS[O$ 9 1-,I STEAM DUMP CONTROL SYSTEM INSTRUCTOR GUIDE Page 8 of 38 alp AR S S U OR T 0 0 A X X X 28. Correctly locate control room controls and indications associated with the steam dump system, including:

a. Mode Switch.
b. Train A & B bypass interlock switches.
c. Valve position status panel.
d. Demand meter.
e. D solenoid indicating light.
f. Armed indicating light.
g. Pressure controller.
h. Main Steam Pressure Indicator (P1-I -33).
i. Annunciator windows for C-9, C-7, and Train A & B cooldown interlock bypassed.

X X X X 29. Describe the in-plant locations of the following:

a. Main Steam Dump Valves.
b. A, B, & D Solenoid Valves for each dump valve.
c. Steam Dump Valve Manual Isolation Valves.
d. Local Panel l-L-65.

WBN 10-2011 NRC RO Exam As Submitted 811512011

62. 071 K3.05 062 Given the following:

- Unit 1 is operating at 100% power.

- Waste Gas Decay Tank J relief valve develops a flange leak and the tank contains high activity gas.

Which ONE of the following identifies how the radiation monitors listed below will respond to the gas release?

Note:

0-RE-90-1 18, Waste Gas Rad Monitor 0-RE 101, Auxiliary Building Ventilation Monitor A. Only 0-RE-90-1 18 will detect the release.

B Only 0-RE-90-101 will detect the release.

C. Neither 0-RE-90-1 18 nor 0-RE-90-1 01 will detect the release.

D. Both 0-RE-90-1 18 and 0-RE-90-1 01 will detect the release.

Page 163

WBN 10-2011 NRC RO Exam As Submitted 8/1512011 DISTRA CTOR ANAL YSIS:

A. Incorrect, leakage must enter the WGDT release line to pass by waste gas radiation monitor 0-RE-90-1 18. Flange leakage would not enter this line but rather the general Aux Building Spaces where it would eventually pass by Aux building stack radiation monitor 0-RE-QO-IQI. Therefore for flange leakage, Q-RE-90-1 18 would not detect the release but 0-RE-90-101 would. Plausible if applicant does not understand the relationship between the ventilation system and rad monitors.

B. Correct, leakage must enter the WGD T release line to pass by waste gas radiation monitor 0-RE-90-118. Flange leakage would not enter this line but rather the general Aux Building Spaces where it would eventually pass by Aux building stack radiation monitor 0-RE-90-101. Therefore for flange leakage, 0-RE-90-118 would not detect the release but 0-RE-90-101 would detect the release.

C. Incorrect, leakage must enter the WGDT release line to pass by waste gas radiation monitor Q-RE-90-118. Flange leakage would not enter this line but rather the general Aux Building Spaces where it would eventually pass by Aux building stack radiation monitor 0-RE 101. Therefore for flange leakage, 0-RE 118 would not detect the release but 0-RE-90-101 would detect the release. Plausible if applicant does not understand the relationship between the ventilation system and rad monitors and thinks the gas would exit via the Shield Building Stack which is where a normal gas decay tanks release is routed.

D. Incorrect, leakage must enter the WGDT release line to pass by waste gas radiation monitor 0-RE-90-1 18. Flange leakage would not enter this line but rather the general Aux Building Spaces where it would eventually pass by Aux building stack radiation monitor 0-RE-90-101. Therefore for flange leakage, 0-RE-90-1 18 would not detect the release but 0-RE-90-101 would detect the release. Plausible if applicant does not understand the relationship between the ventilation system and rad monitors.

Question Number: 62 Tier: 2 Group: 2 K/A: 071 K3.05 Waste Gas Disposal System (WGDS)

Knowledge of the effect that a loss or malfunction of the Waste Gas Disposal System will have on the following:

ARM and PRM systems Importance Rating: 3.2 I 3.2 IOCFRPart55: 41.7/45.6 IOCFR55.43.b: Not applicable Page 164

WBN 10-2011 NRC RO Exam As Submitted 8115/2011 K/A Match: K/A is matched because the question requires knowledge of how a leaking flange on the will affect the radiation monitoring systems at the station.

Technical

Reference:

1-47W830-4 R20 1-47W866-1 R57 1 -47W866-2 R27 1-47W866-1O R33 Proposed references None to be provided:

Learning Objective: 3-OT-SYSO3OA

4. Describe all Auxiliary Building HeatingNentilation/Air-Conditioning System major components as to power supplies, start and stop logic and operations.

3-OT-SYSO77B

9. List each of the waste gas release points at the plant.

Cognitive Level:

Higher Lower X Question Source:

New Modified Bank Bank X Question History: SQN question 060 AA1 .02 022 that was used on the January 2008 exam with some wording changes in the stem and all choices, but not a modified question.

Comments:

Page 165

Sr\t GtiiJ) 1

&4e-fflOA-060 AA1.02 022 Waste Gas Decay Tank J contains high activity gas. Waste Gas Decay Tank J relief valve d elops a large flange leak.

Which ONE (1) of the following would identify the response of the Rad Monitors listed?

A. Waste Gas Rad Monitor (RE-90-1 18) would alarm, but the Auxiliary Building Ventilation Monitor (RE-90-1 01) would NOT alarm.

B. Waste Gas Rad Monitor (RE- 90-118) would alarm, and the Auxiliary Building Ventilation Monitor (RE-90-101) would alarm.

C Waste Gas Rad Monitor (RE- 90-118) would NOT alarm, but the Auxiliary Building Ventilation Monitor (RE-90-1 01) would alarm.

D. Waste Gas Rad Monitor (RE- 90-118) would NOT alarm, and the Auxiliary Building Ventilation Monitor (RE-90-101) NOT alarm.

3-OT-SYSO30A Revision 10 Page4of73 I. PROGRAM Watts Bar Operator Training II. COURSES A. License Training B. Non-License Training C. License Requal D. Non-License Requal III. TITLE AUXILIARY BUILDING VENTILATION SYSTEM IV. LENGTH OF LESSON A. Licensed Training 2.0 hours0 days <br />0 hours <br />0 weeks <br />0 months <br /> B. Non-Licensed Training 2.0 hours0 days <br />0 hours <br />0 weeks <br />0 months <br /> C. NOTP 4.0 hours0 days <br />0 hours <br />0 weeks <br />0 months <br /> V. TRAINING OBJECTIVES A R S S U 0 R T 0 0 A X X X X 1. State the design basis of the Auxiliary Building HeatingNentilation/Air Conditioning system in accordance with FSAR section 9.4.3.

X X X 2. Regarding Technical Specifications and Technical Requirements for this system:

a. Identify the conditions and required actions with completion time of one hour or less.
b. Explain the Limiting Conditions for Operation, Applicability,

- and Bases.

c. Given a status/set of plant conditions, apply the appropriate Technical Specifications and Technical Requirements.

X X X X 3. Explain when Auxiliary Building area temperature monitoring is applicable.

3-OT-SYSO3OA Revision 10 Pa5Of73 A R S S U 0 R T 0 0 A X X X X 4. Describe all Auxiliary Building Heating/Ventilation/Air-Conditioning System major components as to power supplies, start and stop logic and operations.

X X X X 5. Explain why building pressure control is important in the Auxiliary, Control and Annulus Buildings.

X X X X 6. List the ABI initiation signals.

X X X X 7. Explain what events take place on an ABI and why.

X X X X 8. Discuss how the Shutdown Board rooms are ventilated and cooled.

X X X X 9. Describe the purposes of the 480v Electrical Board Rooms Pressurizing Fans.

X X X X 10. Describe the purpose for ventilation in the I 25v Vital Battery rooms.

X X X X 11. State the temperature at which the exhaust fans for the north and south valve vault rooms start.

X X X X 12. Explain the primary concern with a temperature of < 50°F in the north and south valve vaults.

X X X X 13. Describe the electrical logic, including trips, for the Fuel Handling Exhaust Fans.

X X X X 14. Describe how the ECCS rooms and ESF areas are cooled.

X X X X 15. Describe the emergency ventilation systems provided for the Turbine Driven Auxiliary FW Pump Rooms.

X X X X 16. Explain how the Post Accident Sampling Facility is ventilated.

X X X X 17. [Explain the need for checking the position of backdraft dampers on ventilation equipment when standby fans are placed in service and/or on shift routines.] WBPER92O218 X X X X 18. Explain the purpose of the Auxiliary Building Heating System.

X X X X 19. State the purpose and describe the basic operation of the major components of the Auxiliary Building Heating System.

3-OT-SYS03OA Revision 10 Page6of73 A R S S U 0 R T 0 0 A X X X X 20. Explain how the hot water flow to the Auxiliary Building preheat coils is regulated.

X X X X 21. State how to place the Auxiliary Building air preheating system in service.

X X X X 22. List the precautions concerning the Auxiliary Building heat system.

X X X X 23. Describe what must be done if the air temperature entering the auxiliary building heating and cooling coils is < 35°F.

X X X X 24. Explain the purpose of the Auxiliary Building Cooling System.

X X X X 25. Explain the basic operating principle of the Auxiliary Building General Vent Chiller Package.

X X X X 26. Describe the chilled water flowpath through the Auxiliary Building Cooling System.

X X X 27. Correctly locate control room controls and indications associated with the Auxiliary Building Heating/Ventilation/Air Conditioning system, including:

a. Auxiliary Building General Supply Fans
b. Auxiliary Building General Exhaust Fans
c. Tornado Dampers
d. Battery Room Exhaust Fans

. e. Shutdown Board Room Pressurizing Fans X X X X 28. Describe the in-plant location of the following:

a. Auxiliary Building General Supply and Exhaust Fans
b. Fuel Handling Area Exhaust Fans
c. Auxiliary Building General Vent Chillers
d. Auxiliary Building General Vent Chillers Chilled Water System Components

3-OT-SYSO77B Revision 9 Page 5of38 I. PROGRAM Watts Bar Operator Training II. COURSES A. License Training B. Non-License Training III. TITLE Waste Gas System IV. LENGTH OF LESSON A. Licensed Training 3 hour3.472222e-5 days <br />8.333333e-4 hours <br />4.960317e-6 weeks <br />1.1415e-6 months <br /> B. Non-Licensed Training 3 hours3.472222e-5 days <br />8.333333e-4 hours <br />4.960317e-6 weeks <br />1.1415e-6 months <br /> V. TRAINING OBJECTIVES A R S S U 0 R T 0 0 A X X X X 1. Discuss the purpose of the Waste Gas System in accordance with FSAR 11.3.

X X X X 2. List the equipment that vents to the vent header.

X X X X 3. Sketch the Waste Gas Compressor, include arrows showing gas flow and liquid flow.

X X X X 4. Describe the waste gas compressors, include power supplies, type and means of sealing.

X X X X 5. Explain how the compressors operate, include starts and stops.

X X X X 6. Describe how PCV-77-89 and 97 operate and their purpose.

X X X X 7. Describe the gas decay tanks, include number and maximum pressure.

X X X X 8. List the sources of gases and types in the waste gas system.

X X X X 9. List each of the waste gas release points at the plant.

X X X 10. Describe the general procedure to make a gaseous release.

3-OT-SYSO77B Revision 9 Pe 6 of 38 A R S S U 0 R T 0 0 A X X X 1 1. List the procedure that implements the Technical Specification requirements for Explosive Gas and Storage Tank Radioactive Monitoring.

X X X X 12. Identify the cause of High 02 in the Reactor Coolant Drain Tank as described in WBPER961228.

X X X X 13. [Identify the cause of high radiation at Kewaunee Nuclear Plant as reflected in CE 1867.]

X X X X 14.Demonstrate the ability to determine the probable cause and corrective actions in response to a local alarm using ARI O-L 2A, ARI O-L-2B, ARI O-L-2C or ARI O-L-2D.

WBN 10-2011 NRC RO Exam As Submitted 8/1512011

63. 072 A4.01 063 Given the following:

- Unit I is operating at 100% power.

- While performing a board walkdown, the incoming OAC determines annunciator 174-E, 1-RR-90-1 AREA MONITORS INSTR MALE, is LIT.

Which ONE of the following identifies...

(1) how the operator would determine which Area Radiation Monitor was the cause of the annunciator being LIT and (2) a condition that would cause the alarm?

A (1) GREEN light DARK on the associated rate meter.

(2) Rate meter function switch in ALARM ADJ B. (1) GREEN light DARK on the associated rate meter.

(2) Sample flow less than setpoint C. (1) RED, AMBER and GREEN lights LIT on the associated rate meter.

(2) Rate meter function switch in ALARM ADJ D. (1) RED, AMBER and GREEN lights LIT on the associated rate meter.

(2) Sample flow less than setpoint Page 166

WBN 10-2011 NRC RO Exam As Submitted 811512011 DISTRA CTOR ANALYSIS:

A. Correct; In accordance with ARI 1 74-E, a corrective action is to CHECK rate meters [O-M-12] for loss of green light to determine which instrument is malfunctioning and the function switch being in the ALARM ADJ position is listed as a probable cause of the alarm.

B. Incorrect, Plausible because the associated rate meter green light being dark is correct and a sample flow less than setpoint will cause an instrument malfunction alarm on a process radiation monitor.

C. Incorrect, Plausible because there are condition that will cause all 3 lights to be lit on an area radiation monitor (e.g. momentary loss of power) and the function switch being in the ALARM ADd position is listed as a probable cause of the alarm.

D. Incorrect, Plausible because there are condition that will cause all 3 lights to be lit on an area radiation monitor (e.g. momentary loss of power) and a sample flow less than setpoint will cause an instrument malfunction alarm on a process radiation monitor.

Page 167

WBN 10-2011 NRC RO Exam As Submitted 8/1512011 Question Number: 63 Tier: 2 Group: 2 KIA: 072 A4.01 Area Radiation Monitoring System Ability to manually operate and/or monitor in the control room:

Alarm and interlock setpoint checks and adjustments Importance Rating: 3.0* /33 10 CFR Part 55: 41.7/ 45.5 to 45.8 IOCFR55.43.b: Not applicable K/A Match: K/A is matched because the question requires the ability to monitor the conditions of the area radiation monitor using control room indications and the knowledge of the effect of placing the function switch alarm adjustment position.

Technical

Reference:

ARI-1 73-1 79, U-i Radiation Detectors, Revision 0046 Proposed references None to be provided:

Learning Objective: 3-OT-SYSO9OA

07. Determine Interlocks and/or cause-effect relationships between the Rad Monitoring Systems (ARM & Process) and the areas they monitor.

Include HVAC systems and area isolations.

Cognitive Level:

Higher Lower X Question Source:

New Modified Bank X Bank Question History: WBN bank question SYSO9OA.32 003 modified.

Comments:

Page 168

a OM12 jiI U-i RADIATION DETECTORS F

WN (J1 adiation Detectors ARt-I 73-1 79 Unit I Rev. 0046 Pagel6of5l 174-E Source Setpoint 1-RR-90-I 1 -RM-90-1 Instrument failure MONITORS 1-RM-90-2 INSTR MALF O-RM-90-3 O-RM-90-4 /

O-RM-90-5 (Page 1 of 1) 1-RM-90-6 1 -RM-90-7 1-RM-90-8 O-RM-90-9 I -RM-90-1 0 0-RM-90-1 1 I -RM-90-59 1 -RM-90-60 1 -RM-90-61 0-RM-90-230 O-RM-90-231 Probable A. Loss of signal from detector; i.e., loss of counts Cause: B. Failure of readout modules intern I power supply C. Function switch in ALARM ADJ D. 0-BKR-278-MOI2A OPEN [1-M-7/120V AC Instrument Power Rack AIBkr 23))

E. 0..BKR-278-M012B OPEN [1-M-7/120V AC Instrument Power Rack AIBkr 27]

Corrective [1] CHECK rate meters [0-M-12] for loss of green light to determine which Action: instrument is malfunctioning.

[2] ENSURE green operating lights lit and counts indicating on readout modules:

IF green operating light NOT lit and no counts are present on readout module, THEN PERFORM source check on channel that has lost counts: c.i]

[2.1] PLACE function switch to CHECK.

[2.2] OBSERVE meter rise, and DEPRESS green light.

[2.3] RETURN function switch to OPERATE.

[3] NOTIFY Radiation Protection to evaluate compensatory monitoring.

[4] NOTIFY Instrument Maintenance to investigate alarm.

References:

47W61 0-90-2 45N1651-3, -6

WBN U-i Radiation Detectors AI-13-179 Unit I Rev. 0046 Page 9 of 51 173-D Source Setpoint 1-FS-90-1 12A <5.6 SCFM; > 8 in Hg vac UPR CNTMT AIR 1-FIS-90-112C 1 SCFM INSTR MALF 1-RM-90-1 1 2A Instrument failure 1-RM-90-112B 1-RM-90-112C (Page 1 of 1)

Probable A. Loss of signal from detector Cause: B. Failure of readout modules internal power supply C. Breaker #35 on 120V ac Vital Tnstrument Power Bd 1-li. OPEN D. Sample flow less than setpoint E. Sample vacuum abnormal F. Function switch in ALARM ADJ G. Filter failure NOTES

1) 1 -RE-90-1 12 has NO AUTO flow control.
2) Computer points Ri 01 5A, R1O16A and R1O17A correspond to the listed source channels.

Corrective [1] CHECK 1-RM-90-1 12A, B or C Operate green light lit, IF NOT, THEN Action: PERFORM source check on affected channel and reset the monitor.

[2] ENSURE correct valve and switch alignment on panel O-M-12 per SOI-90.02, Gaseous Process Radiation Monitors.

[3] DISPATCH Operator with Radiation Protection coverage to investigate alarm and initiate corrective action.

[4] NOTIFY Chemistry Countroom to initiate sampling lAW 1-Sl-90-25 if required.

[5] NOTIFY Instrument Maintenance to investigate alarm.

[6] IF 1-RM-90-1 12 is aligned to lower containment, THEN REFER TO Tech Specs (LCO 3.4.15).

References:

47W61 0-90-3 45N706-2 45N1651-5, -6, -13 SOI-90.02 1-S 1-90-25

SYSO90A.32 003 1 Qo Which of the following will cause an instrument malfunction alarm for Upper Compartment Radiation Monitor RM-90-112?

a. Selector switch in trip adjust position.
b. Monitor transferred to alternate power supply.
c. Monitor saturated (offscale high).
d. Vent Isolation Rad Mon Block switch, O-HS-90-136A1, selected to RM-II2AB and pulled.

The correct answer is A Rev 6/8/94 by AVW

0 3-OT-SYSO9OA Revision 18 PeTOf80 PROGRAM Watts Bar Operator Training II. COURSES LICENSE TRAINING NOTP LICENSED REQUALIFCATION NAUO REQUALIFCATION III. TITLE RADIATION MONITORING SYSTEM IV. LENGTH OF LESSON A. Licensed Training 4 hour4.62963e-5 days <br />0.00111 hours <br />6.613757e-6 weeks <br />1.522e-6 months <br /> B. NOTP 8 hours9.259259e-5 days <br />0.00222 hours <br />1.322751e-5 weeks <br />3.044e-6 months <br /> License Requalification and NAUO Requalification times will be determined after objectives are identified.

V. TRAINING OBJECTIVES AR S S U OR T 0 0 A X X X X 01 State the Function of the Radiation Monitoring System (RMS) per the System Description.

X X X X 02. State the Design Basis of the Area Radiation Monitoring (ARM) system per FSAR 12.3.4.1.

X X X X 03. Explain the basic operation of a G-M Tube.

X X X X 04. Identify MCR panel where Area Radiation Monitors read out.

X X X X 05. Describe the purpose of a source check and how to perform a source check on a Rad monitor.

X X X X 06. Identify 10 areas that use Area Radiation Monitors (ARMs).

o 3-OT-SYSO9OA Revision 18 Pe8 Of 80 AR S S U OR T 0 CA X X X X 07. Determine Interlocks and/or cause-effect relationships between the Rad Monitoring Systems (ARM & Process) and the areas they monitor. Include HVAC systems and area isolations.

X X X 08. Regarding Technical Specifications and Technical Requirements for this system:

a. Identify the conditions and required actions with completion time of one hour or less.
b. Explain the Limiting Conditions for Operation, Applicability, and Bases.
c. Given a status/set of plant conditions, apply the appropriate Technical Specifications and Technical Requirements.

X X X X 09. Explain Actions to take if an Area Rad Monitor alarms in a work area.

X X X X 10. Explain Purpose of the Post Accident Area Monitors (PAARMs).

X X X X 1 1. Identify cause(s) for a Post Accident Area Monitor malfunction alarm.

X X X X 12. Identify 5 areas that use Post Accident Area Rad Monitors.

X X X X 14. State Purpose of the Main Steam Line Post Accident Monitors X X X X - 15. Describe where the Main Steam Line Monitors read out.

X X X X 16. Identify where Post Accident Monitors are used & read out.

17.DELETED X X X X 18. Describe Location and Components of the Shield Bldg Vent Post Accident Monitors.

X X X X 19. Explain the Eberline Monitor status lamps and local alarms.

X X X X 20. Describe the Design Basis of the Continuous Air Monitors (CAMs) per FSAR 12.3.4.2.

X X X X 21. Identify 5 locations that use portable CAMs.

X X X X 22. DELETED

3-OT-SYSO9OA Revision 18 P9Of80 AR S S U OR T 0 CA X X X X 23. Explain how a Scintillation detector operates.

24. DELETED X X X X 25. DELETED X X X X 26. DELETED X X X X 27. Describe the Design Basis of the Process Radiation Monitoring System per FSAR 11.4.1.

X X X X 28. Describe the advantages and disadvantages to off-line process radiation monitors.

X X X X 29. Describe Local Process Monitor Controls, Alarms, & Indications.

X X X X 30. Identify 3 types of Process Monitors.

X X X X 31. Identify 6 places that use Process Monitors.

X X X X 32. Describe the Radiation Detectors associated with a Gaseous Process Monitor (PIG).

33. DELETED.
34. DELETED.
35. DELETED.

X X X 36. Correctly Locate Rad Monitoring System MCR Controls &

Indications, including:

a. Radiation Monitor Block Switches.
b. Upper and Lower Cntmt Monitors and Recorders.
c. Continuous Air Monitor Recorders.
d. Area Monitor Recorders.

WBN 10-2011 NRC RO Exam As Submitted 811512011

64. 075 G2.1.31 064 Given the following:

- Unit I has reduced power due to a condenser tube leak that has been identified in the East Side tubes.

- The unit is currently at 42% power.

- CCWP I D has been removed from service.

- CCWPs 1A, 1B, and IC are each running 235 amps.

- East side amertap system has been shutdown.

- Condenser Vacuum Pumps IA and lB are in service.

Which ONE of the following identifies an additional operating condition that must be established when removing the East Side Waterboxes from service in accordance with SOI-27.O1, Condenser Circulating Water System?

A. Turbine load must be reduced to less than 35%.

B An additional CCWP is required to be stopped.

C. Stop one of the Condenser Vacuum Pumps.

D. Discharge valves on running CCWPs must be throttled.

Page 169

WBN 10-2011 NRC RO Exam As Submitted 8/1512011 DIS TRACTOR ANAL YSIS:

A. lncorrect, Plausible because dropping to less than 35% is recommended it is not a requirement and the procedure states ... Operation in this condition should be below the line of the Backpressure vs. Load limitation Curve below. While load below 35% is recommended, the actual power during performance is at the Shift Managers discretion.

B. Correct, In accordance with SOl-027.O1, Section 8.6.1 has a caution stating Maximum of two CCW Pumps may be RUNNING when either Water box is isolated. Also see Precaution B below.

C. Incorrect, Plausible because the air removal line from the water box is required to be isolated but the vacuum pump is not required to be stopped.

D. Incorrect, Plausible because there are conditions that allow/require the CCWP discharge valves to be throttled when less than 4 CCWPs are running, but not when removing a set of waterboxes with three pumps running. See Precaution B below.

3.0 PRECAUTIONS AND LIMITATIONS B. Condenser Inlet & Outlet valves should NOT be opened or fully closed with four, (4) CCWPs operating. One Condenser flow path may be isolated with two, (2) -pump operation.

C. When operating with less than 4 CCWP5, pump discharge valves may be throttled to maintain pump amps less than 256 (rated).

Page 170

WBN 10-2011 NRC RO Exam As Submitted 811512011 Question Number: 64 Tier: 2 Group: 2 KIA: 075 G2.1.31 Circulating Water System Conduct of Operations Ability to locate ontrol room switches, controls, and indications, and to determine that they correctly reflect the desired plant lineup.

Importance Rating: 4.6 / 4.3 IOCFRPart55: 41.10/45.12 IOCFR55.43.b: Not applicable KIA Match: K/A is matched because the question requires the ability to determine the desired plant (CCW) alignment for the conditions stated in the question (removal of a set of water boxes).

Technical

Reference:

SOI-27.01, Condenser Circulating Water System, Revision 0047 Proposed references None to be provided:

Learning Objective: 3-OT-5Y50027A 12 State and explain the different modes of cooling tower operation.

19. Explain how to drain the water boxes and what precautions that need to be adhered to during drain down.

Cognitive Level:

Higher X Lower Question Source:

New X Modified Bank Bank Question History: New question for the WBN 10/2011 NRC exam Comments:

Page 171

. Page4of63 Table of Contents (continued) 8.4.2 Desilting Unit I Cooling Tower Basin 41 8.4.3 Unit I Cooling Tower Basin Refill 42 8.5 Unit 2 Cooling Tower Basin Drain, Desilt, and Return to Service 43 8.5.1 Unit 2 Cooling Tower Basin Drain 43 8.5.2 Desilting Unit 2 Cooling Tower Basin 45 8.5.3 Unit 2 Cooling Tower Basin Refill 46 8.6 Water box Removal from Service 47 8.6.1 Removing Condenser Water boxes from Service 48 8.6.2 Returning Condenser Water boxes to Service 52 9.0 RECORDS 58 9.1 QARecords 58 9.2 Non-QA Records 58 Appendix A: Deicing (Central Shutoff) Valve Operation 59 Appendix B: MWe vs. CCW Temperature & Condenser Backpressure 60 Appendix C: CCW Valve Placard 62 Source Notes 63 EXTERNAL ATTACHMENTS Attachment I P: Power Checklist 27.0 1-1 P Attachment 2P: Trash Rack Hoist Power Checklist 27.01-2P Attachment IV: Valve Checklist 27.0 I-IV Attachment IH: Handswitch Checklist 27.0I-IH

. Page7of63 2.2 Developmental References (continued)

H. WB-DC-40-37, Heat Rejection System I. WBN PEPSE Version 6.0, Thermal Performance Model J. SOl-27.03, Supplemental Condenser Circulating Water System K. N3-1 -4002, System Description for Main Steam L. EDC 53260 3.0 PRECAUTIONS AND LIMITATIONS A. Serious inlet tunnels damage/failure can occur if both sets of condenser water boxes are isolated with a CCW Pump (CCWP) running.

B. Condenser Inlet & Outlet valves should NOT be opened or fully closed with four, (4) CCWPs operating. One Condenser flow path may be isolated with two, (2) -pump operation.

C. When operating with less than 4 CCWPs, pump discharge valves may be throttled to maintain pump amps less than 256 (rated). 0 D. Do NOT operate pumps at less than 35 psid when 2 CCWPs are operating through both sides of the condenser and cooling tower bypass.

E. Pump discharge valve closing to less than 50% will trip pump.

F. When a CCWP is SHUT DOWN, the motor heaters should be energized to keep motor windings dry.

G. When river flow is less than 3500 cfs, Cooling Tower Blowdown (CTBD) diverts to Holding Pond to avoid violating thermal or chemical discharge limits.

H. Only batch releases from CT Desilting Basin are authorized by NPDES permit; special regulator approval is required before CTBD can be routed in a flow-thru mode by opening Desilting Basin Sluice Gate and CT Sluice Gate(s).

I. During cold weather (below 35°F), the Cooling Towers should be monitored for icing. Before turbine shutdown or after a turbine trip, an evaluation should be made as to the potential for icing.

J. CT blowdown flow rate shall be maintained below 44,000 gpm with CT blowdown in service with 0-FCV-27-100 and 0-FCV-27-101 closed.[c.2]

K. CT Bypass Operation

WBN CQnthnser Circulating Water System SQl-2701 Unit I Rev. 0047 Page 8 of 63 3.0 PRECAUTIONS AND LIMITATIONS (continued)

1. Only 2 CCWPs may be used during BYPASS MODE. Flow from more than 2 pumps forces water into the CT fill which results in icing during cold weather, and produces excessive vibration in bypass piping. Flow from one pump allows pump run out too far on its flow/head curve, which results in pump cavitation.
2. The condenser outlet valves may need to be throttled to maintain the CCWPs within their flow/head curve. (TI-4 has pump curve)

Unless required for special tests, CCWPs are NOT operated one at a time except during system startup and shutdown. One pump operation should be held to 2 minutes. If one pump operation is required for an extended period, then the TOTAL HEAD ACROSS THE CCWPs MUST BE MAINTAINED greater than 25.1 psid. During this condition CCWP must be monitored locally due to pressure gauges being local.

L. Discharge of Radioactive Waste to the CT blowdown shall be discontinued when either blowdown flow rate is insufficient for proper dilution or when blowdown is diverted to the holding pond.

M. Instrument Maintenance Department should be notified to ensure required instrumentation will be placed in service as necessary to support system operation.

N. CCWP suction and discharge valves should NOT be hand cranked past electrical stops.

0. If the condenser (raw water side) is NOT in operation for more than a few days, the raw water side of the condenser should be drained.

P. Deicing valves 1-FCV-27-140 and 1-FCV-27-141 should NOT be fully stroked (closed to open, open to closed) more than one time during a 15 minute period.

A second full stroke could cause the valve to exceed its 15 minute duty cycle and thermal out the breaker. One full stroke requires 12 minutes.

Q. The cooling Tower level gages upstream and downstream of 1-RACK-27-3 read in tenths of a foot. These level gages were installed to determine the level drop across 1 -RACK-27-3 and use a different reference mark than the placard that is installed for determining cooling tower level. The trash rack level gages are not being used to report cooling tower level.

R. Trash rack 1-RACK-27-3 shall not be allowed to reach a level differential of greater than 1 foot. This is based on the structural strength of the trash rack components. To prevent reaching 1 foot level differential, the trash rack should be cleaned when the level differential reaches approximately 1/2 a foot.

WBN Condenser Circulating Water System SOI-27.O1 Unit I Rev. 0047 Page 48 of 63 Date INITIALS 8.6.1 Removing Condenser Water boxes from Service CAUTIONS

1) When CCW Pump(s) are RUNNING, only one Water box may be removed from service.
2) Maximum of two CCW Pumps may be RUNNING when either Water box is isolated.
3) This section may be performed while the unit is online. During the time that only one waterbox is in service, unusually high condenser backpressures may occur. Operation in this condition should be below the line of the Backpressure vs. Load limitation Curve below. While load below 35% is recommended, the actual power during performance is at the Shift Managers discretion. faC7 (2.

NOTE

1) If draining one water box, NIA steps NOT used.
2) Automatic level control may not respond as quickly as desired when removing a waterbox from service. Manual level control may be used if required.

[1] IF desired, THEN PLACE Hotwell level control, 1-LIC-2-3, DUMPBACK TO CST FROM HW PMP DISCHARGE andlor 1-LIC-2-9, AUTO MAKEUP FROM CST TO HW in MANUAL [1-M-2] (N/A this step if not performed)

[2] ENSURE Steam Dumps are NOT in service to the Main Condenser.

  • 1-HS-1-103A, STEAM DUMP CONTROL OFF-ON BYPASS A [1-M-4] to OFF
  • 1-HS-1-103B, STEAM DUMP CONTROL OFF-ON BYPASS B [1-M-4] to OFF

WBN Condenser Circulating Water System SOI-27.O1 Unit I Rev. 0047 Page 49 of 63 Date INITIALS 8.6.1 Removing Condenser Water boxes from Service (continued)

[3] ISOLATE affected Condenser Vacuum Pump inlet (NIAvalve NOT used):

WATER PERF NOMENCLATURE LOCATION POSITION BOX UNID INITIAL MAIN CONDENSER T3H/720 CLOSED EAST 1-ISV-2-706 VACUUM LINE ISOL MAIN CONDENSER T3H/720 CLOSED WEST 1-ISV-2-716 VACUUM LINE ISOL

[4] ENSURE CCWP(s) stopped as required and PERFORM the following (NIA handswitches NOT repositioned):

SWITCH PERF NOMENCLATURE LOCATION POSITION UNID INITIAL CCWP 1A& 1-M-15 CLOSE VLV 1-HS-27-9A DISCH FCV-27-11 CCWP lB & 1-M-l5 CLOSE VLV 1-HS-27-19A DISCH FCV-27-21 CCWP 1C & 1-M-15 CLOSE VLV 1-HS-27-29A DISCH FCV-27-31 CCWP 1D& 1-M-15 CLOSE VLV l-HS-27-39A DISCH FCV-27-41 CAUTION Condenser Vacuum will be affected due to reduced cooling capability.

[5] MONITOR Condenser Vacuum and associated Turbine Operating Parameters, AND CLOSE desired Water box Outlet Valve (NIA valve NOT used):

PERF NOMENCLATURE LOCATION POSITION UNID INITIAL EAST COND CCW l-M-15 CLOSED l-FCV-27-77 OUTLET WEST COND CCW l-M-15 CLOSED l-FCV-27-87 OUTLET

WBN Condenser Circulating Water System SOI-27.O1 Unit I Rev. 0047 Page 50 of 63 Date INITIALS 8.6.1 Removing Condenser Water boxes from Service (continued)

[6] CLOSE desired Water box Inlet Valve (N/A valve NOT used):

PERF NOMENCLATURE LOCATION POSITION UNID INITIAL EAST COND CCW 1-M-15 CLOSED 1-FCV-27-55 INLET WEST COND CCW 1-M-15 CLOSED 1-FCV-27-65 INLET NOTE The following step may be performed at any time during the rest of this section.

[7] WHEN Condenser Vacuum and Hotwell levels have stabilized, THEN ENSURE Hotwell level control, 1-LIC-2-3, DUMPBACK TO CST FROM HW PMP DISCHARGE and 1-LIC-2-9, AUTO MAKEUP FROM CST TO HW are in AUTO [1-M-2].

[8] OPEN desired Water box Vent Valve (NIA valves NOT used):

PERF NOMENCLATURE LOCATION POSITION UNID INITIAL CCW EAST SIDE T5H/675 OPEN 1-VTV-27-515 CONDENSER INLET WATER BOX VENT CCW WEST SIDE T4H/675 OPEN 1-VTV-27-516 CONDENSER INLET WATER BOX VENT CCW EAST SIDE T5E/675 OPEN 1-VTV-27-517 CONDENSER OUTLET WATER BOX VENT CCW WEST SIDE T4E1675 OPEN 1-VTV-27-518 CONDENSER OUTLET WATER BOX VENT

WBN Condenser Circulating Water System SOI-27.O1 Unit I Rev. 0047 Page 51 of 63 Date INITIALS 8.6.1 Removing Condenser Water boxes from Service (continued)

CAUTION Extreme care should be used when draining water boxes to avoid flooding foundation sump area.

NOTE Valves in 8.6.1[9] and 8.6.1[10] may be opened more than one turn if required to accommodate inlet or outlet valve seat leakage, provided that drain sump capacity is NOT exceeded.

[9] PERFORM the following (NIA valves NOT used):

PERF NOMENCLATURE LOCATION POSITION WATER BOX UNID INITIAL CCW INLET WATER T7H/685 THROTTLE as EAST 1-DRV-27-591 BOX DRAIN desired COW INLET WATER T7H/685 THROTTLE as WEST 1-DRV-27-592 BOX DRAIN desired CCW OUTLET T5C1685 THROTTLE as EAST 1-DRV-27-593 WATER BOX DRAIN desired CCW OUTLET T5C/685 THROTTLE as WEST 1-DRV-27-594 WATER BOX DRAIN desired

[10] WHEN water boxes are drained, THEN PERFORM the following (NIA valves NOT used):

PERF NOMENCLATURE LOCATION POSITION WATER BOX UNID INITIAL CCW INLET T4H/675 1 turn OPEN EAST 1-DRV-27-595 WATER BOX DRAIN CCW INLET T5H1675 1 turn OPEN WEST 1-DRV-27-596 WATER BOX DRAIN CCW OUTLET T4E1675 1 turn OPEN WEST 1-DRV-27-597 WATER BOX DRAIN CCW OUTLET T5D1675 1 turn OPEN EAST 1-DRV-27-598 WATER BOX DRAIN End of Section

3-OT-SYSO27A Revision 7 Page5of4O I. PROGRAM Watts Bar Operator Training II. COURSES License Training NOTP License Requalification NAUO Requalification III. TITLE Condenser Circulating Water System IV. LENGTH OF LESSON A. License Training 2 Hours B. NOTP 4 Hours License Requalification and NAUC Requalification times will be determined after objectives are identified.

V. TRAINING OBJECTIVES ARSS UORT 0 CA X X X X 1. State the design basis/purpose of the Condenser Circulating Water (CCW) system in accordance with FSAR section 10.4.5.

X X X X 2. Draw the CCW and SCOW systems, including flowpath and major components.

3. Deleted.
4. Deleted.

X X X X 5. Describe the major components of the COW system including:

a. Cooling towers
b. COW pumps
c. Main condenser

3-OT-SYSO27A Revision 7 Pageof40 ARSS UORT 0 OA X X X X 6. Draw a cross-sectional view of the cooling tower and label each section.

Include in the drawing:

a. Chimney
b. Basin
c. Risers
d. Distribution pipes e Fill
f. Drift eliminator waves X X X X 7. Describe the CCW pumps; include logic, interlocks, power supply, type, capacity and support systems.

X X X X 8. Explain how to start a CCW pump.

X X X X 9. Identify the type of seal employed for the CCW pumps.

10. Deleted.

X X X X II. Describe the main condenser.

X X X X 12. State and explain the different modes of cooling tower operation.

X X X X 13. Explain how a cooling tower works.

X X X X 14. Explain how water is lost from the COW system during normal operation and list the sources of makeup.

X X X X 15. State what happens on low and high level in the cooling tower flume.

X X X X 16. Explain the minimum cooling tower blowdown flow rate interlock with Radwaste, S/G Blowdown, and Cond Demin discharge valves.

X X X X 17. Explain what automatic functions occur in the cooling tower blowdown when river flow drops below 3500cfs.

X X X X 18. Explain how to fill and vent the COW system and explain why venting is important.

X X X X 19. Explain how to drain the water boxes and what precautions that need to be adhered to during drain down.

X X X X 20. Identify four indications of plant flooding.(SOER 85-05) [commitment]

X X X X 21. Describe the SCOW subsystem including the differences between summer and winter operation.

L

3-OT-SYSO27A Revision 7 Page7of4o ARSS UORT O CA X X X X 22. Describe the location of major system components including the following:

a. CCW pumps
b. Condenser inlet and outlet valves
c. Local controls for pumps and FCVs
d. Cooling Tower bypass valves
e. Cooling Tower deicing valves
f. Deleted
g. Deleted
h. SCCW System supply, bypass, and discharge valves X X X 23. Correctly locate control room controls and indications associated with the CCW system, including:
a. CCW pump control switches.
b. Condenser inlet and outlet temperature indicators.
c. Condenser inlet and outlet FCV handswitches.

WBN 10-2011 NRC RO Exam As Submitted 8/1512011

65. 086 K1.02 065 Which ONE of the following identifies why O-SW-026-0320, RAW SERVICE WATER ISOLATION VALVE O-FCV-26-320, located in the Makeup Water Treatment Plant Control Room would be placed to OFF?

A. Prevent exceeding maximum flow rating of the water treatment system following the start of any fire pump.

B. Prevent overpressurizing the water treatment system following the start of any fire pump.

C. Prevent the automatic closure of O-FCV-26-320 due to a diesel fire pump start.

D Prevent automatic closure of O-FCV-26-320 when an electric fire pump start signal is generated.

DISTRA CTOR ANALYSIS:

A. Incorrect, Plausible because due to the pressure rise on the system when a fire pump is started there would be an increase in flow through the system.

B. Incorrect, Plausible because the pressure on the system rises rapidly when a fire pump is started and this would result in an increase in the pressure inside the demin beds and piping.

C. Incorrect, Plausible because the actual reason is to prevent the isolation of the RSW to the Makeup water Treatment building during a start of the fire pumps but it is the electric fire pumps starting not the diesel fire pump starting that causes the isolation.

D. Correct, As identified in SOI-26.O1, High Pressure Fire Protection System, the switch is placed to OFF to prevent the isolation of the RSW to the Makeup Water Treatment Building during a start of the electric fire pumps.

SOI-26.01 8.3.2 Electric Fire Pump Manual Start

[4] IF RSW/Dl Process water isolation to Makeup Water Treatment Building is NOT desired, THEN PLACE switch 0-SW-026-0320, RAW SERVICE WATER ISOLATION VALVE O-FCV-26-320 located in compartment HO of 0-MCC-281-1, to OFF (MWTP Control Room). (N/A in emergency)

Question Number: 65 Tier: 2 Group 2 Page 172

WBN 10-2011 NRC RO Exam As Submitted 811512011 K/A: 086 K1.02 Fire Protection System Knowledge of the physical connections and/or cause/effect relationships between the Fire Protection System and the following systems:

Raw service water Importance Rating: 2.7* / 3.2*

10 CFR Part 55: 41.2 to 41.9/45.7 to 45.8 IOCFR55A3.b: Not applicable KIA Match: K/A is matched because the question requires knowledge of the physical connections between the Fire Protection System and the Raw Service Water system as well as a cause/effect relationship with the Fire protection system when a fire pump is started and how the effect can be defeated.

Technical

Reference:

SOl-26.01, High Pressure Fire Protection System, Revision 26 1-45W760-26-1 R25 1-45W600-57-3 R16 Proposed references None to be provided:

Learning Objective: 3-OT-SYSO26A

5. Correctly locate and state the normal alignment of control room controls and indications associated with the HPFP system, including:
a. Pump handswitches
b. Valve handswitches Cognitive Level:

Higher Lower X Question Source:

New X Modified Bank Bank Question History: New question for the WBN 10/2011 NRC exam Comments:

Page 173

WBN High Pressure. Fire PrctectiQn System SQL-Z6Qi Unit 0 Rev. 0026 Page 27 of 163 Date 8.2.3 Local Manual CAUTION This method does not start the HPFP Pumps.

[1] OPEN Emergency Release manual valve for Viking Preaction Deluge Valve, or 3-way valve on top of Maxitrol Deluge Valves for area needed.

8.3 Electric Fire Pump Start and Return to Standby NOTES

1) Valves 0-FCV-26-31 45 and 0-FCV-26-31 46 are self actuating pressure control valves that will close as noted in Section 3.0, Precautions and Limitations. Manual reset for the valves and chillers after the valves have closed is required. These valves GO TO the Office Bldg chiller and selected Service Bldg chillers.
2) Valve 0-FCV-26-320 will close on a signal from an electric fire pump start circuit. This valve goes to Makeup Water Treatment Plant process water supply.

8.3.1 Electric Fire Pump Automatic Start

[1] IDENTIFY fire pump START signal.

[2] WHEN determined that fire pump is not needed, THEN PERFORM Section 83.3.

WBN High Pressure Fire Protectiçn System SOi-26.O1 Unit 0 Rev. 0026 Page 28 of 163 Date 8.3.2 Electric Fire Pump Manual Start NOTES

1) The electric fire pumps in A-P AUTO should be started when I or 2 electric fire pumps are to be started for a non-emergency reason. This will lessen the possibility of 3 or 4 electric fire pumps running should a start signal be received from the FPS circuit after the pump(s) has been manually started.
2) When an electric fire pump in A-P AUTO is started then one of the following windows on 1-M-15 will be initiated:

Window 171B: TRA HPFP PMP CNTL SWITCH MISALIGNED Window 172B: TR B HPFP PMP CNTL SWITCH MISALIGNED

3) When an electric fire pump is started then the following window on 1-M-15 will be initiated:

Window 172A: HPFP PMP RUNNING

[1] ENSURE Prerequisite Actions, COMPLETE (N/A in emergency).

[2] ENSURE an NPG-SPP 18.4.6, FPIP has been secured to address the defeat of the O-FCV-26-320, -3145, and -3146 (N/A in emergency).

[3] ENSURE the following hand switches are in the TEST position (N/A in emergency):

A. O-HS-26-3145A [MOB Chiller Room, north wall, el. 725, O-JB-294-6889]

B. O-HS-26-3146A [Service Bldg., Mechanical Equip. Rm.,

S4/SM, el. 741, O-JB-295-6890]

[4] IF RSW/DI Process water isolation to Makeup Water Treatment Building is not desired, THEN PLACE switch O-SW-026-0320, RAW SERVICE WATER ISOLATION VALVE O-FCV-26-320 located in compartment HC of O-MCC-281 -1, to OFF (MWTP Control Room). (N/A in emergency)

CV

WBN High Pressure Fire Protection System - SQI-26.01 Unit 0 Rev. 0026 Page 32 of 163 Date 8.3.3 Electric Fire Pump Return to Standby (continued)

[2] IF the pump(s) was started for an emergency reason and the as-found handswitch positions were not recorded in Section 8.3.2[5], THEN PERFORM the following:

A. ENSURE Fire is OUT if applicable, and Area COOLED below flash point.

B. ENSURE fire pump START signal RESET (NA if manually started).

C. STOP fire pump (s).

D. PERFORM Section 5.1 to Place HPFP System in Standby, THEN RETURN to Section 8.3.3[3]

[3] IF SWITCH O-SW-026-0320 HC was placed in the OFF position in Section 8.3.2[4], THEN PLACE SWITCH O-SW-026-0320 RAW SERVICE WATER ISOLATION VALVE O-FCV-26-320, located in compartment HC of O-MCC-281-1, to ON. (MWTP Control Room)

CV IV

[4] IF switch O-SW-026-0320 for O-FCV-26-320 was not placed in OFF in Section 8.3.2[4], THEN RESETIOPEN O-FCV-26-320 locally at O-HS-26-320, MAKEUP WTR TRTMT PLANT HPFP ISOL VLV

[Southeast corner of Water Treatment Plant].

WBN High Pressure Fire Protection System SOl-26.01 Unit 0 Rev. 0026 Page 65 of 163 Checklist I (Page 8 of 8)

Checklist I Date PERF VERIF LOAD LOCATION POSITION TAG NO. I UNID INITIAL INITIAL O-FCV-26-3146 HPFP TO SERVICE BLDG ACU RAW SERVICE PRESS CNTL RADIOCHEMLABAC LC21,SERV ON BKR6 ELECTRONIC CONTROL BLDG, SKS5, SYSTEM PLS3889

- EL. 741 (Mens Restroom, WC Area) CV IV TEST SWITCH FOR SERV BLDG, NORMAL O-HS-26-3146A O-FCV-26-31 46 54/5K, EL 741 IV MANUAL CONTROL SERV BLDG, AUTO O-HS-26-3146B SWITCH FOR S4/SK, O-FCV-26-3 146 EL 741 IV O-FCV-26-320 HPFP TO WATER TREATMENT PLANT WATER TREATMENT MWTP Control ON BKR HC PLANT Room, O-SW-026-0320 CV IV

3-OT-SYS26A Revision 5 Page4of7t I. PROGRAM Watts Bar Operator Training II. COURSES A. License Training B. Non-License Training III. TITLE High Pressure Fire Protection Iv. LENGTH OF LESSON A. Licensed Training 2 hours2.314815e-5 days <br />5.555556e-4 hours <br />3.306878e-6 weeks <br />7.61e-7 months <br /> B. Non-Licensed Training 4 hours4.62963e-5 days <br />0.00111 hours <br />6.613757e-6 weeks <br />1.522e-6 months <br /> V TRAINING OBJECTIVES ARSS UORT 0 CA x x x x 1. State the design basis of the HPFP system in accordance with System Description, N3-26-4002.

x x x x 2. State the function of the HPFP system in accordance with the system description.

x x x x 3. Sketch a basic drawing of the HPFP equipment located at the intake structure; include pumps, strainers, pressure control valve, intake pit and mud valve.

x x x x 4. Describe the HPFP pumps; include capacity, power supplies, type and logic.

x x x 5. Correctly locate and state the normal alignment of control room controls and indications associated with the HPFP system, including:

a. Pump handswitches
b. Valve handswitches x x x x 6. State the places the fire pumps can be started and stopped.

x x x 7. Upon HPFP actuation, state the sequence in which the fire pumps start.

x x x x 8. Describe the local checks to be made on the HPFP when in standby and when running.

x x x x 9. State when the valves O-FCV 26-3 and O-FCV 26-8, mud valves would be opened.

3-OT-SYS26A Revision 5 Page-S of 71 A R S S U 0 R T 0 0 A x x x x 10. Discuss the purpose of 0-PCV 26-18, Back pressure Control Valve.

x x x x 11. State how the HPFP system remains charged / pressurized while the HPFP pumps are off.

12. Objective Deleted.

x x x x 13. List all the systems that use the Maxitrol Sentry valve.

x x x x 14. Describe the basic construction and operation of a Maxitrol Sentry deluge valve.

x x x x 15. List the different type sprinkler heads used in the HPFP system.

x x x x 16. Describe the basic construction and operation of a Viking deluge valve.

x x x x 17. Explain the difference between a wet header and dry header system.

x x x x 18. Explain how some dry headers are monitored to ensure the heads are intact.

x x x x 19. Given a loss of instrument air/control power and fire header pressure, determine the effect on the following valves.

a. Maxitrol Sentry
b. Viking x x x x 20. Explain how the HPFP foam units operate.

x x x x 21. Describe how the 5th DG and the Backup Security DG buildings are protected during fire.

x x x x 22. Briefly explain how to reset the HPFP system after actuation, include resetting of the deluge valves and the foam tanks.

x x x x 23. State the different ways to initiate all the HPFP systems automatically and manually.

x x x x 24. List all uses of HPFP in the Reactor Buiding.

x x x x 25. Briefly describe the purpose of the periodic HPFP flushes and biocide injection.

x x x 26. Given the condition/status of the HPFP system/component and the appropriate sections of Fire Protection Plan, determine if operability requirements are met and what actions, if any, are required.

x x x x 27. State the reason for not initiating deluge systems on energized equipment except in a case of emergency. (This objective is a commitment and shall not be altered or deleted without the approval of the OTM.)

x x x x 28. Describe the installation and construction of the Diesel Driven Fire Pump.

3-OT-SYS26A Revision 5 Page6of7i A R S S U 0 R T 0 0 A x x x x 29. Describe the control functions for the Diesel Driven Fire Pump; specifically how the pump is started and stopped (manually and automatically) include interlocks.

VI. TRAINING AIDS A. Whiteboard and markers.

B. Multimedia projector.

C. Overhead projector VII. MATERIALS One copy of each of the following for each participant as required:

Append ices None B. Attachments None C. WBN Prints (Latest Revision)

1) 47W845-1
2) 47W845-2
3) 47W845-3
4) 47W845-4
5) 47W845-5 VIII. REFERENCES NUMBER TITLE I BEN RCA 89-101 (BEN LER 89-028-01) Fire Deluge System Contributes to Electrical Failure 2 (Internal) Add statement which cautions against initiating fire suppression water deluge systems on energized equipment.

Text in this lesson plan which is annotated as commitments shall not be deleted without approval by the Operations Training Manager.

WBN 10-2011 NRC RO Exam As Submitted 811512011

66. G2.1.14 066 Consider the two following events:

- Unit I reactor is taken critical in accordance with GO-2, Reactor Startup.

- Operators are responding to a reactor trip in accordance with ES-O.1, Reactor Trip Response.

Which ONE of the following identifies if the respective procedure requires Public Address (PA) announcement be made for the event?

Reactor Critical Reactor Trip A. No Yes B. No No Cv Yes Yes D. Yes No DISTRA CTOR ANAL YSIS:

A. Incorrect, Plausible because the reactor trip PA announcement being required is correct and the applicant may be aware that the procedure for reactor startup requires an announcement when rod withdrawal begins and not relate that another announcement is also required when the reactor reaches critical.

B. Incorrect, Plausible because the applicant may be aware that the procedure for reactor startup requires an announcement when rod withdrawal begins and not relate that another announcement is also required when the reactor reaches critical. Additionally, the announcement E-O is after the transition to ES-O. I which could lead applicant to think that an announcement is required only if a safety injection was also initiated.

C. Correct, Both of the events require a PA announcement as directed by a step in the respective procedure or each event.

D. Incorrect, Plausible because an announcement being required when the reactor reaches a critical condition is correct and the announcement E-O is after the transition to ES-O. I which this could lead applicant to think that an announcement is required only if a safety injection was also initiated.

Page 174

WBN 10-2011 NRC RO Exam As Submitted 811512011 Question Number: 66 Tier: 3 Group: n/a KIA: G2.1.14 Conduct of Operations Knowledge of criteria or conditions that require plant-wide announcements, such as pump starts, reactor trips, mode changes, etc.

Importance Rating: 3.1 / 3.1 IOCFRPart55: 41.10/43.5/45.12 IOCFR55.43.b: Not applicable K/A Match: K/A is matched because the question requires knowledge of plant conditions requiring an announcement to the plant.

Technical

Reference:

GO-2, Reactor Startup, Revision 0039 ES-0.1, Reactor Trip Response, Revision 0023 Proposed references None to be provided:

Learning Objective: 3-OT-SPP1 000

5. Describe Unit Operator responsibilities.
20. Describe the responsibilities of operations personnel as it pertains to Back To Basics fundamentals, including: h. Communications Cognitive Level:

Higher Lower X Question Source:

New X Modified Bank Bank Question History: New question for the WBN 10/2011 NRC exam Comments:

Page 175

WBN - Reactor Startup GO-2 Unit I Rev. 0039 Page 30 of 43 Date_________ Initials_____

5.3 Reactor Startup (continued)

NOTES

1) Blocking SRM Reactor Trip disables detectors outputs and removes audio count rate signal.
2) Steps 5.3[21 .5] through 5.3[21 .7] should, at SM discretion, be performed at the time of criticality. However, SR Trip must be BLOCKED conservatively before the SR Trip setpoint of cps.

Start of Critical Step(s)

[21.5] BLOCK SR Trip by placing handswitches 1-N33A, SR TRIP TR A RESET-BLOCK P-6, and 1-N33B, SR TRIP TR B RESET-BLOCK P-6, to BLOCK.

End of Critical Step(s)

[21.6] CHECK the following:

[21.6.1] Permissive 64 C, SOURCE RANGE TRIP BLOCKED, LIT.

[21.6.2] Alarm 81 C, SOURCE RANGE HI FLUX AT SHUTDOWN ALM BLOCKED, NOT LIT.

[21.7] SELECT 1-NR-92-145 to record the highest indicating IR channel and one PR channel.

[22] ANNOUNCE Reactor Criticality on the P/A.

6. ENSURE all control and shutdown IF two or more rods are NOT fully rods fully inserted: inserted, THEN RPIs at bottom scale.

INITIATE boration of 3250 gals of greater than or equal to 6120 ppm boron for each rod not fully inserted:

  • REFER TO AOI-34, Immediate Boration.
7. ANNOUNCE reactor trip over PA system.
8. MONITOR S/G levels:
a. At least one S/G NR level greater a. ENSURE feed flow greater than than 29%. 410 gpm.
b. S/G NR levels less than 50% and b. IF any SIG NR level continues to controlled. rise, THEN ISOLATE feed flow to affected S/G.
9. CONTROL S/G NR levels between 29% and 50%.
10. INITIATE BOP realignment:

Page 7 of 19

3 -OT-SPP 1000 Revision 11 Page 4 of 45 I. PROGRAM WATTS BAR OPERATOR TRAINING II. COURSE A. LICENSED OPERATOR TRAINING B. NON-LICENSED OPERATOR TRAINING III. TITLE OPDP-1, CONDUCT OF OPERATIONS and SPP-10.0 and OPDP-10 IV. LENGTH OF LESSON A. LICENSED OPERATOR TRAINING 2 HOURS B. NON-LICENSED OPERATOR TRAINING 4 HOURS V. TRAINING OBJECTIVES A R S S U 0 R T 0 0 A X X X X 1. Explain the purpose of SPP-10.0, Plant Operations, and OPDP- 1, Conduct of Operations.

X X X X 2. State who may hold the control room command function.

X X X 3. Describe Unit Supervisor responsibilities.

X X X 4. Describe STA responsibilities.

X X X X 5. Describe Unit Operator responsibilities.

X X X X 6. Describe shift staffing requirements.

X X X X 7. Explain the requirements for Operator rounds performance.

X X X 8. Describe the actions to be taken when an instrument failure is suspected.

3 -OT-SPP 1000 Revision 11 Page5of45 A R S S U 0 R T 0 0 A X X X 9. Explain the differences between the expected response to alarms during steady state conditions and transients.

X X X 10. [Describe the conditions when a licensee may take reasonable action to depart from a license condition or a Technical Specification, as noted in OPDP-1 Section 5.1.H.

(SOER-87-001, Rec. 6)]

X X X X 11. Explain the importance of proper logkeeping.

X X X X 12. Describe the information to be recorded in Operating logs.

X X X X 13. Describe the WCC SRO responsibilities.

X X X X 14. State the types of briefings and associated standards as described in OPDP-1.

X X X 15. Describe the circumstances and exceptions which allow a Unit Operator to take prudent action without waiting for SRO concurrence, as described in OPDP-1.

X X X 16. Describe the information required for a temporary relief.

X X X X 17.DELETED.

X X X X 18. Define Operator Workarounds, Burdens, Challenges.

X X X 19. Describe who may manipulate controls directly affecting reactor reactivity or power level.

3 -OT-SPP 1000 Revision 11 Page 6 of 45 AR S S U OR T 0 0 A X X X X 20. Describe the responsibilities of operations personnel as it pertains to Back To Basics fundamentals, including:

a. Logkeeping
b. Operator Rounds
c. Procedure Use and Adherence
d. Control of Equipment
e. Verification
f. Reactivity Management
g. Tagging
h. Communications
i. Control Board Awareness
j. Technical Specification Compliance
k. Human Performance
1. Cultural Foundations X X X 21. Describe the responsibilities and requirements for reactivation of an inactive license. (OPDP-10)

X X X 22. Describe the conditions and expectations for the establishment of parameter thresholds for initiating operator actions to place the plant in a safe condition, as described in OPDP-1.

VI. TRAINING AIDS A. Marker Board B Assorted Markers C. Overhead Projector

WBN 10-2011 NRC RO Exam As Submitted 811512011

67. G2.1.19 067 Using the attached ICS AFD TARGET DISPLAY screen printout, which ONE of the following choices completes the following statements?

The AFD HIGH LIMIT is (1)

If (2) outside of the limit, TS LCO 3.2.3, Axial Flux Difference, action statement will be entered.

REFERENCE PROVIDED Li)

A. -5 any NIS channel is B. -5 at least 2 NIS channels are C. +7 any NIS channel is Dv +7 at least 2 NIS channels are DISTRA CTOR ANAL YSIS:

A. Incorrect, Plausible if candidate determines that -5 (high end of the normal control band) is the high limit. Also, it takes 2 channels outside the band to require entry in the Tech Spec LCO.

B. Incorrect, Plausible if candidate determines that -5 (high end of the normal control band) is the high limit. Also, the second part of the response is correct.

C. Incorrect, Plausible since the first part of the distractor is correct however it takes two channels outside the limit for entry into the Tech Spec LCO.

D. Correct, The AFD high limit as demonstrated on the supplied AFD Target Display and COLR indicates that 7 is the high limit. Per T. S. 3.2.3, if two or more NI channels are indicating outside the limit then entry into the Tech Spec LCO is required.

Page 176

WBN 10-2011 NRC RO Exam As Submitted 8/1512011 Question Number: 67 Tier: 3 Group n/a K/A: G2.1.19 Conduct of Operations Ability to use plant computers to evaluate system or component status.

Importance Rating: 3.9 I 3.8 10 CFR Part 55: 41.10 I 45.12 IOCFR55.43.b: Not applicable KIA Match: Applicant is required to review an ICS (plant computer) screen to determine the AFD limits and how the points provided by the NIS channels would be used to identify Tech Spec entry.

Technical

Reference:

T.S. 3.2.3, Axial Flux Difference (AFD)

COLR Unit 1 Cycle 11, Figure 3 Axial Flux Difference Acceptable Operation Limits as a Function of Rated Thermal Power (RAOC), Revision 0 Proposed references ICS AFD TARGET DISPLAY with the high and low to be provided: values & limits removed from display Learning Objective: 3-OT-T1S0302

3. Given plant conditions, determine if the unit is in compliance with applicable LCOs.

Cognitive Level:

Higher Lower X Question Source:

New X Modified Bank Bank Question History: SQN bank question G 2.1.19 used on the 02/2010 exam. Values changed for WBN.

Comments:

Page 177

AFD 3.2.3 3.2 POWER DISTRIBUTION LIMITS 3.2.3 AXIAL FLUX DIFFERENCE (AFD)

LCO 3.2.3 The AFD in % flux difference units shall be maintained within the limits specified in the COLR.

NOTE The AFD shall be considered outside limits when two or more OPERABLE excore channels indicate AFD to be outside limits.

APPLICABILITY: MODE 1 with THERMAL POWER > 50% RTP.

ACTIONS CONDITION REQUIRED ACTION COMPLETION TIME A. AED not within A.l Reduce THERMAL POWER to 30 minutes limits. < 50% RTP.

SURVEILLANCE REQUIREMENTS SURVEILLANCE FREQUENCY SR 3.2.3.1 Verify AFD within limits for each 7 days OPERABLE excore channel.

AND Once within 1 hour1.157407e-5 days <br />2.777778e-4 hours <br />1.653439e-6 weeks <br />3.805e-7 months <br /> and every 1 hour1.157407e-5 days <br />2.777778e-4 hours <br />1.653439e-6 weeks <br />3.805e-7 months <br /> thereafter with the AED monitor alarm inoperable Watts Bar-Unit 1 3.29

120 110 100 90 80 70 C

60 2

I a)

.rz

-t:3 50 a)

Cu 9-o 40 30 20 10 0

-30 -20 -10 0 10 20 30 40 50

-50 -40 Flux Difference (M) %

Figure 3 Power (RAOC)

Axial Flux Difference Acceptable Operation Limits as a function of Rated Thermal Cycle 11 Unit I Page 8 oflO Revision 0 WATTS BAR

Main larms Graphics Trends Points Zoom/Layers Print Help L ce&y) PR I iV -

06-JUL 2011 07 : 46: 16 SELECT RIlIC. KIEV OR 11JRII-OII CODE DOGHOUSE SCHP I a 110 P 100 \

POWER LEVEL 99*4

\ \

\ \, GIRL BANK [) (STEPS) 220.0 o \ \

90- \ \

W = \ \ MIS CHANNEL 41 -7.6

\ \ AFD 80- 42 -7,4 E \ \ AED NIS CHANNEL AFD MIS CHANNEL 43 -7,7 R 70 AFD NIS CHANNEL 44 -7*4 60 NIS ACTUAL AFD -7.5 ,

L =

50 NI S TARGET AFD -7.5 ;

40 AFD LOW LIMIT -12.2 9 o 7 ) v E, AFD HIGH LIMIT I L .j L

0/ 10 CONTROL BAND LOW L TM -10.0 c* n CONTROL BAND HIGH [IN fl II I I II I I J

- :35 -30 -25 -20 -15 -10 -5 5 10 15 20 AXIAL FLUX DIFFERENCE  % TIME OUT OF BAND ACCUM UIIN 0.0 j

F 1JiJ 1T059 i1K=AIIOIIYMOIJS SEC LVL= 15 PRIM! FACK CPU I MODE 1

Main Jarms Graphics Trends Points Zoom/Layers Print Help 06-JUL-2011 07:46:16 SELEcT RIlIC. KEY OR 11JRM-0H CODE DO6HOUSE> a POWER LEVEL 994 GIRL BANI( D (STEPS) 220.0 AFD MIS CHANNEL 41 -7+6 AF[) MIS CHANNEL 42 7+4)

AF[) MIS CHANNEL 43 7.7 AFD HIS CHANNEL 44 -7+4 NIS ACTUAL AFD -7.5 %

N IS TARGET AF[) -7.5  ;

0 AFD LOW [BITT AF[) HIGH LIMIT 1)

CONTROL BAND LOW LI H 0

CONTROL BAND HIGH [IN TIHE OUT OF BANE) AGGIJH (HIM)

I.

ILIiE SEC LVL= Th PRIH/IACI( CPU I IIODE 1

Main ,Oarms Graphics Trends Points Zoom/Layers Print Help RE2cc TO B 06-.]qJL-2011 07:46:16 SELECT F1JHC. KEY OR 11JRIJ -ON CODE [}OGHOU SE:>

110 P ioo POWER LEVEL 99+4 GIRL BANK D (STEPS) 220+0 90-w AFD NIS CHANNEL 41 7.6  ;

80 E AFD MIS CHANNEL 42 7*4 AED NIS CHANNEL 43 7.7 R AFD MIS CHANNEL 44 J*4 9 60 NIS ACTUAL AFD 75 %

L N I S TARGET AFD 7.5  ;

50-E 4o-V AFD LOW LIMIT E AFD HIGH LIMIT L

20 0/ 10 CONTROL BAND LOW LIH CONTROL BAND HIGH UN II I II I I I II II I I I I I I III ji I II II I I I I I I

-35 -30 -25 -20 -15 -10 - 5 10 15 20 25 I AXIAL FLUX DIFFERENCE TIME OUT OF BAND ACCUM (111W)

TTOS9 i1K= AIIOIIYIIOUS SEC L/L= 15 PR1Il/BACK CPU I NODE 1

SG QuHo1

1. Using the attached ICS AFD TARGET DISPLAY screen printout, which ONE of the choices completes the following statements?

The AFD HIGH LIMIT is If was/were outside of the limit, TS LCO 3.2.1, Axial Flux Difference, action statement will be entered.

REFERENCE PROVIDED A. 7.2; any NIS channel B 7.2; at least 2 NIS channels C.4; any NIS channel D.4; at least 2 NIS channels

3-OT-T/S0302 Revision 1 Page4oflO I. PROGRAM WATTS BAR OPERATOR TRAINING II. COURSE A. LICENSE PREP B. LICENSE CERTIFICATION C. ONSITE D. LICENSED OPERATOR REQUAL III. TITLE T/S 3.2, POWER DISTRIBUTION LIMITS AND BASES IV. LENGTH OF LESSON A. LICENSE PREP 1 HOUR B. LICENSE CERTIFICATION 1 HOUR C. ONSITE 1 HOUR LICENSED OPERATOR REQUAL TIME WILL BE DETERMINED WHEN OBJECTIVES ARE IDENTIFIED V. TRAiNING OBJECTIVES AR S S U OR T 0 0 A X X X 1. Define each of the following terms:

a. Heat Flux Hot Channel Factor (FQZ)
b. Nuclear Enthalpy Rise Hot Channel Factor (F N)
c. Axial Flux Difference (AFD)
d. Quadrant Power Tilt Ratio (QPTR)

X X 2. Determine the bases for the LCOs in section 3.2, Power Distribution Limits.

X X X 3. Given plant conditions, determine if the unit is in compliance with applicable LCOs.

3 -OT-T/S03 02 Revision 1 Page 5 of 10 X X 4. Given plant conditions, determine what action(s) must be taken and the bases for those actions.

WBN 10-2011 NRC RO Exam As Submitted 811512011

68. G2.1.40 068 Given the following:

- Unit I is in a refueling outage with nozzle dams installed.

- Maintenance is in the process of tensioning the reactor head bolts.

Which ONE of the following identifies the reason the pressurizer manway would be removed whenever the reactor head is in place and the SG U-tubes are drained or pressurizer level is off scale low in accordance with GO-b, Reactor Coolant System Drain and Fill Operations?

This ensures.

A. that a steam void in the RCS piping would not prevent natural circulation flow.

By an adequate vent exists to allow gravity fill from RWST on a station blackout event.

C. that on a loss of level in the RCS rapid refill capability using RHR pumps would be available.

D. an adequate vent exists to prevent a vacuum from being developed in the RCS piping on a rapid loss of RCS level.

Page 178

WBN 10-2011 NRC RO Exam As Submitted 8/1512011 DISTRACTOR ANAL YSIS:

A. Incorrect, Plausible because a steam void could impede or prevent natural circulation, but the conditions in the stem will not allow RCS natural circulation because the steam generators are drained. The conditions in the stem if applicant thinks that natural circulation could be developed for the conditions in the stem cases, however S/Gs could be drained thus prohibiting Nat Circ.

B. Correct, See below from GO-b, Precaution N. Keeping RCS from being pressurized would allow natural circulation following a loss of RHR if all AC power were lost.

C. Incorrect, Plausible a vent would be needed if the RHR pumps were to be used to add inventory to the RCS while the system was intact to prevent a pressure rise but that is not the reason for removing the pressurizer manway.

D. lncorrect, Plausible because a vent would be needed to prevent a vacuum in the RCS if a rapid level loss occurred but that is not the reason the pressurizer manway would be removed.

While in RuM, it is necessary to have proper hot leg venting paths in the event of a loss of RHR cooling. This is necessary to prevent the boil off from the core decay heat pressurizing the RCS above two psig. 2 The requirements are applicable until Cold Leg breaches are closed. (e.g. CL SIG manways closed, system openings, which cannot be isolated, CL injection check valve bonnet replaced, High pressure seal on thimble guides, RCPs coupled up or properly blocked.)

1. Prior to refueling, the vent path shall be either:
a. Reactor vessel head removed.
b. One or more SG Hot Leg manways removed. (Hot Leg nozzle dam NOT installed on applicable SG loop)
2. After refueling, any of the following combinations are adequate vent paths:
a. Reactor vessel head removed.
b. One or more SG Hot Leg manways removed. (Hot Leg nozzle dam NOT installed on applicable SG loop)
c. One Hot Leg SIS check valve bonnet removed and at least two Pressurizer safety valves removed.
d. One Hot Leg SIS check valve bonnet removed and the Pressurizer manway removed.
e. With Reactor Vessel head installed, alu nozzle dams installed, and only one Pressurizer Safety valve removed, two CCPs shall be available with only one aligned for injection to comply with TS 3.4.12. The second CCP can only be aligned for injection if the other becomes unavailable.

Page 179

WBN 10-2011 NRC RO Exam As Submitted 8/15/2011 Question Number: 68 Tier: 3 Group n/a K/A: G2.1.40 Conduct of Operations 2.1.40 Knowledge of refueling administrative requirements.

Importance Rating: 2.8 / 3.9 IOCFRPart55: 41.10 IOCFR55.43.b: Not applicable KIA Match: This question matches the K/A by asking the candidate to identify the operations administrative requirement associated with the pressurizer manway if when the reactor head has been set during refueling.

Technical

Reference:

GO-la, Reactor Coolant System Drain and Fill Operations, Revision 0045 Proposed references None to be provided:

Learning Objective: 3-OT-GO1000

1. Identify the condition(s) necessary for Reduced Inventory/Mid-loop operations to exist.
7. Identify the reason for each prerequisite and precaution as discussed in this lesson plan or provided in GO-b.

Cognitive Level:

Higher X Lower Question Source:

New Modified Bank Bank X Question History: SQN bank question G 2.1.40 068 used on the SQN 02/2010 exam but reformatted, stem modified, and 3 choices relocated to move correct answer.

Comments:

Page 180

WBN Reactor Coolant System GO-b Unit I Drain And Fill Operations Rev. 0045 Page 14 of 358 3.1 Partial Draindown I Reduced Inventory I Mid-Loop Operation Mode 6 (continued)

N. While in RuM, it is necessary to have proper hot leg venting paths in the event of a loss of RHR cooling. This is necessary to prevent the boil off from the core decay heat pressurizing the RCS above two psig. [c.21 The requirements are applicable until Cold Leg breaches are closed. (e.g. CL SIG manways closed, system openings, which cannot be isolated, CL injection check valve bonnet replaced, High-pressure seal on thimbleguides, RCPs coupled up or properly blocked.)

1. Prior to refueling, the vent path shall be either:
a. Reactor vessel head removed.
b. One or more SG Hot Leg manways removed. (Hot Leg nozzle dam NOT installed on applicable SG loop)
2. After refueling, any of the following combinations are adequate vent paths:
a. Reactor vessel head removed.
b. One or more SG Hot Leg manways removed. (Hot Leg nozzle dam NOT installed on applicable SG loop)
c. One Hot Leg SIS check valve bonnet removed and at least two Pressurizer safety valves removed.
d. One Hot Leg SIS check valve bonnet removed and the Pressurizer manway removed.
e. With Reactor Vessel head installed, all nozzle dams installed, and only one Pressurizer Safety valve removed, two CCP5 shall be available with only one aligned for injection to comply with TS 3.4.12.

The second CCP can only be aligned for injection if the other becomes unavailable.

0. The Maintenance supply is an unqualified feed to the shutdown board and can only be used in Mode 5 or 6. Anytime the maintenance feed is used to supply the shutdown board, the board is inoperable. [C.6]
1. The RHR pump aligned and operating to support RI I M operation should NOT be supplied from a shutdown board aligned to its maintenance supply.

5QN I3Pic 770/Li

1. Which ONE of the following identifies the reason the pressurizer manway must be open with air flow unobstructed whenever the reactor head is in place and the SG U-tubes are drained or pressurizer level is off scale low in accordance with O-GO-9, Refueling Procedure?

This ensures A. that on a loss of level in the RCS rapid refill capability using RHR pumps would be available.

B. that a steam void in the RCS piping would not prevent natural circulation flow.

C an adequate vent exists to allow gravity fill from RWST on a station blackout event.

D. an adequate vent exists to prevent a vacuum from being developed in the RCS piping on a rapid loss of RCS level.

3-OT-GO 1000 Revision 8 Page4of 132 I. PROGRAM WATTS BAR OPERATOR TRATNTNG II. COURSE A. LICENSE TRAINING B. NON-LICENSE TRAINING C. NOTP III. TITLE GO-b, REACTOR COOLANT SYSTEM DRAIN AND FILL OPERATIONS IV. LENGTH OF LESSON A. LICENSE TRAINING 3 Hours B. NON-LICENSE TRAINING 3 Hours V. TRAINING OBJECTIVES AR S S U OR T 0 0 A X X X X 1. Identify the condition(s) necessary for Reduced Inventory/Mid-Loop operations to exist.

X X X X 2. [Identify the Residual Heat Removal pump operating band during Mid-Loop operation. (SOER 88-3, Rec. 3c and SOER 85-4]

X X X X 3. [Describe the types of Main Control Room indications available to the operator for Reactor Coolant System level during Reduced Inventory/Mid-Loop operations. (SOER 88-3, Rec. 3a and SOER 85-4, Rec. la]

X X X X 4. [Identify the indication(s) of Residual Heat Removal pump cavitation and the checks required per GO-b. (SOER 88-3, Rec. 3.c)]

3-OT-GO1000 Revision 8 Page 5 of 132 V. TRAINING OBJECTIVES (Continued)

A R S S U 0 R T 0 0 A X X X X 5. [Identify the procedure to which the operator is referred if Residual Heat Removal cooling is lost while in during Reduced Inventory/Mid-Loop operations. (SOER 88-3 &

SOER 85-4)]

X X X X 6. Identify at what point can RuM requirements be exited.

X X X X 7. Identify the reason for each prerequisite and precaution as discussed in this lesson or provided in GO-lO.

X X X X 8. Discuss the major steps necessary to enter and exit Reduced Inventory/Mid-Loop operation as discussed in class.

VI. TRAINING AIDS A. Marker Boards B. Markers VII. MATERIALS One each per participant:

A. Appendices Appendix A, RCS Diagram For Reduced Inventory/MID-Loop Operation Appendix B, Level Instruction Elevation Comparison Chart Appendix C, Level Instrumentation Required Below el 725 (Reactor Flange)

(With Fuel in the Core)

B. Attachments Attachment-l: GO-b, REACTOR COOLANT SYSTEM DRAIN AND FILL OPERATIONS (Latest Revision)

Attachinent-2: Ultrasonic Level Measurement Instrumentation Operational Anomalies Attachment-3: SOER 88-3, Loss of RHR at Reduced Levels Attachment-4: Modifications To Zions Reactor Vessel Water Level Indicating System Attachment-5: PER 115184, NOUE Based Upon Potential RCS Leak MRC Report Attachment-6 SOER 09-1, Shutdown Safety

WBN 10-2011 NRC RO Exam As Submitted 8/15/2011

69. G 2.2.17 069 Given the following:

- Unit I is operating at 100% power.

- Maintenance is to be performed on AFW Pump I B-B.

- Protected Equipment portable sign stands have been placed locally at AFW Pump lA-A.

- An engineer request permission from the CRC to enter the area protected.

In accordance with ODM-4.0, Protected Equipment, which ONE of the following identifies...

(1) the organization responsible for the placement of the Protected Equipment portable sign stands and (2) who the CR0 would direct the engineer to contact to get approval to enter the area protected?

Organization Approval A Operations Shift Manager B. Operations Work Control SRO C. Maintenance Shift Manager D. Maintenance Work Control SRO Page 181

WBN 10-2011 NRC RO Exam As Submitted 811512011 DISTRACTOR ANALYSIS:

A. Correct, ODM-4, Protected Equipment, assigns the responsibility for the placement of the Protected Equipment signs to Operations and the CR0 would direct the engineer to the Shift Manager to get approval to enter the area protected.

B. Incorrect, Plausible because Operations has the responsibility for placing the signs and the CR0 directing the engineer to the Work Control SRO is plausible because the Work Control SRO is the individual who has the primary interface role with plant support organizations.

C. Incorrect, Plausible because Maintenance is the organization doing the work and because the CR0 directing the engineer to the Shift Manager to get approval to enter the area protected is correct.

D. Incorrect, Plausible because Maintenance is the organization doing the work and the CR0 directing the engineer to the Work Control SRO is plausible because the Work Control SRO is the individual who has the primary interface role with plant support organizations.

Page 182

WBN 10-2011 NRC RO Exam As Submitted 8/15/2011 Question Number: 69 Tier: 3 Group: N/A KIA: G2.2.17 Equipment Control Knowledge of the process for managing maintenance activities during power operations, such as risk assessment, work prioritization, and coordination with the transmission system operator.

Importance Rating: 2.6 / 3.8 10 CFR Part 55: 41.10 / 43.5 /45.13 IOCFR55.43.b: Not applicable KIA Match: K/A is matched because the question requires knowledge of the process for managing on-line work to reduce the risk of making redundant equipment inoperable while performing maintenance on safety related equipment.

Technical

Reference:

ODM-4.0, Protected Equipment, Revision 1 Proposed references None to be provided:

Learning Objective: 3-OT-T11240

03. Describe the procedures/processes for controlling temporary and aggregate (cumulative) risk.

Cognitive Level:

Higher Lower X Question Source:

New X Modified Bank Bank Question History: New question for the WBN 10/2011 NRC exam Comments:

Page 183

WBN Protected Equipment ODM-4.0 Operations Revision 1 Directive Manual Page 1 of 20 PURPOSE: The intent of protecting systems and components is to provide additional administrative barriers to guard against inadvertently rendering a component or system that is important to unit risk and nuclear safety inoperable or unavailable. It is also applicable to those systems and activities that pose a potential risk to generation.

APPROVAL: Approved by Greg Evans 5/28/10 Date

REFERENCES:

(A) SPP 10.7 Temporary Equipment Evaluation, approval # 04 1009-01 (B) Transient Combustible Evaluation # A09-00 18 (C) TI-12.16, Diesel Generator Outage T/S or SR Contingency Actions DIRECTIVE:

I. Responsibilities A. Shift Manager or Designee

1. Identify if protection of systems or components is required for emergent work.
2. Determine the appropriate method of protecting equipment if the methods described in this document are unable to be performed.
3. Directs the placement of Protected Equipment signs, tags or barriers on Protected Equipment and Systems prior to starting the task. Ensures this is documented on Attachment 2- Protected Equipment Log.
4. Ensures the removal of the specified Protected Equipment signs, tags or barriers upon completion of the task. Ensures the restoration is documented on Attachment 2- Protected Equipment Log.
5. Maintain the active Attachment 2- Protected Equipment Log (normally in the MCR) and discuss Protected Equipment and Systems at the POD and OPS turnover meetings.

B. Reviewing SRO/Work Week Manager (WWM)

1. For preplanned work, the SRO should review the Schedule for any activity or items and identify if protection for equipment or systems is required.
2. When a system or component is designated as being Protected, the SRO notifies the WWM to add an activity to the schedule to put the protection in place on the preceding shift. An activity should also be placed on the schedule to remove the protection following restoration of the redundant component.
3. The Reviewing SRO and the WWIvI shall ensure that planned work that could impact a protected system or component is çhçç within tiwwnow Qfproection.

WBN Protected Equipment ODM-4.0 Operations Revision 1 Directive Manual Page 2 of 20

4. LAW TI-124 (Equipment to Plant Risk Matrix), ensure that the Work Week risk assessment is reviewed and any required risk management actions are taken prior to the equipment being removed from available status. These actions should be reviewed 24 hours2.777778e-4 days <br />0.00667 hours <br />3.968254e-5 weeks <br />9.132e-6 months <br /> in advance by the shift to ensure they can be implemented.

II. General Requirements A. Applicability

1. This directive is applicable when the Unit is in Mode 1 though 4. If the Unit is in Modes 5, 6, or defueled, protected equipment requirements will be lAW with OSSDM 4.0, Operational Defense in Depth Assessment.
2. Prior to the start of an outage, any equipment that is protected on the unit will be transferred to the OSSDM Defense in Depth Log. Likewise, when the outage is completed, any remaining protection devices will be transfened to the Protected Equipment Log of this directive.

B. Equipment protection will be implemented in response to the following:

1. When a Tech Spec component is inoperable (or attendant equipment that will render Tech Spec Equipment inoperable) and the loss of the redundant component would result in a shutdown LCO of less than 8 hours9.259259e-5 days <br />0.00222 hours <br />1.322751e-5 weeks <br />3.044e-6 months <br /> or entry into TS 3.0.3.
2. lAW TI-124 (Equipment to Plant Risk Matrix) when PSA Risk levels are yellow or higher establish risk management actions are required. The work week risk assessment may have risk management actions that are required for the evaluated condition even if the assessment is green. These actions may consist of Protecting Specific equipment to maintain the risk profile acceptable.
3. When a component is out of service and the loss of an additional component serving the same function would threaten generation. For example, if one of the Stator Cooling Water Pumps is unavailable, the redundant pump should be protected.
4. Shift Manager/designee can determine level of protection for simple short duration activities.

For example, if a pump is placed in pull-to-lock for venting for a short duration, protection can be established by briefing on contingencies requiring return of component to service.

5. When deemed prudent by the Shift Manager. The Shift Manager has final authority in detemrining systems and equipment to be protected.

C. Methods of equipment protection:

1. Consider protection of the point of control by placing a protected equipment tag on the control switch. See Attachment 1 for an example.

WBN Protected Equipment ODM-4.0 Operations Revision 1 Directive Manual Page 3 of 20

2. Protect the components power source by placing a protected equipment placard on the energy source, usually electric breaker or air supply valve.
3. Protect access to the component by placing barriers and a protected equipment placard on the equipment room entrance or establishing a temporary barrier around freestanding equipment.
4. For areas with multiple entrance points, Protected Equipment controls should be placed at each entrance.

D. Placement Of Signs/Tags/Barriers

1. Signs, tags and barriers are used to protect vital system equipment. As such, they should be easily recognizable, be clean and easy to read, placed for broadest viewing range and should not be left in the field when not in use. An example of a barrier sign is provided in Attachment 1 of this directive. Other styles of postings and warnings may be employed, subject to the approval of the SM or designee.
2. If the equipment to be protected is inside a room, the protection boundary should be established at the room entrance. Consideration should be given to locking access doors.
3. For free standing equipment, barrier tape or ropes will be used to establish a barrier. The barrier will be placed in a neat and professional manner and will not be draped over plant equipment.

Stanchions will be used as necessary. Barriers will be placed in accordance with Reference (A).

Attachment 3 summarizes References (A) and (B), and shall be utilized when protecting equipment.

E. Use of the Protected Equipment Log:

1. The Protected Equipment Log is used to designate the systems and components that will be protected, and to track the installation and removal of signs and barriers.
2. Prior to releasing activities that will cause a component/equipment/system (that requires use of this directive) to be Inoperable, Unavailable, or Out Of Service ,the US/WCC Supervisor will ensure the Protected Equipment Program for the activity has been implemented as follows:

a) Identify and designate components to be protected on Attachment 2- Protected Equipment Log. Document the method utilized to protect the equipment in sufficient detail to allow understanding of the location of the protection. Ensure that any Risk Management Actions detailed in the Risk Assessment are addressed.

b) Obtain an independent review of the Risk Management Actions.

c) Direct an Operator to establish the Protected Equipment controls identified on Attachment 2-Protected Equipment Log for the affected equipment.

d) Document by completing the DATE iNSTALLED block on Attachment 2- Protected

WBN Protected Equipment ODM-4.0 Operations Revision 1 Directive Manual Page 4 of 20 Equipment Log when the associated controls are in place.

e) The SRO should document in the station logs that all Risk Assessment actions are addressed and all Protective equipment controls are in place in accordance with ODM-4.O Protected Equipment when the desired equipment has been protected.

2. When Protected Equipment controls are no longer required, the US/WCC Supervisor will ensure their removal as follows:

a) Ensure an Operator removes the Protected Equipment controls identified in Attachment 2-Protected Equipment Log. The Protected Equipment controls shall be stored only in the approved storage area(s) listed in Reference (A). Attachment 3 summarizes References (A) and (B), and shall be utilized when removing protected equipment controls.

b) When Protected Equipment controls are removed, document by completing DATE REMOVED on Attachment 2- Protected Equipment Log.

c) A station log entry should also be made when all Protective equipment controls are removed.

F. Authorization to Perform Work on or Near Protected Equipment

1. No work will be performed on or around the protected equipment. That includes cleaning, vibration checks, oil samples, scaffold construction or lagging repairs.
2. Access to the area should be limited to emergency situations (Operations, Fire Operations or Security) and non-intrusive monitoring of nmning equipment per the Operator Rounds. Access to a protected equipment area for any other reason must be authorized by the SM/US on a case-by-case basis prior to conducting the evolution.
3. When the SM authorizes entry or work, he/she shall ensure that the appropriate precautions are taken and the necessary oversight is provided based on the risk from the activity. The personnel accessing the area are to be briefed as to the potential consequences of an error while in the area and that the time in the area shall be minimized.
4. Once approved, the personnel conducting the evolution in the protected area will notify the control room prior to actual entry and immediately upon exit.

G. Records

1. The Protected Equipment Log is not required to be retained and may be discarded when completed.

WBN Protected Equipment ODM-4.0 Operations Revision 1 Directive Manual Page 5 of 20 Attachment 1 Figure 1 Sign Example 4 PROTECTED

  • EQUIPMENT DO NOT TNOP

- JVW-fP-tht J

Figure 2 Control Tag Example

WBN Protected Equipment ODM-4.0 Operations Revision 1 Directive Manual Page 6 of 20 Attachment 2 - Protected Equipment Log (page 1 of 14)

Instructions for the use of the Protected Equipment Log:

1. Identify and designate components to be protected on Attachment 2- Protected Equipment Log. Document the method utilized to protect the equipment in sufficient detail to allow understanding of the location of the protection.

a) Ensure that any Risk Management Actions detailed in the Risk Assessment are addressed and documented in Attachment 2- Protected Equipment Log.

SROIV b) Ensure that any additional measures taken are documented in Attachment 2-Protected Equipment Log. Utilize the guidance provided on Attachment 2-Protected Equipment Log for protecting previously identified items.

Equipment listed in Attachment 2- Protected Equipment Log that is not required to be protected may be marked N/A.

SRO IV c) Retain the applicable pages of Attachment 2- Protected Equipment log in the Main Control Room.

SRO

2. Direct an Operator to establish the Protected Equipment controls identified on Attachment 2- Protected Equipment Log for the affected equipment. Direct the operator to adhere to the requirements of Attachment 3 and Reference (A).
3. Document by completing the DATE iNSTALLED block on Attachment 2- Protected Equipment Log when the associated controls are in place.
4. Document in the station logs that all Risk Assessment actions are addressed and all Protective equipment controls are in place in accordance with ODM-4.O Protected Equipment when the desired equipment has been protected.
5. When Protected Equipment controls are no longer required, the US/WCC Supervisor will ensure removal as follows:

d) Ensure an Operator removes the Protected Equipment controls identified in Attachment 2- Protected Equipment Log, adhering to the storage requirements of Attachment 3 and Reference (A).

e) When Protected Equipment controls are removed, document by completing DATE REMOVED on Attachment 2- Protected Equipment Log.

6. Log in the station log that all Protective equipment controls are removed.

WBN Protected Equipment ODM-4.O Operations I I Revision 1 I Directive Manual Page 7 of 20 Attachment 2 - Protected Equipment Log (page 2 of 14)

Degraded/OOS Component:

PROTECTION DATE DATE PROTECTED COMPONENT COMMENTS (Signs, tape, etc) INSTALLED REMOVED

WBN Protected Equipment ODM-4.0 Operations Revision 1 Directive Manual Page 8 of 20 Attachment 2 Protected Equipment Log (page 3 of 14)

DegradedlOOS Component RIIR PUMP 1A Protected Component Location Date Installed Date Removed Comments 1-HS-74-20A RHR PMP B 1-M-6 lB RHR PUMP ROOM AUX BLDG ELEV 676 1 -BKR-74-20 6.9kv SD Bd lB-B RHR PUMP lB-B COMP 14 DegradedlOOS Component RUR PUMP lB Protected Component Location Date Installed Date Removed Comments 1-HS-74-1OA 1-M-6 RHR PMP A 1 A RHR PUMP ROOM AUX BLDG ELEV 676 1 -BKR 10 6.9kv SD Bd lA-A RHR PUMP I A-A COMP 14 DegradedlOOS Component SI PUMP 1A Protected Component Location Date Installed Date Removed Comments 1-HS-63-15A SI PMP B 1 -M-6 lB SI PUMP ROOM AUX BLDG ELEV 692 1-BKR-63-15 6.9kV SD Bd lB-B SAFETY INJ PUMP lB-B COMP 15 DegradedlOOS Component SI PUMP lB Protected Component Location Date Installed Date Removed Comments 1-HS-63-1OA l-M-6 SI PMP A 1A SI PUMP ROOM AUX BLDG ELEV 692 1-BKR-63-10 6.9kv SD Bd lA-A SAFETY NJ PUMP lA-A COMP 15

WBN Protected Equipment ODM-4.O Operations Revision 1 Directive Manual Page 9 of 20 Attachment 2 - Protected Equipment Log (page 4 of 14)

DegradedlOOS Component CCP l Protected Component Location Date Installed Date Removed Comments 1-HS-62-104A CCP B-B 1-M-5 lB CCP ROOM AUX BLDG ELEV 692 1-BKR-62-104 6.9kv SD Bd lB-B CENT CHARGING PUMP lB-B COMP 18 Degraded!OOS Component CCP lB Protected Component Location Date Installed Date Removed Comments 1-HS-62-108A 1-M-5 CCP A-A IA CCP PUMP ROOM AUX BLDG ELEV 692 1-BKR-62-108 6.9kv SD Bd lA-A CENT CHARGING PUMP IA-A COMP 18 DegradedlOOS Component CS PUMP 1A Protected Component Location Date Installed Date Removed Comments l-HS-72-1OA 1-M-6 CNTMT SPRAY PMP B lB CS PUMP ROOM AUX BLDG ELEV 676 1-BKR 10 6.9kv SD Bd lB-B CNTMT SPRAY PUMP lB-B COMP 13 Degraded!OOS Component CS PUMP lB Protected Component Location Date Installed Date Removed Comments 1-HS-72-27A 1-M-6 CNTMT SPRAY PMP A 1A CS PUMP ROOM AUX BLDG ELEV 676 1-BKR-72-27 6.9kv SD Bd lA-A CNTMT SPRAY PUMP lA-A COMP 13

WBN Protected Equipment ODM-4.0 Operations Revision 1 Directive Manual Page 10 of 20 Attachment 2 Protected Equipment Log (page 5 of 14)

DegradedlOOS Component CCS PUMP 1A Protected Component Location Date Installed Date Removed Comments 2-HS-70-5 1A O-M-27B CCS PMP C-S NORMAL ACB 1-HS-70-38A Rx Trip Potential O-M-27B CCS PMP lB-B C-S CCS PUMP AUX BLDG ELEV 713 Rx Trip Potential lB-B CCS PUMP AUX BLDG ELEV 713 O-BKR-70-51A 480V SD Bd 2B2-B NORM FDR FOR CCS PUMP C- COMP 2D 1-BKR-70-38 480V SD Bd lB 1-B Rx Trip Potential CCS PUMP lB-B COMP 3C DegradedlOOS Component CCS PUMP lB Protected Component Location Date Installed Date Removed Comments 1 -HS-70-46A Rx Trip Potential O-M-27B CCS PMP lA-A Rx Trip Potential lA-A CCS PUMP AUX BLDG ELEV 713 1 -BKR-70-46 480V SD Bd 1A1-A Rx Trip Potential CCS PUMP lA-A COMP 3B Degraded/OOS Component CCS PUMP CS Protected Component Location Date Installed Date Removed Comments 1-HS-70-46A 0 M 7B CCS PMP IA-A - --

lA-A CCS PUMP AUX BLDG ELEV 713 1-BKR-70-46 480V SD Bd 1A1-A CCS PUMP lA-A COMP 3B

WBN Protected Equipment ODM-4.0 Operations Revision 1 Directive Manual Page 11 of 20 Attachment 2 Protected Equipment Log (page 6 of 14)

Degraded!OOS Component ERCW PMP A-A Protected Component Location Date Installed Date Removed Comments O-HS-67-32A O-M-27A ERCW PMP B-A ERCW PMP B-A PS O-BKR-67-32 6.9kV SD Bd lA-A ERCW PUMP B-A COMP 9 Degraded/OOS Component ERCW PMP B-A Protected Component Location Date Installed Date Removed Comments O-HS-67-28A O-M-27A ERCW PMP A-A ERCW PMP A-A PS O-BKR-67-28 6.9kv SD Bd lA-A ERCW PUMP A-A COMP 8 Degraded!OOS Component ERCW PMP C-A Protected Component Location Date Installed Date Removed Comments O-HS-67-40A O-M-27A ERCW PMP D-A ERCW PMP D-A PS O-BKR-67-40 6.9kv SD Bd 2A-A ERCW PUMP D-A COMP 8 DegradedlOOS Component ERCW PMP fl-A Protected Component Location Date Installed Date Removed Comments O-HS-67-36A O-M-27A ERCW PMP C-A ERCW PMP C-A IPS O-BKR-67-36 6.9kv SD Bd 2A-A ERCW PUMP C-A COMP 9

WBN Protected Equipment ODM-4.0 Operations Revision 1 Directive Manual Page 12 of 20 Attachment 2 Protected Equipment Log (page7ofl4)

Degraded!OOS Component ERCW PMP E-B Protected Component Location Date Installed Date Removed Comments O-HS-67-5 1A O-M-27A ERCW PMP F-B ERCW PMP F-B PS O-BKR-67-5 1 6.9kv SD Bd lB-B ERCW PUMP F-B COMP 8 Degraded!OOS Component ERCW PMP F-B Protected Component Location Date Installed Date Removed Comments O-HS-67-47A O-M-27A ERCW PMP E-B ERCW PMP E-B PS O-BKR-67-47 6.9kv SD Bd lB-B ERCW PUMP E-B COMP 9 DegradedlOOS Component ERCW PMP G-B Protected Component Location Date Installed Date Removed Comments O-HS-67-59A O-M-27A ERCW PMP H-B ERCW PMP H-B PS O-BKR-67-59 6.9kv SD Bd 2B-B ERCW PUMP H-B COMP 8 Degraded!OOS Component ERCW PMP H-B Protected Component Location Date Installed Date Removed Comments O-HS-67-55A O-M-27A ERCW PMP G-B ERCW PMP G-B IPS O-BKR-67-55 6.9kv SD Bd 2B-B ERCW PUMP G-B COMP 9

WBN Protected Equipment ODM-4.0 Operations Revision 1 Directive Manual Page 13 of 20 Attachment 2 - Protected Equipment Log (page 8 of 14)

DegradedlOOS Component MDAFW PUMP lA-A Protected Component Location Date Installed Date Removed Comments 1-HS-46-56A-S 1 M 4 T-D AFWP T&T VLV -

1-HS-3-128A 1 M4 AFWPMPB-B - -

TDAFW PUMP Room Door AUX BLDG ELEV 692 MDAFW PUMP lB AUX BLDG ELEV 713 1-BKR-3-128 6.9kv SD Bd lB-B AFW PUMP lB-B COMP 10 DegradedlOOS Component MDAFW PUMP lB-B Protected Component Location Date Installed Date Removed Comments 1-HS-46-56A-S 1 M4 T-D AFWP T&T VLV - -

1-HS-3-118A 1 M 4 AFW PMP A-A - -

TDAFW PUMP Room Door AUX BLDG ELEV 692 MDAFW PUMP IA AUX BLDG ELEV 713 l-BKR-3-118 6.9kVSDBd1A-A AFW PUMP lA-A COMP 10

WBN Protected Equipment ODM-4.0 Operations Revision 1 Directive Manual Page 14 of 20 Attachment 2 - Protected Equipment Log (page 9 of 14)

Degraded!OOS Component TDAFW PUMP Protected Component Location Date Installed Date Removed Comments 1-HS-3-118A 1 M 4 AFW PMP A-A - -

1-HS-3-128A 1 M4 AFWPMPB-B - -

MDAFW PUMP A AUX BLDG ELEV 713 MDAFW PUMP B AUX BLDG ELEV 713 1-BKR-3-118 6.9kVSDBd lA-A AFW PUMP lA-A COMP 10 1-BKR-3-128 6.9kv SD Bd lB-B AFW PUMP lB-B COMP 10 1AEDG DGB 1BEDG DGB 480V Diesel Aux Bd IA1-A DGB ELEV 760 480V Diesel Aux Bd 1A2-A DGB ELEV 760 480V Diesel Aux Bd lB 1-B DGB ELEV 760 480V Diesel Aux Bd 1B2-B DGB ELEV 760 Degraded!OOS Component SD BD RM CHLR A-A Protected Component Location Date Installed Date Removed Comments 0-HS-31-49A 1 M 9 SDBDROOMA/CSYSB-B - -

6.9KV SD BD RM CHLR B-B AUX BLDG ELEV 737 0-BKR-31-4912 480V SD Bd 1B2-B SD BD RM CHLR PKG B-B COMP 3C

WBN Protected Equipment ODM-4.0 Operations Revision 1 Directive Manual Page 15 of 20 Attachment 2 - Protected Equipment Log (page 10 of 14)

Degraded!OOS Component SD BD RM CHLR B-B Protected Component Location Date Installed Date Removed Comments O-HS-31-400A 1 M 9 SD BD ROOM A/C SYS A-A - -

6.9KV SD BD RM CHLR A-A AUX BLDG ELEV 737 O-BKR-31-36/2 480V SD Bd 2A2-A SD BD RM CHLR A-A COMP 3C DegradedlOOS Component MCR CHILLER A-A Protected Component Location Date Installed Date Removed Comments O-HS-31-I1A 1-M-9 MCR CHLR & AHU B MCR CHLR-B-B AUX BLDG ELEV 737 O-BKR-31-96/2 480V SD Bd 1B2-B MCR CHLR -B-B COMP COMP 2B DegradedlOOS Component MCR CHILLER B-B Protected Component Location Date Installed Date Removed Comments O-HS-31-12A 1 M 9 MCRCHLR&AHUA - -

MCR CHLR-A-A AUX BLDG ELEV 737 O-BKR-31-80/2 480V SD Bd 1A2-A MCR CHLR-A-A COMP COMP 2C

WBN Protected Equipment ODM-4.0 Operations Revision 1 Directive Manual Page 16 of 20 Attachment 2 Protected Equipment Log (page 11 of 14)

DegradedlOOS Component EBR CHILLER A-A Protected Component Location Date Installed Date Removed Comments O-HS-3 1-3 IA 1-M-9 EBR CHLR & AHU B-B ELEC BD RM CHLR B-B CB ELEV 692 O-BKR-31-129/2 480V SD Bd 2B2-B ELEC BD RM CHLR B-B COMP 2B Degraded/OOS Component EBR CHILLER B-B Protected Component Location Date Installed Date Removed Comments O-HS-3 l-30A 1-M-9 EBR CHLR & AHU A-A ELEC BD RM CHLR A-A CB ELEV 692 O-BKR-3 1- 12 8/2 480V SD Bd 2A2-A ELEC BD RM CHLR A-A COMP 2C DegradedlOOS Component EGTS A-A Protected Component Location Date Installed Date Removed Comments O-HS-65-42A 0-M-27B EGTS FAN B & DISCH DMPR

  • EGTS FAN B AUX BLDG ELEV 757 O-BKR-65-42 480V C&A VENT BD lB 1-B EGTS FAN B/DISCH ISOL COMP 4D Degraded/OOS Component EGTS B-B Protected Component Location Date Installed Date Removed Comments O-HS-65-23A 0 M 7B EGTS FAN A & DISCH DMPR -

EGTS FAN A AUX BLDG ELEV 757 O-BKR-65-23 480V C&A VENT BD 1A1-A EGTS FAN AJDISCH ISOL COMP 4D

WBN Protected Equipment ODM-4.0 Operations Revision 1 Directive Manual Page 17 of 20 Attachment 2 - Protected Equipment Log (page 12 of 14)

DegradedlOOS Component ABGTS A-A Protected Component Location Date Installed Date Removed Comments 2-HS 1 57A 0-M-25 ABGTS FAN B-B AUX BLDG ELEV 737 2-BKR-30-157 480V C&A VENT BD 2B1-B ABGTS EXHAUST FAN 2B-B COMP 3D Degraded/OOS Component ABGTS B-B Protected Component Location Date Installed Date Removed Comments 1-HS 146-A O-M-25 ABGTS FAN A-A ABGTS FAN A-A AUX BLDG ELEV 737 1-BKR-30-146 480V C&A VENT BD 2A1-A ABGTS EXHAUST FAN lA-A COMP 3D

WBN Protected Equipment ODM-4.0 Operations Revision 1 Directive Manual Page 18 of 20 Attachment 2 Protected Equipment Log (page 13 of 14)

Degraded!OOS Component PG lA-A Protected Component Location Date Installed Date Removed Comments DG BUILDING ELEV 742 DG lA-A ROOM DOOR SEE NOTE 1 BELOW DOOR DOlO ROOM D104 1-HS-82-18 O-M-26 SEE NOTE 1 BELOW DG lA-A MODE SELECTOR Degraded/OOS Component PG 2A-A Protected Component Location Date Installed Date Removed Comments DG BUILDING ELEV 742 DG 2A-A ROOM DOOR SEE NOTE 1 BELOW DOOR DOl 1 ROOM D105 1 -HS-82-78 O-M-26 SEE NOTE 1 BELOW DG 2A-A MODE SELECTOR DegradedlOOS Component PG lB-B Protected Component Location Date Installed Date Removed Comments DG BUILDING ELEV 742 DG lB-B ROOM DOOR SEE NOTE 1 BELOW DOOR D012 ROOM D106 1 -HS-82-48 O-M-26 SEE NOTE 1 BELOW DG lB-B MODE SELECTOR DegradedlOOS Component PG 2B-B Protected Component Location Date Installed Date Removed Comments DG 2B-B ROOM DOOR DG BUILDING ELEV 742 SEE NOTE 1 BELOW DOORDO13ROOMD1O7 1 -HS-82-108 O-M-26 SEE NOTE I BELOW DG 2B-B MODE SELECTOR NOTE 1: REVIEW TI- 12.16 for additional protected equipment, dependant upon the scope of the DG outage.

WBN Protected Equipment ODM-4.0 Operations Revision 1 Directive Manual Page 19 of 20 Attachment 2 Protected Equipment Log (page 14 of 14)

Degraded!OOS Component 480 v Board Room Chiller Protected Component Location Date Installed Date Removed Comments 1-HS-31-461A, 480 BD RM

{1-M-9]

IA AHU lA-A 480V EBR A/C UNIT lA-A C&A Vent Bd 1A1-A (1-AHU-31-461) ChiC 1-HS-31-475A, 480 BD RIvI

[1-M-9]

1BAHU lB-B 480V ELECT BD RM A/C C&A Vent Bd lB 1-B lB-B C/11B 2-HS-31-461A, 480 BD RIvI

[2-M-9]

2A AHU 2A-A 480V EBR A/C UNIT 2A-A C&A Vent Bd 2A1-A (2-AHU-31-461) C/I1C 2-HS-31-475A, 480 BD 4

[2- M -]

9 2B AHU 2B-B 480V ELEC BD RM A/C C&A Vent Bd 2B1-B 2B-B ChuB N/A the cooler that is out of service.

WBN Protected Equipment ODM-4.0 Operations Revision 1 Directive Manual Page 20 of 20 Attachment 3 Protected Equipment Signs I Barriers Requirements Storage requirements:

1. The authorized storage areas, listed in Reference (A), are the only locations that have been evaluated for the storage of the protected equipment stanchions I portable sign stands.
2. They must be bundled by ropes and maintain a minimum 4 foot distance from safety related components I equipment.
3. In accordance with Reference (B), the following requirements shall be met when using / storing protected equipment stanchions/ portable sign stands:
a. Do not store more than 10 stanchions, or 5 portable sign stands, in any one approved storage location.

In use requirements to prevent impact to any safety related equipment or components:

1. The stanchions shall not be placed within 3 feet of safety related components I equipment. This distance is measured from the outer edge of the base to the safety related equipment! component
2. The portable sign stands shall not be placed within 4 feet of safety related components I equipment.

3-OT-T11240 Revision 6 Page 3 of 22 I. PROGRAM: Watts Bar Operator Training II. COURSE: Licensed Training III. TITLE: TI-124 Risk Assessment Equipment to Plant Risk Matrix IV. LENGTH OF LESSON: 1 HOUR V. TRAINING OBJECTIVES:

AR S S U OR T 0 0 A X X X 1. State the goals of risk management with regard to changes in plant design or operation.

. X X X 2. State the WBN Core Damage Frequency (CDF) and Large Early Release Frequencies and describe the major contributors to WBNs risk profile.

X X X 3. Describe the procedures/processes for controlling temporary and aggregate (cumulative) risk X X X 4. Describe the four risk levels used in the Equipment to Plant Risk Matrix and the significance of each level.

X X X 5. Describe the latest changes in the Risk Matrix and given plant conditions, use the Risk Matrix to determine the risk level.

X X X 6. Given a typical weekly schedule file, use the Sentinel program to evaluate removing equipment from service.

WBN 10-2011 NRC RO Exam As Submitted 8/1512011

70. G2.2.18 070 During shutdown for a Refueling Outage, which ONE of the following identifies the highest level of risk that can be voluntarily entered and who must approve entering the risk in accordance with NPG-SPP-07.2.1 1, Shutdown Risk Management.

A ORANGE Plant Manager B. ORANGE Outage Manager C. RED Plant Manager D. RED Outage Manager Page 184

WBN 10-2011 NRC RO Exam As Submitted 811512011 DISTRACTOR ANAL YSIS:

A. Correct, NPG-SPP-07.2. 11, Shutdown Risk Management, identifies an Orange Risk as the highest level of risk to be voluntarily entered and identifies the approval of the Plant Manager as being required to enter the risk.

B. Incorrect, Plausible because Orange is the highest level of risk that can be entered voluntarily and the Outage Manager can approve entering lower level risk.

C. Incorrect, Plausible because RED is a established risk category that can be entered but it is only entered due to unexpected condition caused by equipment failure/plant alignment and voluntary entry into the risk is not allowed. Also, because the Plant Manager is correct for entering the highest level of risk that can be voluntarily entered.

D. Incorrect, Plausible because RED is a established risk category that can be entered but it is only entered due to unexpected condition caused by equipment failure and is voluntary entry into the risk is not allowed. Also, because the Outage Manager can approve entering lower level risk.

NPG-SPP-07.2. 11 3.4 J.3 Orange Condition Considered high risk with less than the desired complement of equipment.

Written guidance/contingency plans should be made before entering preplanned conditions of this type. These can include, but are not limited to; temporary systems, temporary procedures, procedure revisions, or requirements in shift orders to compensate for degraded plant conditions. The plant manager must authorize this evolution.

3.4 J.4 Red Condition Considered very high risk with less than the adequate complement of plant equipment. This condition should be avoided and if inadvertently entered, immediate actions should be taken to return the plant to higher DID conditions (i.e., Orange, Yellow, or Green).

Previously written contingency plans will be utilized when attempting to restore from a Red Condition. If conditions change, or equipment failures result in red condition, prompt reversal of the evolution in progress, restoration of equipment or implementation of a contingency plan should occur.

Page 185

WBN 10-2011 NRC RO Exam As Submitted 8115/2011 Question Number: 70 Tier: 3 Group n/a KIA: G2.2.18 Equipment Control Knowledge of the process for managing maintenance activities during shutdown operations, such as risk assessments, work prioritization, etc.

Importance Rating: 2.6 / 3.9 IOCFRPart55: 41.10/43.5/45.13 IOCFR55.43.b: Not applicable KIA Match: K/A is matched because the question requires knowledge of the process for managing risk associated with maintenance activities during shutdown conditions.

Technical

Reference:

NPG-SPP-07.2.11, Shutdown Risk Management, Revision 0000 Proposed references None to be provided:

Learning Objective: 3-OT-Tl1240

03. Describe the procedures/processes for controlling temporary and aggregate (cumulative) risk.

Cognitive Level:

Higher Lower X Question Source:

New X Modified Bank Bank Question History: New question for the WBN 10/2011 NRC exam.

Comments:

Page 186

NPG Standard Shutdown Risk Management NPG-SPP-07.2.11 Programs and Rev. 0000 Processes Page 12 of 30 3.3 Outage Planning and Schedule Development (continued)

8. Independent Safety Evaluation A safety / licensing individual with extensive experience in identifying shutdown safety issues and developing methods to reduce shutdown risk.
9. Technical Expertise An individual with technical experience of outage activities that are first time evolutions or evolutions that significant technical expertise was relied upon to make decisions in outage planning for appropriate logic ties or sequencing.

3.4 Outage Execution A. The Work Control Outage Group should maintain risk books and risk status boards that reflect the risk status of the outage unit based on the outage schedule and the Defense-In-Depth DID Checklist. The location of these risk books should be the Main Control Room for the outage unit and the Outage Control Center I Work Control Center. The risk status board should be located in the Outage Control Center.

Additional Outage Risk Status Boards may be provided in locations to be determined by Outage Management.

B Risk books should contain the current copy of the Safety Plan, and the time to boil curves, and the latest Defense In Depth (DID) sheets.

C. Risk sheets should be included in the daily outage review reports I packages as a means to communicate the current plant risk status to all departments. Each department should use the Risk Sheet as reference during shift briefings.

D. Each 24 hour2.777778e-4 days <br />0.00667 hours <br />3.968254e-5 weeks <br />9.132e-6 months <br /> period, an assessment will be performed of the plant schedule using Outage Risk Assessment Management (ORAM) software. An ORAM report should be generated from the updated schedule. This ORAM report should be compared to information provided by Operations on the daily DID assessment, based on actual equipment availability. This comparison provides a redundant verification for determining the risk each day during the outage. The new ORAM report should be compared to the original ORAM report generated prior to the outage, and the Outage Safety Plan, to determine if new conditions exist. The Shift Outage Manager, with the 0CC, should determine if the new conditions require further actions. If it is determined that existing contingency plans need to be modified, or new contingency plans developed, the Shift Outage Manager should oversee the development of the plan, and provide to PORC and Senior Management for approval.

E. Defense-In-Depth DID Checklist should be filled out each shift by Operations and be provided to the Shift Outage Manager for review. See Attachment 1 through 3 for the appropriate DID sheet. Any Delta(s) indentified from the current ORAM will be reviewed by the Operation Duty manager and Outage Management to determine the correct Risk level call. This will be documented on the DID sheet and in the corrective action program as appropriate.

F. Shift Manager concurrence should be obtained for release and closure of outage system work windows that have impact on shutdown safety functions.

NPG Standard Shutdown Risk Management NPG-SPP-07.2.11 Programs and Rev. 0000 Processes Page 13 of 30 3.4 Outage Execution (continued)

G. Schedule activities to minimize time in lowered and reduced inventory. The schedule should be thoroughly planned and prepared prior to entering either lowered inventory conditions or reduced inventory conditions, by verifying the availability of equipment needed to restore from a lowered inventory and / or reduced inventory condition well before the condition that would challenge outage risk or defense in depth.

H. To minimize risk at lowered inventory or reduced inventory, a specific CIPTE briefing shall be conducted in accordance with NPG-SPP-06.9.1, Conduct of Testing, prior to entry into a lowered inventory and I or reduced inventory condition.

The Operation Outage Shift Manager should ensure proper distribution of the DID Checklist to the Main Control Room for the Outage Unit, The Outage Control Center, Operation Work Control and other locations deemed necessary.

J. DID Checklist and/or the outage schedule as processed through ORAM can be used for the evaluation of the outage risk level. These items are only tools and are not to be relied upon solely, or in place of, individuals with training and experience in the area of outage risk management. The risk conditions are defined as follows:

1. Green Condition Considered fully acceptable with respect to relative level of risk.

May either be full or less than full complement of equipment to satisfy a specific Safety Function.

2. Yellow Condition Considered an acceptable level of risk. Usually will indicate a less than full complement of equipment to satisfy a specific Safety Function.
3. Orange Condition Considered high risk with less than the desired complement of equipment. Written guidance/contingency plans should be made before entering preplanned conditions of this type. These can include, but are not limited to; temporary systems, temporary procedures, procedure revisions, or requirements in shift orders to compensate for degraded plant conditions. The plant manager must authorize this evolution.
4. Red Condition Considered very high risk with less than the adequate complement of plant equipment. This condition should be avoided and if inadvertently entered, immediate actions should be taken to return the plant to higher DID conditions (i.e., Orange, Yellow, or Green). Previously written contingency plans will be utilized when attempting to restore from a Red Condition. If conditions change, or equipment failures result in red condition, prompt reversal of the evolution in progress, restoration of equipment or implementation of a contingency plan should occur.

K. When ORAM or the DID checklist results in a condition identifying an unplanned reduced DID condition not previously planned, The Outage Manager shall also ensure that the Work Control (WCC) SRO is taking compensatory measures to protect equipment that could potentially further reduce the current Defense in Depth level.

Unplanned DID changes should be captured in the Corrective Action Program. This information should be checked as part of the shift turnover process to ensure the information is still valid and necessary.

NPG Standard Shutdown Risk Management NPG-SPP-07.2.11 Programs and Rev. 0000 Processes Page 14 of 30 3.4 Outage Execution (continued)

L. The Unit Outage Manager should notify Operations and Outage Management of any planned or unplanned changing DID conditions as soon as possible.

M. The Outage Manager working with the Outage Control Center should initiate actions to exit from an unplanned Orange or Red condition as soon as possible.

N. Risk management actions are established and defined in NPG-SPP-07.3, Work Activity I Risk Management Process, including the following types of actions:

1. Discussion of activity with operating shift and operator approval of planned evolution.
2. Pre-job briefing of maintenance personnel emphasizing the risk aspects of the evolution.
3. Presence of appropriate technical personnel for appropriate portions of the activity.
4. Pre-staging of parts and materials.
5. Walk down tag out and activity prior to conducting maintenance.
6. Conduct of training and mock ups to familiarize personnel with the activity.
7. Perform activity around the clock.
8. Establish contingency plans to restore the out of service rapidly, if needed.
9. Minimizing other work in areas that could affect event initiators (e.g. reactor protection system areas, switchyard, diesel generator rooms, switchgear rooms) to decrease the frequency of initiating events that are mitigated by the safety function served by the out of service SSC.
10. Minimize work in areas that could affect other redundant systems such that there is enhanced likelihood of the availability of the safety functions at issue served by the SSCs in those areas.
11. Outage briefings will be held prior to entry into any scheduled ORAM orange condition.
12. Activities will be scheduled to evaluate current plant conditions against the Outage Safety Plan, prior to entry into a scheduled high risk period or evolution.
0. Establishment of alternate success paths for performance of the safety function of the out of service SSC (note that this equipment does not necessarily have to be in the scope of the Maintenance Rule).

3-OT-T11240 Revision 6 Page 3 of 22 I. PROGRAM: Watts Bar Operator Training II. COURSE: Licensed Training III. TITLE: TI-124 Risk Assessment Equipment to Plant Risk Matrix IV. LENGTH OF LESSON: 1 HOUR V V. TRAINING OBJECTIVES:

AR S S U OR T 0 0 A X X X 1. State the goals of risk management with regard to changes

  • in plant design or operation.

V x X X 2. State the WBN Core Damage Frequency (CDF) and Large Early Release Frequencies and describe the major contributors to WBNs risk profile.

X X X 3. Describe the procedures/processes for controlling temporary and aggregate (cumulative) risk X X X 4. Describe the four risk levels used in the Equipment to Plant Risk Matrix and the significance of each level.

X X X 5. Describe the latest changes in the Risk Matrix and given plant conditions, use the Risk Matrix to determine the risk level.

X X X 6. Given a typical weekly schedule file, use the Sentinel V program to evaluate removing equipment from service.

WBN 10-2011 NRC RO Exam As Submitted 8115/2011

71. G 2.2.40 07 Given the following plant conditions:

- The crew implemented AOI-33, Steam Generator Tube Leak, due to a tube leak in SG #1 and has placed the unit in Mode 3.

- ES-0.1, Reactor Trip Response, has been completed and the crew has resumed performance of AOl-33.

As a result of actions directed in AOl-33, which ONE of the following will require an entry into a Technical Specification Action statement?

A. Closing the SG #1 Main Steam Isolation Valve.

B. Adjusting the SG #1 PORV controller setpoint to 90%.

CY Isolating the TD AFW pump steam supply valve from SG #1.

D. Blocking Low Steam line pressure and Low PZR pressure SI actuation.

DISTRACTOR ANAL YSIS:

A. Incorrect, Plausible because the MSIVs components included in Tech Specs but the act of closing them does not require a Tech Spec entry.

B. Incorrect, Plausible because the SG PORVs are components included in Tech Specs but the action to adjust the setpoint does not require a Tech Spec entry.

C. Correct, Isolating the TD AFW pump steam supply from the SG #1 requires entry into LCO 3.7.5 action statements with the unit in Mode 3.

D. Incorrect, Plausible because the SSPS functions can result in a Tech Spec entry but the action to block the signal at directed in the procedure does not require a Tech Spec entry.

Page 187

WBN 10-2011 NRC RO Exam As Submitted 8/15/2011 Question Number: 71 Tier: 3 Group n/a KIA: G2.2.40 Equipment Control Ability to apply Technical Specifications for a system.

Importance Rating: 3.4 / 4.7 10 CFR Part 55: 41.10 / 43.2 / 43.5 / 45.3 IOCFR55.43.b: Not applicable K/A Match: K/A is matched because the question requires the ability to recognize conditions resulting in an applicable Tech Spec LCO being required to be entered.

Technical

Reference:

AOl-33, Steam Generator Tube Leak, Revision 0034 Tech Spec LCO 3.7.5, Auxiliary Feedwater System, Amendment 55 Proposed references None to be provided:

Learning Objective: 3-OT-SYSOO3B

7. Identify in which Modes the CST and the AFW System are governed by Tech Specs.

Cognitive Level:

Higher X Lower Question Source:

New Modified Bank X Bank Question History: WBN bank question Comments:

Page 188

WBN Steam Generator Tube Leak AOl-33 Unit I Rev. 0034 Step Action/Expected Response Response Not Obtained 3.0 OPERATOR ACTIONS(continüed)

CAUTION If RCS press increases above 1970 psig, SI actuation circuits will automatically unblock.

17. DEPRESSURIZE RCS to block SI actuation at 1970 psig.
a. USE normal pzr Spray. a. IF letdown is in service, THEN USE auxiliary spray.
  • IF NOT, THEN USE one pzr PORV.
b. CHECK RCS pressure - LESS b. DO NOT CONTINUE until RCS THAN 1970 PSIG. pressure is less than 1970 psig.
c. BLOCK SI actuation:
  • Low streamline pressure
  • Low pzr pressure Page 15 of 42

WBN Steam Generator Tube Leak AOl-33 Unit I Rev. 0034 Step _Action/Expected Response Response Not Obtained 3.0 OPERATOR ACTIONS (continued)

20. ALIGN PORV on leaking SG(s):
a. CHECK controller in AUTO and a. PLACE controller in AUTO and set set at 90%. at9O%.
b. CHECK HS in P-AUTO b. PLACE HS in P-AUTO.
c. CHECK leaking SG pressure c. WHEN leaking SG pressure is less LESS THAN 1130 PSIG. than 1130 psig, THEN PERFORM Step [201 d.
d. CHECK leaking SG PORV d. OBTAIN RADPRO support, and CLOSED. LOCALLY ISOLATE leaking SG PORV:
  • Loop 1, 1-ISV-1-619 [SVR]
  • Loop 2, 1-ISV-I -620 [NVRJ
  • Loop 3, 1-ISV-1-621 [NVR]
  • Loop 4, 1-ISV-I -622 [SVR]
21. CHECK TD AFW pump supplied from ISOLATE leaking SG steam supply to Intact SG. TD AFW pump:

SGI- 1-FCV-1-15 OR

  • SG4- 1-FCV-1-16 Page 17 of42

WBN Steam Generator Tube Leak AOl-33 Unit I Rev. 0034 Step Action/Expected Response Response Not Obtained 3.0 OPERATOR ACTIONS (continued)

V

22. ISOLATE leaking SG. V
a. CLOSE leaking SG MSIV and a. I.F leaking SG(s) MSIV OR bypass bypass valve, can NOT be closed, THEN PERFORM App B, Leaking SIG Isolation and Local Steam Moisture Trap Isolation.
b. CLOSE leaking SG Blowdown Isolation Valves.
23. MAINTAIN leaking SG Level GREATER THAN 22%.
a. CHECK leaking SG NR level a. MAINTAIN feed flow to leaking GREATER THAN 22%. SG.

WHEN leaking SG NR level is greater than 22%,

THEN ISOLATE feed flow to leaking SG.

b. ISOLATE feed flow to leaking SG.
24. DO NOT CONTINUE until leaking SG isolated.

Page 18 of 42

AFW System 3.7.5 3.7 PLANT SYSTEMS 3.7.5 Auxiliary Feedwater (AFW) System LCO 3.7.5 Three AFW trains shall be OPERABLE.

Only one AFW train, which includes a motor driven pump, is required to be OPERABLE in MODE 4.

APPLICABIUTY: MODES 1, 2, and 3, MODE 4 when steam generator is relied upon for heat removal.

ACTIONS INUILZ LCO 3.0.4.b is not applicable when entering MODE 1.

CONDITION REQUIRED ACTION COMPLETION TIME A. One steam supply to turbine A.1 Restore steam supply to 7 days driven AFW pump OPERABLE status.

inoperable.

10 days from discovery of failure to meet the LCO B. One AFW train inoperable in B.1 Restore AFW train to 72 hours8.333333e-4 days <br />0.02 hours <br />1.190476e-4 weeks <br />2.7396e-5 months <br /> MODE 1, 2 or 3 for reasons OPERABLE status.

other than Condition A. AND 10 days from discovery of failure to meet the LCO (continued)

Watts Bar-Unit 1 3.7-11 Amendment 55

AFW System 3.7.5 ACTIONS (continued)

CONDITION REQUIRED ACTION COMPLETION TIME C. Required Action and C.1 Be in MODE 3. 6 hours6.944444e-5 days <br />0.00167 hours <br />9.920635e-6 weeks <br />2.283e-6 months <br /> associated Completion Time for Condition A or B not met. AND OR C.2 Be in MODE 4. 18 hours2.083333e-4 days <br />0.005 hours <br />2.97619e-5 weeks <br />6.849e-6 months <br /> Two AFW trains inoperable in MODE 1,2, or3.

D. Three AFW trains inoperable D.1 NOTE in MODE 1, 2, or 3. LCO 3.0.3 and all other LCO Required Actions requiring MODE changes are suspended until one AFW train is restored to OPERABLE status.

Initiate action to restore Immediately one AFW train to OPERABLE status.

E. Required AFW train E.1 Initiate action to restore AFW Immediately inoperable in MODE 4. train to OPERABLE status.

Watts Bar-Unit 1 3.7-12

AFW System 3.7.5 SURVEILLANCE REQUIREMENTS SURVEILLANCE FREQUENCY SR 3.7.5.1 Verify each AFW manual, power operated, and 31 days automatic valve in each water flow path, and in both steam supply flow paths to the steam turbine driven pump, that is not locked, sealed, or otherwise secured in position, is in the correct position.

SR 3.7.5.2 NOTE --

Not required to be performed for the turbine driven AFW pump until 24 hours2.777778e-4 days <br />0.00667 hours <br />3.968254e-5 weeks <br />9.132e-6 months <br /> after 1092 psig in the steam generator.

Verify the developed head of each AFW pump at the 31 days on a flow test point is greater than or equal to the required STAGGERED TEST developed head. BASIS SR 3.7.5.3 NOTE Not applicable in MODE 4 when steam generator is relied upon for heat removal.

Verify each AFW automatic valve that is not locked, 18 months sealed, or otherwise secured in position, actuates to the correct position on an actual or simulated actuation signal.

(continued)

Watts Bar-Unit 1 3.7-13 Amendment 13

AFW System 3.7.5 SURVEILLANCE REQUIREMENTS (continued)

SURVEILLANCE FREQUENCY SR 3.7.5.4 ------- NOTES---

1. Not required to be performed for the turbine driven AFW pump until 24 hours2.777778e-4 days <br />0.00667 hours <br />3.968254e-5 weeks <br />9.132e-6 months <br /> after 1092 psig in the steam generator.
2. Not applicable in MODE 4 when steam generator is relied upon for heat removal.

Verify each AFW pump starts automatically on an 18 months actual or simulated actuation signal.

SR 3.7.5.5 Verify proper alignment of the required AFW flow Prior to entering paths by verifying flow from the condensate storage MODE 2 after initial tank to each steam generator. fuel loading and whenever unit has been in MODE 5 or6for>30 days Watts Bar-Unit 1 3.7-14 Amendment 13

3-OT-SYSO03B Revision 13 Page 5of60 PROGRAM Watts Bar Operator Training II. COURSES License Training NOTP License Req ualification NAUO Req ualification III. TITLE Auxiliary Feedwater System IV. LENGTH OF LESSON A. License Training 4 Hours B. NOTP 4 Hours License Requalification and NAUO Requalification times will be determined after objectives are identified.

V. TRAINING OBJECTIVES A SS R

U RT 0

0 CA

1. State the design basis of the AFW system in accordance with FSAR section x 10.4.9.
2. State the function of the AFW system in accordance with the System Description Manual.
3. Describe how the Auxiliary Feedwater pumps are protected from low flow conditions.

X X X X 4. Identify the steam generators that each AFW pump supplies.

5. Describe the Reserve Auxiliary Feedwater Capacity in the CSTs and include x x how it is ensured.
6. Identify the required CST volume needed for AFW operation as stated in Tech Specs and the basis for this volume.
7. Identify in which Modes the CST and the AFW System are governed by Tech

_ Spcs - -

3-OT-SYSOO3B Revision 13 Page6f6O A SS R

U RT C

C CA

8. Describe the automatic opening signal for the ERCW supply valves to the AFW system.

X X X X 9. Identify the power supplies to the Motor-Driven AFW pumps.

X X X X 10. Identify the A-Auto start signals of the Motor-Driven AFW pumps.

X X X X 1 1. Describe the automatic actuations that occur when an AFW pump is started.

12. Explain the reasons (bases) for precautions associated with AFW operation as described in SOl-3.02. (NOTE 3)
13. Describe the caution associated with opening the ERCW supply valves as stated in SCI-3.02.
14. Describe the normal and alternate steam supplies to the Turbine-Driven AFW Pump.

X X X X 15. Describe the sequence of events that occur on a steam supply swapover.

16. Identify the isolation signals of the steam supply valves (FCV-1-17 &18) to the Turbine-Driven AFW pump.

X X X X 17. Identify the Turbine-Driven Auxiliary Feedwater pump Auto start signals.

18. List the trips on the Auxiliary Feedwater Pumps and include any local steps to resetting them.
19. Identify the pressure at which the Motor-Driven AFW pump discharge pressure control valve should be set.

X X X X 20. Identify what the SIG level setpoint is for the AFW system LCVs.

21. Explain how to manually control S/G levels with the Motor-Driven Auxiliary Feedwater pumps LCVs.
22. Identify the initiating signals that swap the Motor-Driven Pumps LCVs from normal to the bypass LCV.
23. Using plant drawings, determine the effect of a loss of instrument air/control power on the following valves/components:

X X X a. MDAFWP regulating valve (main and bypass)

b. TDAFWP regulating valve
c. AFW pumps.
24. [Identify the problem that disables the AFW pumps and describe Watts Bars solution to the problem. (SOER 84-3, Rec. #1-4)]

3-OT-SYSO03B Revision 13 Page7of60 A SS R

U RT 0

0 CA

25. Describe the changes that take place when the Turbine-Driven AFW pump transfer switch (XS-46-57) is placed in Aux position.
26. Identify the steps to gain local control of the Turbine-Driven Auxiliary Feedwater pump and SG levels.
27. Describe how to take manual control of the Turbine-Driven AFW pump from the control room.
28. [Describe how to reset an electrical and mechanical overspeed trip on the X X X X Turbine-Driven AFW pump, both electrically and locally (SOER-82-08, Rec.

4)]

29. Explain the purpose of each controller and indicator on the local Control panel for the Turbine-Driven AFW Pump.
30. [Identify three (3) contributing factors that have left the Trip and Throttle valve in a tripped position when the Operators thought it was reset and describe controls put in place at WBN to keep this from occurring. (SOER 82-8, Rec.
  1. 4)]
31. [Identify the dominant causes of failures leading to significant losses of AFW.

(SOER 86-1)

32. [Identify three possible adverse effects which could occur if the AFW pump turbine overspeeds (SOER-89-01)]

X X X X 33. Sketch the Auxiliary Feedwater System, beginning at the CSTs and ending at the SGs. (NOTE 1)

X X X X 34. List each main isolation valve in the suction line of the Auxiliary Feedwater System and state in which building it is located. (NOTE 1)

X X X X 35. List each of the checks that should be made on the Auxiliary Feedwater Pumps normally and while running.

3-OT-SYSOO3B Revision 13 Page 8 of 60 A S S R

U R T 0

0 0 A xx XX36. Describe the in-plant location of the following: (NOTE 1)

a. Auxiliary Feedwater Pumps
b. Auxiliary Feedwater Level Control Valves
c. Flood Mode Spool Pieces
d. Auxiliary Feedwater Supply Valves
e. Steam Supply Valves to Turbine-Driven Auxiliary Feedwater Pumps
f. Turbine-Driven Auxiliary Pump Trip and Throttle Valve
g. Motor-Driven AFW Recirc Valves
37. Correctly locate control room controls and indications associated with the AFW system, including: (NOTE 2)
a. AFWPs xxx b. AFW regulating valve
c. SIG level
d. CST level NOTE 1: Objective accomplished during walk downs of the system. Student is responsible for this information.

NOTE 2: Objective accomplished during simulator demo of the system. Student is responsible for this information.

NOTE 3: Use latest revision of SOl-3.02 for discussion of the precautions and limitations for AFW.

WBN 10-2011 NRC RO Exam As Submitted 811512011

72. G 2.3.13 072 Given the following:

- The Unit is in Mode 6, with core reload in progress.

- The 13th fuel assembly is currently being set in the core.

- The Shutdown Monitor was last reset when the count rate was 6 cps.

- Annunciator8l-B, SOURCE RANGE HI FLUXAT SHUTDOWN, alarms.

- Both Source Range monitors indicate the same count rate value and have risen unexpectedly.

Which ONE of the following identifies...

(1) the minimum count rate that would have caused annunciator 81-B to alarm and (2) the action(s) required as a result of this change in Source Range counts?

Minimum count rate Required actions A. 9 cps Stop fuel movement and ensure containment is evacuated.

B 19 cps Stop fuel movement and ensure containment is evacuated.

C. 9 cps Stop fuel movement, Containment evacuation NOT required.

D. 19 cps Stop fuel movement, Containment evacuation NOT required.

Page 189

WBN 10-2011 NRC RO Exam As Submitted 811512011 DISTRA CTOR ANALYSIS:

A. Incorrect, Plausible because a count rate of 9 is a change from 6 cps by a factor of 0.5 (instead of an increase by 0.5 decade) and fuel movement being stopped and containment being evacuated is correct.

B. Correct, A rise to 19 cps will cause the alarm. (See calculation below) The alarm results from the count rate rising by a factor on 0.5 decades above background which was set when the monitor was reset at 6 cps. Since both SRMs have increase by a factor of 2, fuel movement is required to be stopped in accordance with FHI-7. Also, the ARI for the annunciator required an announcement to be made to evacuate containment.

C. Incorrect, Plausible because a count rate of 9 is a change from 6 cps by a factor of 0.5 (instead of an increase by 0.5 decade) and the applicant may be aware that a count rate rise by a factor of 2 requires fuel movement to be stopped and not aware the alarm does cause containment to be evacuated.

D. Incorrect, Plausible because the count rate rising to 19 cps is correct and the applicant may be aware that a count rate rise by a factor of 2 requires fuel movement to be stopped and not aware the alarm does cause containment to be evacuated.

Log (x/6) = 0.5 X!6 = 3.16227766 X= 3.16227766X6 X= 18.97366596cps Page 190

WBN 10-2011 NRC RO Exam As Submitted 811512011 Question Number: 72 Tier: 3 Group: n/a K/A: G2.3.13 Radiation Control Knowledge of radiological safety procedures pertaining to licensed operator duties, such as response to radiation monitor alarms, containment entry requirements, fuel handling responsibilities, access to locked high-radiation areas, aligning filters, etc.

Importance Rating: 3.4 / 3.8 10 CFR Part55: 41.12/43.4/45.9/45.10 10CFR55A3.b: Not applicable K/A Match: K/A is matched because the question requires the knowledge of how a change in source count rate affects fuel handling activities, as well as what causes a radiation alarm and the required radiological safety actions resulting from the alarm.

Technical

Reference:

ARI-81-87, NIS & Rod Controls, Revision 0033 FHI-7, Fuel Handling and Movement, Revision 0035 Proposed references None to be provided:

Learning Objective: 3-OT-SYSO92A

10. Identify all indications, alarms, permissives and trips associated with the SRMs.

Cognitive Level:

Higher X Lower Question Source:

New Modified Bank Bank X Question History: WBN bank question SYSO9O 003 with wording changes.

Comments:

Page 191

WBN NIS & Rod Controls ARI-81-87 Unit I Rev. 0033 Page 5 of 48 81-B Source Setpoint Bistables: 0.5 decade increase above 1 N31AX background count rate at shutdown SOURCE RANGE (variable) HI FLUX IN32AX AT SHUTDOWN NC31CX NC32CX (Page 1 of 1)

Probable A. Positive reactivity insertion from:

Cause: 1. RCS cooldown

2. RCS dilution
3. Rod withdrawal
4. Xenon decay Corrective [1] ENSURE CNTMT evacuation alarm initiates.

Action: [2] IF count rate continues to rise, THEN

[2.1] ENSURE all personnel are evacuated from CNTMT.

[2.2] GO TO AOl-34, IMMEDIATE BORATION.

[3] GO TO AOl-4, NUCLEAR INSTRUMENTATION MALFUNCTION.

References:

1 -45W600-92-1 1 -45W600-92-2 1 -45W600-57-1 3 AOl-4 AOl-34

3.0 PRECAUTIONS AND LIMITATIONS A. During refueling operation when ABI/CVI crosstie is required, the following conditions apply: (Refer to Tech Spec Bases 3.3.6, 3.3.8, 3.7.12, & 3.9.8.)

1. 1-HS-90-410-A & 415-B are placed in the REFUEL position prior to the beginning of refuel operations (Mode 5) when the Contaiment and/or Annulus is open to the Auxiliary Bldg ABSCE spaces and returned to NORMAL postion prior to entering Mode 4 from Mode 5 unless repositioned by another approved configuration document (eg. 18 Month GO Test, DG Blackout Test, etc.). When moving irradiated fuel inside Containment, both trains of the Containment Purge System must be operable and at least one train must be operating, or Containment must be isolated.
2. If 1-HS-90-410-A & 415-B are required to be placed in the NORMAL position during movement of irradiated fuel in containment or the auxiliary bldg, then fuel movement must be stopped or containment must be isolated.
3. When moving irradiated fuel in the Auxiliary Bldg with the Containment open to the Auxiliary Bldg ABSCE spaces, Containment Purge System may be operated, but all Containment ventilation isolation valves and associated instrumentation must remain operable.
4. Both trains of ABGTS must remain operable if Contaiment and/or Annulus is open to the Auxiliary Bldg ABSCE spaces during movement of irradiated fuel inside Containment.

B. Fuel Handling Supervisor (FHS) will ensure persons involved in fuel handling are CERTIFIED / QUALIFIED per NPG-SPP-08.1. Appendix C of this instruction contains Fuel Movement Roles and Responsibilities C. Caution must be used when working around open pits.

D. Preventing loose articles from falling into refueling areas is accomplished per NPG-SPP-06.5.

E. If communication is lost between the Auxiliary Building, the Control Room, or Containment, fuel loading should stop UNTIL communication is reestablished.

If communication is lost between the Control Room and Containment then fuel loading MUST STOP.

F. The following guidelines apply for movement of fuel assemblies.

1. Slow speed (lowest practical speed) is required in the following zones:
a. Guiding and seating of handling tool/gripper mast onto fuel assembly.

WBN FLIeI Handling And Movement FHI-7 Unit I & 2 Rev. 0035 Page 9 of 43 3.0 PRECAUTIONS AND LIMITATIONS (continued)

b. Fuel assembly bottom nozzle approximately 10 inches above to 10 inches below top of Upender, RCCA change fixture, storage cell funnel, or within 10 inches of full down.
c. Inserting fuel-bottom nozzle approximately 10 inches above to 10 inches below the top of seated fuel assemblies and within 10 inches of full down.
d. Fuel movements when an assembly is in close contact with another assembly.
e. Fuel movements outside of zones above, when NOT in contact with adjacent fuel or clearly above the top of seated fuel assemblies may be 40 ft/mm maximum speed.
2. Operations must be stopped immediately IF:
a. Unanticipated count rate increase by a factor of 2 on ALL responding nuclear monitoring channels during any single loading step after the initial nucleus of 8 fuel assemblies are loaded except an anticipated change.
b. Count-rate on ANY nuclear channel increases by a factor of 5 during any single loading step after initial nucleus of 8 assemblies are loaded except an anticipated change due to movement of a source bearing assembly or detector to fuel assembly neutronic coupling.
c. Water in SFP or Core is not clear enough to view Fuel top Nozzles with supplemental lighting.
d. During Core loading, the Lower Core Support Plate and Guide Pins are not visible enough with supplemental lighting to check proper engagement of the Bottom Nozzle with the Lower Core Plate pins.
e. C decreases by more than 20 ppm, on 2 successive coolant 6

samples, until the decrease is explained.

f. Reactor Cavity leakage is such that the FHS is concerned with maintaining adequate inventory. [c.7]

G. Current plant configuration does NOT permit irradiated fuel to be moved in the cask loading area. (Reference PER 96939 and DCN 52211).

SYSO9O 003 3 /1J /J iL jc-j j Given the following plant condiflons:

- The Unit is in Mode 6, with core reload in progress.

- The Shutdown Monitor was last reset when the count rate was 6 cps.

- Source Range instruments indicate counts are rising unexpectedly.

- Annunciator Source Range High Flux Level At Shutdown alarms Which of the following indicates the rise in count rate required to cause the Source Range High Flux Level At Shutdown annunciator to alarm, and, the action(s) required if the alarm is valid?

Alarm setpoint Required actions

a. 9 cps; Announce over the PA system to evacuate containment and notify the Refueling SRO to immediately suspend core alterations.
b. 9 cps; ONLY notify the Refueling SRO to immediately suspend core alterations c.v 18 cps; Announce over the PA system to evacuate containment and notify the Refueling SRO to immediately suspend core alterations.
d. 18 cps; ONLY notify the Refueling SRO to immediately suspend core alterations

3-OT-SYSO92A Revision 10 Page 5of 102 I. PROGRAM Waifs Bar Operator Training II. COURSES A. License Training B. Non-License Training C. NOTP III. TITLE EXCORE NUCLEAR INSTRUMENTATION IV. LENGTH OF LESSON A. Licensed Training 4 hour4.62963e-5 days <br />0.00111 hours <br />6.613757e-6 weeks <br />1.522e-6 months <br /> B. Non-Licensed Training 4 hours4.62963e-5 days <br />0.00111 hours <br />6.613757e-6 weeks <br />1.522e-6 months <br /> C. NOTP 8 hours9.259259e-5 days <br />0.00222 hours <br />1.322751e-5 weeks <br />3.044e-6 months <br /> V. TRAINING OBJECTIVES A R S S U 0 R T 0 0 A x x x x 01. State the purpose of the Excore Instrumentation System.

x x x x 02. State how the Excore Instrumentation determines reactor conditions.

X X X X 03. State and explain the Source Range Monitor (SRM) scale range and units.

x x x x 04. State and explain the Intermediate Range Monitor (IRM) scale range and units.

X X X X 05. State and explain the Power Range Monitor (PRM) scale range and u n its.

x x x x 06. Explain why detector overlap is desirable.

X X X 07. Discuss how each set of detectors overlap readings.

x x x 08. Describe how the NIS protection channels are divided, including color code, channel number and equipment.

3-OT-SYS092A Revision 10 Page 6 of 102 A R S S U 0 R T 0 0 A x x x x 09. State the purpose of the SRMs.

x x x x 10. Identify all indications, alarms, permissives and trips associated with the SRMs. (AUOs NOT RESPONSIBLE for alarms and permissives.)

x x x x ii. Describe the number, location and principle of operation of the SRMs.

x x x x 12. Describe the SRMs method of gamma discrimination.

x x x x 13. State the purpose of the IRMs.

x x x x 14. Identify all indications, alarms, permissives and trips associated with the IRMs. (AUOs NOT RESPONSIBLE for alarms and permissives.)

x x x x 15 Describe the number, location and principle of operation of the IRMs.

x x x x 16. Describe the IRMs method of gamma discrimination.

17. DELETED.

x x x x 18. State the purpose of the PRMs.

x x x x 19. Identify all indications, alarms, permissives and trips associated with the PRMs. (AUOs NOT RESPONSIBLE for alarms and permissives.)

x x x x 20. Describe the number, location and principle of operation of the

. PRMs.

x x x x 21. Describe the PRMs method of gamma discrimination.

X X X 22. State the Excore Instrumentation System parameters that are governed by Technical Specifications.

X X 23. Given a failure of one or more components of the Excore Instrumentation System, identify and/or explain the applicable technical specification LCO and required actions.

x x x x 24. [Explain the effects of a significant core change on the Excore Nuclear Instrumentation.]

3-OT-SYSO92A Revision 10 Page 7 of 102 A R S S U 0 R T 0 0 A x x x 25. Describe the location and purpose of the Gamma-Metrics Wide Range Signal Processor.

x x x 26. Explain the operation of the Gamma-Metrics Shutdown Monitor including the 1IM circuit/display and the High Flux at Shutdown alarm circuit.

X X X 27. List the basic equation for excore detector Al and Axial Offset (A.O.).

X X X 28. Given a set of PRM data, calculate QPTR.

x x x 29. Discuss the calibration of a Power Range instrument when operating at power including how the value for a total channel output is determined and how it is set.

X X X 30. Discuss the calibration of the Power Range Al circuits including how the value for core Al is determined.

x x x 31. Describe the distribution of Instrument and Control Power in the Nuclear Instrumentation System including the effects of a loss of one or both supplies under various plant conditions.

x x x 32. Discuss excore instrument failures and their effects on the plant with and without operator action.

x x x 33. Define DNB and DNBR and the operational effects associated with the excore nuclear instrumentation.

x x x x 34. State the power supplies for the excore nuclear instruments from memory.

WBN 10-2011 NRC RO Exam As Submitted 8115/2011

73. G 2.3.4 073 Given the following plant conditions:

- A LOCA has occurred and a SAE has been declared.

- The TSC and OSC have been activated.

- To prevent damage to equipment needed for protection of the public, it is recommended that an individual make an entry into the IA-A Safety Injection Pump Room IA.

- Projected dose rate in the pump room is 1.0 xl 5 mR/hr.

- Duration of the exposure is expected to be 6 minutes.

Which ONE of the following individuals must authorize this exposure?

A. Radcon Manager B. Site Emergency Director C. Plant Manager D. Site Vice President Page 192

WBN 10-2011 NRC RO Exam As Submitted 811512011 DIS TRACTOR ANAL YSIS:

A. Incorrect, per EPIP- 15 of the Radiological Emergency plan the Site Emergency Director must authorize emergency dose limits in excess of TVA admin limits and 10 CFR 20. Applicant may select since the Radcon Manager is responsible for completing the paperwork and he must authorize exceeding the limit during non-emergency conditions.

B. Correct, with a projected dose of 1.OE+5 mr/hr, the total dose to respond to this emergency condition is 10 Rem.((1.OE+5 mr/hr/ 60 minutes) x 6 minutes). Per EPIP- 15 of the Radiological Emergency plan the Site Emergency Director must authorize emergency dose limits in excess of TVA administrative limits and 10 CFR 20.

C. Incorrect, per EPIP-15 of the Radiological Emergency plan the Site Emergency Director must authorize emergency dose limits in excess of TVA admin limits and 10 CFR 20. Applicant may select since Plant Manager must approve exceeding 5 R during non-emergency conditions.

D. Incorrect, per EPIP- 15 of the Radiological Emergency plan the Site Emergency Director must authorize emergency dose limits in excess of TVA admin limits and 10 CFR 20. Applicant may select since Site Vice President is the highest management level position staffed in the TSC during an event and additional approval must authorized prior exceeding 5 R during non-emergency conditions Page 193

WBN 10-2011 NRC RO Exam As Submitted 811512011 Question Number: 73 Tier: 3 Group n/a KIA: G2.3.4 Radiation Control Knowledge of radiation exposure limits under normal or emergency conditions.

Importance Rating: 3.2 / 3.7 10 CFR Part 55: 41.12 / 43.4 / 45.10 IOCFR55.43.b: Not applicable KIA Match: K/A is matched because the question requires the ability to calculate a dose, realize the dose is above normal limits and have the knowledge of the position required to authorize receiving the dose.

Technical

Reference:

EPIP-15, Emergency Exposure Guidelines, Revision 0014 NPG- SPP-05.1, Radiological Controls, Revision 0002 Proposed references None to be provided:

Learning Objective: 3-OT-RAD0003

6. List the extreme emergency exposure guidelines Cognitive Level:

Higher Lower X Question Source:

New Modified Bank Bank X Question History: WBN bank question G2.3.4 072 9 (question was used on the SQN 1/2009 exam)

Comments:

Page 194

NPG Standard Radiological Controls NPG-SPP-05.1 -

Programs and Rev. 0002 Processes Page 12 of 45 3.4.1 External Exposure Control* (continued)

F. Administrative Dose Levels*

1. Occupational radiation dose limits at NPG facilities are consistent with the limits given in 10 CFR 20.
2. Administrative dose levels (ADLs) to be used as guidelines for maintaining doses below regulatory limits have been established within the NPG and shall be observed for routine work. This program is not applicable to minors or declared pregnant women. Obtain appropriate station supervision and radiation protection management approval to increase a workers administrative dose level.

Examples of a bona fide need for a dose extension are that 1) the unique ability or experience of the individual will minimize collective dose; and 2) other qualified individuals with lower doses are not available. However, do not repeatedly give the same individuals higher dose for the sake of reducing overall collective dose.

The NPG Administrative Dose Level Program is summarized in Table 1 below:

TABLE I ADMINISTRATIVE DOSE LEVEL PROGRAM Dose Equivalent (Rem) Requirement Authorization to exceed (signatures)

Up to 0.5 TEDE Statement of current year dose Not applicable (or 1.5 LDE or 5.0 SDE) at and previous years dose signed TVA by individual Up to 1.0 TEDE NRC FORM-4 or equivalent to Not applicable (or 3.0 LDE or 10 SDE) all document current year and sources previous years dose equivalent To exceed 1.0 TEDE Same as above RPM/RSO (or 3.0 LDE or 10 SDE) all sources To exceed Form-4 information must be RPM/RSO, Plant 5.0 TEDE 3 all sources verified and a Planned Special , and Site VP 1

Manager 2 Exposure initiated or SED as appropriate.

To exceed IN 4 all Form-4 must be verified RPM/RSO, Plant sources , and Site VP 1

Manager 2 or SED as appropriate.

At non-nuclear plant sites, this will be the RSOs immediate supervisor.

2 At non-nuclear plant sites, this will be the applicable WA VP:

Authorizations for a planned special exposure will only be considered in an exceptional situation when alternatives that might avoid the dose estimated to result from the planned special exposure are unavailable or impractical.

Total effective dose equivalent should not exceed IN rem, where N equals the individuals age in years at last birthday, without the authorization signatures delineated.

3. ADLs are based on dosimeter(s) used in determining the reported dose. Results which exceed an administrative level, based on other dosimeter data, do not violate the ADL.

NPG Standard Radiological Controls NPG-SPP-05.1 Prgrarn hd Rev. 0002 Processes Page 18 of 45 3.4.2 Emergency Exposure Guidance and Planned Special Exposures (PSE)*

A. It is consistent with the risk concept to accept exposures leading to doses in excess of those appropriate for routine operation when recovery from an accident or major operational difficulty is necessary. Saving of a life, measures to circumvent substantial exposure to the general public, or the preservation of valuable installations may be sufficient cause for accepting above normal exposures. Dose limits for an emergency cannot be specified, but they should be commensurate with the significance of the objective and held to the lowest practicable level that the emergency permits.

Guidance for emergency exposure is specified in the TVA Radiological Emergency Plan.

Any decision to embark on emergency operations which would result in exposures in excess of 10 CFR 20.1201 should be done in consultation with the most senior member of RP who is available on a timely basis. Personnel must be made aware of possible consequences of such an exposure and selected on a voluntary basis.

Emergency team members who are expected to respond to a radiological emergency must be made aware of the consequences of such exposure.

B. T\/A may authorize an individual to receive doses in addition to and accounted for separately from the yearly dose limits prescribed in 10 CFR 20.1201 provided that certain conditions are satisfied. These conditions are delineated in 10 CFR 20.1206.

Authorizations for a planned special exposure will only be considered in an exceptional situation when alternatives that might avoid the dose estimated to result from the planned special exposure are unavailable or impractical. Individuals required to authorize planned special exposures are indicated on Table 1.

Personnel must be made aware of possible consequences of such an exposure. PSEs are to be conducted in accordance with 10 CFR 20.1206, 20.21 05, 20.2106, and with guidance from Regulatory Guide 8.35.

WBN Emergency Exposure Guidelines EPIP-15 Unit 0 Rev. 0014 Page 13 of 16 Appendix B (Page4of4)

Authorization To Exceed Occupational Dose Limits Form The persons listed below are authorized to exceed the WA dose limits for the whole body and extremities during the evaluation or mitigation of an emergency situation.

Emergency limits are in excess of 10 CFR 20.1201 limits.

The persons listed below acknowledge they have volunteered for this assignment, and have been briefed on the emergency situation, and have been made aware of possible consequences of the estimated radiation dose(s).

Hand carry or FAX to the TSC for SED Signature NAME EMPLOYEE ID# SIGNATURE DOSE LIMIT (REM) 1 2

3 4

5 6

7 8

9 10 RP Survey No. (If Applicable):

Emergency Location(s)

Estimated Doses Remarks:

Prepared by:

Authorized by:

Site Emergency Director Hand carry or FAX to the OSC after SED signs.

WBN Emergency Exposure Guidelines EPlP-15 UnitO Rev. 0014 Page 16 of 16 Appendix D (Page 1 of 1)

Definitions Accident Exposure Exposure to radiation or radioactive materials that results from an unexpected event. Accident exposure refers to the immediate consequences of the unexpected event and the immediate corrective/mitigative actions of personnel present at the scene where the event occurred. Accident exposures are not controlled by the Radiological Emergency Plan (REP), and may be reportable to the NRC under 10 CFR 20 and/or 10 CFR 50.72.

CDE Committed Dose Equivalent CEDE Committed Effective Dose Equivalent DAC Derived Air Concentration DDE Deep Dose Equivalent Emergency Exposure Exposure to radiation or radioactive materials that is the result of actions taken in response to an emergency condition classified and declared pursuant to the Watts Bar Radiological Emergency Plan (REP).

Emergency exposure refers to radiation exposure caused by those assessments, corrective, and mitigative actions that are required on an immediate basis to protect human lives, or to prevent or minimize the collective exposure of large populations.

Such activities are directed by the Control Room or by the TSC.

LDE Eye Dose Equivalent Life Saving Action Those actions related to the search and rescue of injured persons.

Planned Special Exposure (PSE) As defined in 10 CFR 20, an infrequent exposure to radiation, separate from and in addition to the annual dose limits. PSEs might be warranted in the recovery phase. However, it is unlikely that the PSE requirements could be met during the initial phases of the emergency.

RADIREM For purposes of this implementing procedure, radiation exposure as expressed in units of RIhr and subunits, thereof, is equivalent to dose (Rad) and dose equivalent (Rem) based on ANSI N 13.11 development and terminology. Any acute dose greater than 10 Rem is generally denoted in units of Rad, since that level is considered as the accident range of personnel exposure. Any dose less than that level is considered the protective range of personnel exposure. For purposes of this procedure the assumption of 1 Rad = 1 Rem is assumed for all levels of exposure.

SDE Shallow Dose Equivalent TEDE Total Effective Dose Equivalent

f/J/i1

1. G 2.3.4 072 Given the following plant conditions:

- A LOCA has occurred and a SAE has been declared.

- The TSC and OSC have been activated.

- To prevent damage to equipment needed for protection of the public, it is recommended that an individual make an entry into the lA-A Safety Injection Pump Room IA.

- Projected dose rate in the pump room is I .0 xl 0 mR/hr.

- Duration of the exposure is expected to be 6 minutes.

Which of the following individuals must authorize this exposure?

a. Radcon Manager
b. Site Emergency Director
c. Plant Manager
d. Site Vice President

3-OT-RAD0003 Revision 3 Page3f23 I. PROGRAM WATIS BAR OPERATOR TRAINfNG II. COURSE A. LICENSE TRAINING B. LICENSE PREP III. TITLE RADIATION PROTECTION STANDARDS AND GUIDELINES IV. LENGTH OF LESSON A. LICENSE TRAINING 1 HOUR B. LICENSE PREP 1 HOUR V. TRAINING OBJECTIVES AR S S U OR T 0 0 A NA X X X 1. State the purpose of regulatory limits for radiation protection.

NA X X X . Identify NRC dose limits for individuals in restricted areas.

NA X X X 3. Identify the reactor site criteria for an exclusion area and a low population zone.

NA X X X I.. List or identify TVA radiation dose goals.

NA X X X 5. Identify TVAs recommendations for women who have volunteered to participate in the prenatal exposure program.

NA X X X 5. List the extreme emergency exposure guidelines.

NA X X X 7. Explain the ALARA concept.

3 -OT-RAD0003 Revision 3 Page I6of23 X. LESSON BODY INSTRUCTOR NOTES F. Emergency Exposure Guidance and Planned Objective 6 Special Exposures (PSE)

From SPP-5.1 It is consistent with the risk concept to accept exposures leading to doses in excess of those appropriate for routine operation when recovery from an accident or major Refer to EPIP 15 operational difficulty is necessary. Saving of a life, measures to circumvent substantial exposure to the general public, or the preservation of valuable installations may be sufficient cause for accepting above normal exposures. Dose limits for an emergency cannot be specified, but they should be commensurate with the significance of the objective and held to the lowest practicable level that the emergency permits. Guidance for emergency exposure is specified in TVA Radiological Emergency Plan.

2. Any decision to embark on emergency Annual Limits operations which would result in exposures in excess of 10CFR2O.1201 should be done in consultation with the most senior member of Radiation Protection who is available on a timely basis. Personnel must be made aware of possible consequences of such an exposure and selected on a voluntary basis.

Emergency team members who are expected to respond to a radiological emergency must be made aware of the consequences of such exposure.

3. TVA may authorize an individual to receive Annual and Special Exposure doses in addition to and accounted for Limits separately from the yearly dose limits prescribed in 10CFR2O.1201 provided that certain conditions are satisfied. These conditions are delineated in 10CFR2O.1206.
4. Personnel must be made aware of possible consequences of such an exposure. PSEs are Records Requirements to be conducted in accordance with 10CFR2O.1206, 20.2105, 20.2106, and with guidance from Regulatory Guide 8.35.

3 -OT-RAD0003 Revision 3 Page 17of23 X. LESSON BODY INSTRUCTOR NOTES TVA Emergency Dose Guidelines (TEDE) EPIP-1 5 Also reference Appendix C and D of EPIP 15 Dose Remarks Section 3.1 has additional information 30 rad for lens of 10 Rem May be received to prevent eye, 100 rad any organ, or skin serious damage to plant or and extremity hazard to personnel Plus 3 Time rule see notes 25 Rad May be received to save a life Section 3.1 also states 75 rad lens of eye,250 rad any other Plus 10 Time rule see notes.

organ, or skin and extremity.

5. Personnel must be made aware of possible consequences of such an exposure and selected on a voluntary basis. Such doses are permitted only once per lifetime.

Emergency team members who are expected to respond to a radiological emergency must be made aware of the consequences of such exposure.

G. NRC Regulatory Guides Makes available to licensees, methods of implementing specific parts of NRC regulations.

2. Compliance is not required, but are normally followed by licensees.
3. There are ten broad divisions of regulatory guides. Division 8, Occupational Health, is the most applicable to radiation protection.
a. Reg. Guide 8.8 Information Relevant to Ensuring that Occupational.Radiation Exposure at Nuclear power Stations will be As Low As Is Reasonably Achievable.

WBN 10-2011 NRC RO Exam As Submitted 811512011

74. G2.4.11 074 Given the following plant conditions:

- Unit I is operating at 50% power, with all systems aligned normally with Train A CCP and CCS pumps in service.

- A loss of lA-A 6.9KV Shutdown Board due a differential relay operation occurs.

- The OAC observes RCP Seal Water Injection flow to all RCPs to be 0.

- The Control Building AUO reports extensive damage to the lA-A 6.9 KV Shutdown Board bus bars.

- The crew implements AOl-43.01, Loss of Unit I Train A Shutdown Boards.

Under these conditions, RCP seal cooling is (1) and in accordance with AOl-43.01, the crew will (2) c)

A. available Isolate letdown prior to starting CCP lB-B.

B. unavailable Isolate letdown prior to starting CCP I B-B.

C. available Start CCP I B-B prior to isolating letdown.

D. unavailable Start CCP I B-B prior to isolating letdown.

Page 195

WBN 10-2011 NRC RO Exam As Submitted 811512011 DISTRA CTOR ANAL YSIS:

A. Correct, The loss of the lA-A 6.9KV Shutdown Board will cause a loss of the IA Component Cooling Water pump, which will in turn cause an automatic start of the 18-B Component Cooling Water Pump due to a low pressure and an automatic start of the TBBP 18-B due to low flow. The Component Cooling Water flowing through the thermal barrier heat exchanger will cool the RCS water flowing into the seal prior to the manual start of the CCP lB-B. The AOl will then have letdown isolated prior to the start of the CCP lB-B.

B. Incorrect, RCP seal cooling would not be unavailable. Component Cooling Water flow through the thermal barrier heat exchanger would be supplying the cooling, but letdown would be isolated prior to starting the CCP lB-B. Plausible, since the loss of the IA-A 6.9KV Shutdown Board will cause a loss of the IA Component Cooling Water pump, which would cause a loss of seal cooling if the Component Cooling Water pump lB-B did not have an auto start generated and because isolating letdown prior to starting CCP 18-B is correct.

C. Incorrect, Plausible, since seal cooling will be available as described in B above.

However the AOl will have letdown isolated before the CCPIB-B is manually started. Plausible because seal cooling being available is correct and the AOl would have started the CCP 18-B prior to isolating letdown if the component cooling water flow through the thermal barrier heat exchanger had not been present.

D. Incorrect, RCP seal cooling would not be unavailable. Component Coollng Water flow through the thermal barrier heat exchanger would be supplying the cooling, and the CCP lB-B would not be started prior to isolating letdown. Plausible, since the loss of the IA-A 6.9KV Shutdown Board will cause a loss of the IA Component Cooling Water pump, which would cause a loss of seal cooling if the Component Cooling Water pump 18-B did not have an auto start generated and the AOl would have started the CCP 18-B prior to isolating letdown if the Component Cooling Water flow through the thermal barrier heat exchanger had not been present.

Page 196

WBN 10-2011 NRC RO Exam As Submitted 8/1512011 Question Number: 74 Tier: 3 Group n/a K/A: 062 G2.4.11 Emergency Procedures / Plan Knowledge of abnormal condition procedures.

Importance Rating: 4.0 / 4.2 IOCFRPart55: 41.10/43.5/45.13 IOCFR55.43.b: Not applicable K/A Match: K/A is matched because the question requires knowledge of an abnormal procedure (AOl-43.01 and Annunciator response instructions).

Technical

Reference:

AOl-43.01, Loss of Unit 1 Train A Shutdown Boards, Revision 0009 ARI-1 02-1 08, HVAC and CVCS, Revision 0026 Proposed references None to be provided:

Learning Objective: 3-OT-A014300

4. Demonstrate ability/knowledge of AOl, by:
a. Recognizing Entry conditions
b. Responding to Actions
c. Responding to Contingencies (RNO)
d. Responding to Notes/Cautions Cognitive Level:

Higher X Lower Question Source:

New Modified Bank Bank X Question History: WBN bank question 062 G2.4.11 047 used in 06/0210 on an audit exam.

Comments:

Page 197

WBN Loss of Unit I Train A Shutdown AOI-43.O1 Unit I Boards Rev. 0009 Step Action/Expected Response Response Not Obtained 3.1 Initial Actions (continued)

NOTES

  • Appendix A provides a list of affected equipment
  • RCPs can be operated for up to 10 minutes after a loss of CCS flow.
3. MONITOR RCP seal cooling MANUALLY START available:
  • Seal injection flow
  • CCSPumpIB-B.

Ci OR IF seal cooling not restored, 1r CCS flow through Thermal THEN Barrier Heat Exchangers MONITOR RCP trip criteria (reference vuo AOI-24).

vei --Md

4. EVALUATE ERCW supply on A Train IF any A Train Diesel Generator running headers: with NO ERCW cooling, THEN
a. ENSURE at least one A Train ERCW Pump In-service: 1) EMERGENCY STOP A Train DGs
  • ERCWPumpC-A 2) OPEN 1-FCV-67-68, DG IA-A ERCW SUP from Hdr. 2B, [DG RM
  • ERCWPumpD-A IA-A]
b. START second pump as needed.
3) OPEN 2-FCV-67-68, DG 2A-A ERCW SUP from Hdr. 2B, [DG RM 2A-A]
4) RESET and EMERGENCY START A Train Diesel Generators Stopped due to lack of ERCW Cooling.

Page 7 of 69

WBN HVAC & CVCS ARI-1 02-1 08 Unit I Rev. 0026 Page 40 of 47 108-A Source Setpoint HI: 1-FS-62-93NB 150 gpm CHARGING LO: Computer Point F0128A 47 gpm with BOTH FLOW HIILO 75 gpm orifice valves CLOSED OR 55 gpm with EITHER A 75 gpm orifice valve (Page 1 of 1)

OPEN Probable A. System pipe break Cause: B. Charging pump tripped C. Malfunction of Pressurizer Level Control System Corrective [1] IF ALL the following conditions exist: &

Action:

  • Any RCP Thermal Barrier Out-of-Service
  • In-Service Charging pump trips
  • RCP seal injection flow required THEN IMMEDIATELY START available charging pump to restore seal flow.

[2] CHECK 1-Fl-62-93A, CHARGING FLOW [1-M-5] to determine if flow is high or low.

[3] CHECK 1-Ll-68-320, -335A, and -339A, PZR LEVEL [1-M-4].

[4] IF PZR Level Control System malfunction, THEN GO TO AOl-20.

[5] IF charging flow is low, THEN CHECK letdown temperature AND EVALUATE increasing charging flow OR ISOLATE letdown. cIOJ1,A(-Ui

[6] IF charging is lost, THEN IMMEDIATELY ISOLATE letdown per SOl-62.01.

[7] DETERMINE cause of problem AND INITIATE corrective action, as necessary.

[8] REFER TO SOl-62.01 for CVCS system operation.

References:

1 -47W61 0-62-2 I -47W809-1 AOI-20 SOl-62.01

3-OT-A014300 Rev 5 Page 4 of 130 I. PROGRAM Watts Bar Operator Training II. COURSE License Training III. TITLE AOI-43, Loss of 6.9KV Shutdown Board IV. LENGTH OF LESSON License Training 1.5 Hours NOTP 1.0 Hours V. TRAINING OBJECTIVES ARSS UORT 0 OA X X X 1. Demonstrate ability to recognize a loss of any 6.9KV Shutdown Board.

X X X 2. Analyze alarms and indications for loss of a 6.9KV Shutdown Board, and evaluate their importance to system operation per AOl.

X X X 3. Given plant conditions, determine if Tech Spec entry is required, what actions(s) must be taken, and the bases for those actions.

X X X 4. Demonstrate ability/knowledge of AOl, by:

a. Recognizing Entry conditions
b. Responding to Actions
c. Responding to Contingencies (RNO)
d. Responding to Notes/Cautions X X X X 5. Discuss methods of restoring power to a 6.9KV Shutdown Board X X X X 6. Describe the Purpose/Goal of AOl-43, Loss of 6.9KV Shutdown Board.

WBN 10-2011 NRC RO Exam As Submitted 811512011

75. G2.4.31 075 Given the following plant conditions:

- The plant is operating at 100% power.

- Annunciator 114-A, SSPS-A GENERAL WARNING is LIT.

Which ONE of the following (1) lists a condition that will cause the alarm and (2) describes what indication the operator dispatched locally will use to determine the cause of the alarm, in accordance with the ARI?

A. Reactor Trip Bypass Breaker Board edge LED lights associated A is racked in. with the Semi-Automatic Tester.

B. Blown Ground Return Fuse. Board edge LED lights associated with the Semi-Automatic Tester.

C. Reactor Trip Bypass Breaker Status Lights on the outside of the A is racked in. Local Panels.

D. Blown Ground Return Fuse. Status Lights on the outside of the Local Local Panels.

Page 198

WBN 10-2011 NRC RO Exam As Submitted 811512011 DISTRA CTOR ANAL YSIS:

A. Incorrect, Plausible, since the local action is correct. Racking in the Train A bypass breaker causes a separate alarm.

B. Correcl, LED lights are used to determine cause of alarm, and a blown ground return fuse is a specific cause of the GENERAL WARNING ALARM.

C. Incorrect, Plausible, since the local action is associated with determining the cause of a ROD URGENT FAILURE alarm. Racking in the Train A bypass breaker causes a separate alarm.

D. Incorrect, Plausible, since the local action is associated with determining the cause of a ROD URGENT FAILURE alarm, a blown ground return fuse is a specific cause of the GENERAL WARNING ALARM.

Page 199

WBN 10-2011 NRC RO Exam As Submitted 8/1512011 Question Number: 75 Tier: 3 Group n/a KIA: G2.4.31 Emergency Procedures/ Plan Knowledge of annunciator alarms, indications, or response procedures.

Importance Rating: 4.2 / 4.1 IOCFRPart55: 41.10/45.3 IOCFR55.43.b: Not applicable KIA Match: K/A is matched because the question requires knowledge of an annunciator response procedure.

Technical

Reference:

ARI-109-115, CVCS & RHR- RPS & ESF, Revision 0018 Proposed references None to be provided:

Learning Objective: 3-OT-SYSO99A 14 Identify where General Warning indications can be found.

Cognitive Level:

Higher Lower X Question Source:

New Modified Bank Bank X Question History: WBN Bank question 012 G2.4.31 036 used on 5/2008 exam Comments:

Page 200

WBN CVCS & RHR RPS & ESF

- ARI-109-115 Unit I Rev. 0018 Page 35 of 48 Source Setpoint 114-A SSPS Master Relay K524 See Probable Cause SSPS-A GEN WARNING (Page 1 of 1)

NOTE Board-edge LEDs on the Al 08 Semi-Automatic Tester (top row of boards, 08 slot) can be used to identify the probable Thstr Br-d cause. Number after the probable cause below is the LED irt associated with that condition.

cDth LIDs

) cF1o-t F1O5

- cFS

(_) D92 Probable A. Loss of SSPS 48V or 1 5V power supply (CR92)

Cause: B. Reactor Trip Bypass Breaker closed (CR105)

C. Mode Selector switch in TEST (CR105)

D. Input Error Inhibit switch in INHIBIT (CR101)

E. Multiplexer test switch in INHIBIT (CR103)

F. Loose or missing circuit card (CR98)

G. Logic A switch NOT in OFF (CR103)

H. Permissives switch NOT in OFF (CR105)

I. Memories switch NOT in OFF (CR105)

J. Blown ground return fuse (CR101 and CR 105)

K. Loss of AC Power K646 Relay (CR105)

NOTE SSPS General Warning in Train A and Train B will initiate a reactor trip.

Corrective [1] DISPATCH Operator to determine cause of alarm.

Action: [2] ENSURE proper test performance.

[3] NOTIFY Work Control to initiate corrective action, as necessary.

References:

1-45W600-99-1, 1-47W61 1-99-1, -3 W 7246D1 1-18, W 1802h70-22C, -22B, -23B, -27E, -27B

3-OT-SYSO99A Revision 9 Pge4ot47 PROGRAM Watts Bar Operator Training II. COURSES A. License Training B. Non-License Training III. TITLE Reactor Protection System (RPS)

IV. LENGTH OF LESSON A. License Training 6 hours6.944444e-5 days <br />0.00167 hours <br />9.920635e-6 weeks <br />2.283e-6 months <br />

1. Non-License Training 6 hours6.944444e-5 days <br />0.00167 hours <br />9.920635e-6 weeks <br />2.283e-6 months <br /> V. TRAINING OBJECTIVES 0 0 D 0 C,) (I)
1. Explain the purpose of the Reactor Protection System.
2. Sketch a basic drawing of the Solid State Protection System.
3. Describe the actions that take place when a reactor trip is generated at X X X 100% power.

x x 4. Identify the functions which rely on ESFAS for initiation.

Explain how the fire pumps would be affected by an SI signal.

x 6 Briefly describe the inputs to the SSPS.

x 7 Deleted.

8 Briefly discuss the input relays, Logic Section and Output Section of x x x x the SSPS.

9 Explain how the two trains of SSPS are interconnected.

10 Describe the two ways by which the SSPS opens the Reactor Trip breakers.

11 Describe the four basic outputs of the SSPS.

x 12 Explain the purpose of the reactor trip bypass breakers and how their use is made fail safe.

3-OT-SYSO99A Revision 9 Page5of47 0 0 D 0 0::

U) U) 13 Describe the causes of General Warning on SSPS 14 Identify where General Warning indications can be found.

15 Identify the SSPS equipment which can be tested.

16 Describe operator actions prior to allowing testing of SSPS train.

17 Identify the Reactor trips and give setpoints and list logic required for the Reactor trips.

18 Given the condition/status of the Reactor Protection system/component and the appropriate sections of Tech Specs, determine if operability requirements are met and what actions, if any, are required.

19 Deleted x x x x 20 Deleted 21 Deleted